SlideShare a Scribd company logo
1 of 147
4. A 45-yesr-old male vehicular accident patient was admitted at the ICU for ventilator
support after he was brought to the ER unconscious. Prior to the accident, he was
previously well. He was stable until the 4th ICU day when he had fever and yellowish
secretions. Chest x-ray showed new infiltrates on the left mid-lung field.
Recommended initial antibiotic treatment (HPIM p. 2139):
a. IV ampicillin-sulbactam
b. IV linezoid
c. Vancomycin
d. Ceftazimide + IV levofloxacin
5. A 57-year-old male was admitted because of hemoptysis amounting to 1 ½ cups. He
had a history of recurrent hemoptysis over the last 10 years. Chest x-ray showed
bilateral basal cystic lucencies. Chest CT scan showed bilateral dilated airways at the
bases. Fiberoptic bronchoscopy showed bleeding coming from both basal air passages.
Appropriate management for this patient (HPIM p. 2144)
a. Bronchial arterial embolization
b. Surgical resection
c. Forgarty catheter insertion
d. Pulmonary physiotherapy
6. A 67-year-old male, non-smoker consulted at the OPD because of occasional
shortness of breath and chest wheezing 1-2x a month. On PE, patient had clear breath
sounds . Chest x-ray was unremarkable and pulmonary function test showed more
than 15% improvement post-bronchodilator in FEV₁ and FEV₁ %. Management for this
patient is (HPIM p. 2113):
a. ICS low-dose + β₂ agonist prn
b. LABA + ICS low-dose + β₂ agonist prn
c. Inhaled β₂ agonist
d. Oral β₂ agonist
7. A 55-year-old male was admitted because of low-grade fever of 3 weeks associated
with cough productive of greenish foul-smelling phlegm. PE showed rales at the right
lung base and clubbing. Chest x-ray showed cavity with an air fluid level at the right
base . The patient was given 6 weeks of clindamycin but symptoms persisted and x-ray
findings remained the same. Management of this patient is (HPIM p. 2146):
a. 4-6 weeks of another antibiotic
b. TB treatment
c. Surgery on lesion
d. Anti-fungal treatment
Gynecology consultant plan was to do surgery for the myoma, but only after the
medical problem was managed appropriately. Your management of the patient prior
to surgery is (HPIM p. 2176):
a. Low molecular weight heparin
b. Fibrinolysis with rTPA (tissue plasminogen activator)
c. IVC filter insertion
d. Fresh frozen plasma transfusion
1. The following condition/s can result to hypokinetic pulses:
T a. Left ventricular failure
T b. Shock from massive blood loss
T c. Large perincardial effusion
F d. Mitral regurgitation
F e. Aortic regurgitation
2. Proposed mechanism/s of cardiac arrhythmias:
a. Alterations in impulse initiation
b. Automacity
c. Early after depolarizations and triggered automaticity
d. Abnormal impulse conduction
e. Phase 4 depolarization
3. Causes of heart failure
T a. Myocardial infarction
F b. Pericarditis
T c. Thyrotoxicosis
F d. Dissecting aortic aneurysm
T e. Chronic anemia
4. Which of the following statement/s is/are true regarding diagnostic examinations in heart
failure?
T a. New-onset heart failure warrants a complete blood
count, urinalysis, BUN, creatinine, liver enzymes and electrolytes.
F b. The most useful index of LV function is the EF calculated by the end-diastolic volume
divided by stroke volume.
T c. the gold standard for assessing LV mass and volume is the cardiac MRI
T d. Both B-type natriuretic peptide (BNP) and N-terminal pro-BNP are sensitive markers for
heart failure even in those with preserved EF
F e. A normal ECG virtually excludes LV systolic dysfunction.
5. Features of peripartum cardiomyopathy
T a. Prognosis depends on whether the heart size returns to normal after the first episode
of CHF
T b. May develop during the last trimester of pregnancy or within 6 months of delivery
F c. Typical patients are nulliparous and >30 years
F d. clinical outcome is dependent on whether the delivered via caesarian section or
spontaneous normal vaginal delivery
T e. the cause is unknown
6. Cardinal manifestations of acute pericarditis:
T a. Pain
T b. Pericardial friction rub
T c. Pericardial effusion and tamponade
T d. Paradoxical pulse
F e. Shortness of breath
8. Which of the following is/are risk of factor/s for stroke in a patient with atrial fibrillation?
F a. Obesity
F b. Mitral Regurgitation
T c. Diabetes Mellitus
T d. Spontaneous echo contrast
F e. Market right atrial enlargement
9. Unstable angina is defined as defined as angina pectoris or equavalent chest discomfort with the
following features:
T a. it occurs at rest (or with )usually fasting >10 min
T b. it is severe and of
T c. it occurs with a crescendo pattern (i.e. distinctly more severe prolonged or frequent than
previously)
F d. positive cardiac
F e. BP <90/ 60mmhg requiring inotropic support
10. Which of the following is/are ECG clue/s supporting the diagnosis of ventricular
tachycardia?
T a. Concordance of QRS complex in all precordial leads
F b. Frontal plane axis + 90 to 110 degrees
T c. QRS duration >140ms for RBBB type V1 morphology
T d. AV dissociation (atrial capture, fusion beats)
T e. RS dominant S in V6 for RBBB VT
11. Patients with the following congenital heart disease/s should be advised to avoid
pregnancy:
T a. VSD with Eisenmengerization
T b. Congenital aortic stenosis
T c. Coarctation of the aorta
F d. Large patent ductus arteriosus
T e. Marfan syndrome
12. Features of cardiogenic shock:
T a. Prompt reperfusion, efforts to reduce infarct size, ad treatment of ongoing ischemia
and other complications of MI appear to have reduced its incidence from 20% to about 7%
F b. Inferior location of the myocardial infarction increases the risk
T c. typically , at least 40% of the myocardium is damaged by old scars and new infarcts
F d. Plans for percutaneous transcoronary intervention should be delayed until patient is
stabilized via inotropics
T e. typically, patients who develop cardiogenic shock have severe multivessel coronary
artery disease with evidence of “placemeal” necrosis extending outward from the original
infarct zone

13. Which of the following is/are true about the patterns and causes of hypertension?
T a. Blunting of the day-night BP pattern is seen in sleep apnea and autonomic dysfunction
T b. white coat hypertension is associated with sustained hypertension and target organ
damage
F c. BP tends to be higher at noontime until late in the afternoon
T d. thyrotoxicosis leads to systolic hypertension while hypothyroidism causes diastolic
hypertension
F e. Night time “dips” in BP has been shown to increased cardiovascular disease risk
14. Cause/s of systolic hypertension with widened pulse pressure:
T a. Hyperkinetic heart syndrome
F b. Hypothermia
F c. Hypothyroidism
T d. Aortic regurgitation
F e. Mitral regurgitation
15. Features of resistant hypertension:
F a. Abrupt increase of BP in a patient with underlying hypertension
F b. More common in young patients
T c. May be related to nonadherence to therapy, obesity and excessive alcohol intake,
and use of any number of nonprescription and prescription drugs
T d. refers to patients with BPs persistently >140/90 mmHg despite taking three or
more antihypertensive agents, including a diuretic, in reasonable combination and at
full doses.
T e. Evaluation of patients include home BP monitoring to determine if office Bpare
representative of the usual BP.
16. Features of aortic aneurysms:
T a. Many patients with aortic aneurysms have co-existing risk factors for
atherosclerosis
F b. Cystic medial necrosis characteristically affects the distal aorta
T c. Familial clustering of aortic aneurysms occur in 20% of patients, suggesting a
hereditary basis for the disease
T d. the infectious causes of aortic aneurysms include syphilis, tuberculosis, and other
bacterial infections.
F e. vasculitides associated with aortic aneurysm include Takayasu’ s arteritis and giant
cell arteritis, which may cause proximal ascending aorta and descending thoracic aorta
17. Major manifestations of Rheumatic Fever based on Revised Jones Criteria:
T a. Carditis
F b. Monoarthritis
F c. Myaglia
T d. Subcutaneous nodules
T e. Chorea
18. Clinical variables that increase the likehood of deep venous thrombosis:
T a. Active cancer
F b. Bilateral calf swelling
T c. Pitting edema
T d. Surgery within 4 weeks
F e. Non-healing wound at the toes
19. Organisms commonly involved in infective endocarditis:
T a. Staphylococcus
F b. Pseudomonas aeruginosa
T c. Enterococci
F d. Corynebacterium
T e. Streptococcus
20. Clinical presentation of STEMI:
T a. In up to ½ of cases, a precipitating factor is present
F b. Pain is the most common presenting complaint and is uniformly present
T c. The precordium is usually quiet
T d. Paradoxic spitting of 52
T e. Fever
21. Clinical manifestations of acute rheumatic fever:
T a. There is a latent period of -3 weeks (1-5 weeks) between the precipitating group A
streptococcal infection and the appearance of the clinical features
F b. Chorea and indolent carditis may follow prolonged latent periods lasting up to 12
months
T c. A streptococcal infection is commonly subclinical, in these cases it can only be
confirmed using streptococcal antibody testing
F d. Carditis is more commonly present than polyarthritis
T e. Erythema marginatum and subcutaneous nodules are now rare, being found in
<5% of cases
22. Clinical features of systemic arterial hypertension:
F a. cardiac compensation is by eccentric left ventricular hypertrophy
F b. the ECG is more sensitive than 2D echocardiography for detecting left ventricular
hypertrophy
F c. it is an independent predisposing factor for coronary artery disease, stroke, renal
disease and peripheral vascular disease but not heart failure
T d. Opthalmoscopic examination provides the opportunity to observe the progress of
vascular effects of hypertension
T e. systolic blood pressure has a greater effect on morbidity and mortality than
diastolic blood pressure
23. Features of mitral valve prolapse:
F a. In most patients, the cause is unknown, but in some it appears to be genetically
determined elastin disorder
T b. A frequent finding in patients with heritable disorders of connective tissue
F c. the anterior leaflet is usually more affected than the posterior leaflet
T d. may occur as a sequel to acute rheumatic fever
T e. More common in females and occurs most commonly between the ages of 15
and 30 years
2.What is cause of falsely negative treadmill exercise test?
a. Abnormal serumpotassium
b. Use of cardioactive drugs (e.g. digitalis, anti-arrhythmic agents)
c. Obstruction limited to the circumflex coronary artery
d. Asymptomatic men under the age of 40
3. Most common site of focal spasm in Prinzmetal’s variant angina
a. Proximal left anterior descending coronary artery
b. Distal left anterior descending coronary artery
c. Left circumflex artery
d. Right coronary artery
4. Gold standard in the assessment of myocardial viability
a. SPECT
b. Cardiac MRI
c. PET Scan
d. Stress Echocardiography
6.
a.
b.
c.

Not a feature of atrial septal defects (ASD):
Occurs more frequently in females
Ostium primum defects type are common in Down’s syndrome
Surgical closure should be carried out in patients small defects and trivial left to
right shunts and in those with severe pulmonary vascular disease wihtout a
significant left-to-right shunt
d. ASD of the sinus venosis or ostium secundum types rarely die before the fifth
decade.
7. Not a feature of ventricular septal defect (VSD):
a. Patients with large VSD and pulmonary hypertension are those at greatest risk for
developing pulmonary vascular obstruction
b. Large defects should be corrected surgically early in life when pulmonary vascular
disease is still reversible or not yet developed
c. In patients with severe pulmonary vascular obstruction (Eisenmenger syndrome),
symptoms in adult life consist of exertional dyspnea, chest pain, syncope, and
hemoptysis
d. Spontaneous closure is common even in moderate-sized defects early in adulthood
8. NOT a pharmacologic property of amiodarone:
a. Class III agent delays repolarization due to inhibition of potassium current or
activation of depolarizing current
b. Useful in atrial fibrilliation, ventricular tachycardia and sinus bradycardia
c. Potential toxic effects include both hypothyroidism, chronic lung disease, and liver
function test abnormalities
d. Non-inferior to automatic implantable cardioverter defibrillator indecreasing
arrhythmetic recurrence and deaths
9. In patients with hypertrophic cardiomyopathy, this type of drug amellorates angina
pectoris and syncope in 1/3 to ½ of patients:
a. Nondihydropyridine calcium channel blockers
b. Ace inhibitors
c. Beta blockers
d. Diuretics
10. A major feature of metabolic syndrome:
a. FBS ≥ 110mg/dL
b. HDL in men < 50mg/dL
c. Blood pressure > 130/90
d. Triglycerides > 200mg/dL
11. Hallmark of the restrictive cardionyopathies
a. Systolic dysfunction
b. Myocardial fibrosis
c. Abnormal diastolic function
d. Concomittant endocardial involvment
12. Not a treatment of Prinzmetal’s angina:
a. ASA
b. Nitrates
c. Calcium channel blockers
d. Alpha-adrenoceptor blockers
13. Not an absolute contraindication to the use of fibrinolytics in STEMI:
a. History of cerebrovascular hemorrhage at any time
b. Nonhemorrhagic stroke 5 years ago
c. BP > 180/110mmHg
d. Active bleeding excluding menstruation

14. Most frequent arrythmia seen in alcoholic cardiotoxicity or “holiday heart
syndrome”
a. Ventricular Tachycardia
b. Frequent PVCs
c. Atrial flutter
d. Atrial fibrillation
15.Not cardinal manifestation of severe aortic stenosis
a. Chest pain
b. Syncope
c. Edema
d. Congestive heart failure
16. Most common congenital cardiovascular cause of hypertension:
a. Bicuspid aortic Valve
b. Coarctication of the Aorta
c. Patent Ductus Arteriosus
d. Coronary Arteriovenous Fistula
17. Best diagnostic modality in pericardial effusion:
a. Chest x-ray
b. Myocardial perfusion imaging
c. 2D echocardiogram
d. Cardiac catherization
20. Hormone found to be helpful in differentiating dyspnea caused by cardiac vs other
etiologies
a. BUN
b. BNP
c. ANP
d. D-Dimer
20. Pharmacologic agent found to reduce mortality in heart failure patients:
a. Digoxin
b. ACE inhibitors
c. Salt restriction
d. Isosorbide mononitrate

21. Management of acute myocardial infarction:
a. Aspirin is effective across the entire spectrum of acute coronary syndromes
b. Nitrates increase preload, thus decreasing myocardial oxygen demand
c. Morphine causes venous pooling leading to increased cardiac output
d. Supplemental oxygen is helpful in decreasing the size of the infarcted myocardium
even in patients with normal arterial O2 saturation
22. What is the EKG diagnosis?
a. First degree atrioventricular block
b. 2nd degree atrioventricular block Type 1
c. 2nd degree atrioventricular block Type 2
d. 3rd degree atrioventricular block
23. A feature of mitral stenosis:
a. VT is most common cause of death
b. Atherosclerosis remains the leading cause of mitral stenosis
c. The normal mitral valve orifice is 2-4cm2
d. Percutaneous valvuloplasty is treatment of choice for symptomatic patients, unless
technically not amenable
25. Not present in significant right ventricular infarction:
a. Jugular venous distention
b. Kussmaul’s sign
c. Bilateral mid to basal rales
d. Hepatomegaly
26. Drug of choice for medical management of aortic aneurysms
a. Beta blocker
b. Ace inhibitors
c. Calcium channel blocker
d. nitrates
27. Mechanisms of action of thienopyridines:
a. Inhibition of thromboxane synthesis
b. Inhibition of the clotting factor Xa
c. Inhibition of the P2Y12 ADP receptor
d. Inhibition of the IIIB/IIIA receptor
28. A principle of treadmill exercise testing:
a. Sensitivity for coronary disease is the same as coronary angiogram
b. Specificity for coronary disease is unaffected by the presence of LVH, digoxin use,
or resting ST-T abnormality
c. Contraindications include active myocarditis , symptomatic severe aortic stenosis
and early acute myocardial infarction
d. Routine screening is recommended for all adults over 30 years of age
29. Not a clinical manifestation of orthostatic hypotension:
a. Polypharmacy with antihypertensive or antidepressant drugs is often noncontributory
b. After physical deconditioning such as after prolonged illness with recumbency,
especially in elderly individuals with reduced muscle tone
c. Occurs in hypovolemic states from diuretics, excessive sweating, diarrhea,
vomiting, hemorrhage or adrenal insufficiency
d. Falls in 10 mmHg systolic or 29 mmHg diastolic blood pressure on standing is
diagnostic when due to autonomic dysfunction from various neurologic causes
30. Feature of Atrial Flutter:
a. Atrial flutter does not require anticoagulation because of low thromboembolic
risk.
b. Cardioversion is not effective in terminating hemodynamically tolerated
tachyarrhythmias, and amiodarone is therefore preffered
c. Atrial flutter will not cause tachycardia-induced severe left ventricular dysfnction
d. Rate control with calcium antagonist, betablockers or lanoxin maybe difficult
31. Pathophysiology of hypertension:
a. Both hypertropic (increased cell number increased cell size and increased
deposition of intercellular matrix) and eutrophic (no change in the amount of
material in the vessel wall) vascular remodelling result in decreased lumen size and
contribute to increased peripheral resistance
b. The renin-angiotensin-aldosterone system contributes to arterial pressure
regulation via angiotensin II which causes vasoconstriction, and aldosterone which
causes natriuresis.
c. Stiffer arteries in hypertension, particularly in arteriosclerotic patients results in
high systolic blood pressures and narrow pulse pressures.
d. Pheochromocytoma is an example of hypertension related to increased salt
retention.
32. A characteristics of secondaryhypertension:
a. Hyperthyroidism causes systolic hypertension with a narrow pulse pressure
b. Beta blockers are contraindicated in bilateral renal artery stenosis, or unilateral
stenosis in s solitary kidney
c. Surgical excision is the definitive treatment of pheochromcytoma and results in
cure in-90% of patients
d. Pheochromocytoma is the most common cause
33. A feature of hypertension in pregnancy:
a. For women with severe preeclamsia , BP >160/110 must be aggressively treated;
and delaying the delivery of the fetus and placenta as much as possible improves
maternal outcomes.
b. During pregnancy, a blood pressure of 140/90mmHg is considered to be
abnormally elevated and is associated with an increase in perinatal morbidity and
mortality
c. Magnesium sulfate is no longer used for the prevention and treatment of
eclamptic seizures
d. Methyldopa, ARBs and hydralazine are used most often because they have no
known adverse effect on the fetus.
34. A feature of aortic aneurysms:
a. If asymptomatic, surgery is indicated if the diameter is > 6 cm in thoracic
aneurysms and > 5.5 abdominal aneurysms.
b. Cystic medial necrosis is the most common cause of all aortic aneurysms.
c. Atherosclerosis is the condition not frequently associated with aneurysms of the
aortic archand descending thoracic aorta.
d. Abdominal aortic aneurysms occur more frequently in females.
35. Management of aortic dissections:
a. MPI with sestamibi is the most sensitive test.
b. Ct and MRI are accurate tests in this condition, while transesophangeal
echocardiography is generally unreliable for this coondition.
c. Systemic hypertension, cystic medial necrosis, Marfan’s syndrome are rare
predisposing factors.
d. Medical management is the preferred initial therapy for proximal Type A aortic
dissection.
36.A feature of neurogenic syncope:
a. beta-blockers are popular for the condition because trials have shown it to be
clearly effective
b. ICD implantation are indicated in these cases
c. Tilt table testing has a poorer sensitivity (20-75%) compared to its specificity
(nearly 90%), but specificity decreases with pharmacologic provocation
d. Usually accompanied by tonic clonic episodes
37. An indication for infective endocarditis prophylaxis:
a. Atrial septal defects deemed uncomplicated
b. s/p prosthetic mitral valve replacement
c. LV aneurysms
d. Patients with permanent pacemakers
38. Pulsus paradoxicus consist of a greater than normal inspiratory decline in systolic
arterial pressure by:
a. 10mmHg
b. 15mmHg
c. 20mmHg
d. 25mmHg
39. The significant ankle bachial index to suspect PAD:
a. <1
b. <1.5
c. <2
d. <2.5
40. Not a cause of aortic regurgitation:
a. Congenital
b. Marfan’s syndrome
c. Syphilis
d. Carcinoid
41. Cornerstones of modern therapy for heart failure with a depressed ejection fraction to
prevent disease progression
a. Diuretics and Beta-blockers
b. Beta blockers and ACE-inhibitors
c. Ace-inhibitors or Angiotensin receptor blockers
d. Diuretics and ACE-inhibitors
42. A late sign in cor pulmonale resulting from low cardiac output with systemic
vasoconstriction and ventilation-perfusion mismatches in the lung:
a. carvallo’s sign
b. Dyspnea
c. Clubbing
d. cyanosis
43. Most common maalignant cardiac tumor:
a. Sarcoma
b. Myxoma
c. Hemangioma
d. Lymphoma
44. Not a failure of cardiovascular syndrome caused by thiamine deficiency:
a. Characterized by high-output heart failure, tachycardia, and often elevated left and
right ventricular filling pressures
b. Major cause of the high-output state is vasomotor depression leading to reduced
response to thiamin and may take 1-2 weeks before effects are clinically apparent
c. Reduced systemic vascular resistance
d. Cardiac examination reveal a wide pulse pressure, tachycardia, a third heart sound,
and frequently, an apical systolic murmur.
45. Marks a turning point in the course of a patient with a mitral stenosis, associated
with acceleration of the rate at which symptoms progress:
a. Infective Endocarditis
b. LV dilatation
c. Permanent Atrial Fibrillation
d. MV orifice of <1.5 cm2
46. Not an indication for coronary angiography in chronic stable angina:
a. Patients with chronic stable angina pectoris who are severely symptomatic despite
medical therapy and who are being considered for revascularization, i.e., a
percutaneous coronary artery bypass grafting (CABG)
b. Patients with troublesome symptoms that present diagnostic difficulties in whom
there is a need to confirm or rule out the diagnosis of IHD
c. Patients with no evidence of ischemia on noninvasive testing and no-clinical or
laboratory evidence of ventricular dysfunction
d. Patients with known or possible angina pectoris who have survived cardiac arrest
47. Advantageof bioprosthetic and mechanical valve replacement:
a. Bioprosthetic valves are superior to mechanical prosthetic valves in durability
andthrombogenicity
b. Lifetime anicoagulation is warranted for mechanical prosthetic valves
c. Valve replacement is indicated for asymptomatic moderate mitral or aortic
regurgitation with well-preserved left ventricular systolic function
d. Prosthetic valves pose a low risk for infective endocarditis and antibiotic
prophylaxis is not indicated prior to dental procedures that may cause transient
bacteremia
1. A 55 year old female was admitted due to shortness of breath with associated
bipedal edema. Your resident saw the patient earlier and told you that the present
working impression is cor pulmonale. Which of the following point in the history
and PE does not contribute to the diagnosis?
a. Chronic productive cough for 3 months in each of two consecutive years
b. 30 pack year smoker up to present
c. Normal JVP
d. She has been on warfarin regimen for the past year with irregular intake for history
of deep venous thrombosis
2. The cardiology fellow was called to do a pericardiocentesis procedure on one of the
patients at the emergency room. As he was doing the procedure, he asks you, which
of the ff is not a feature of cardiac tamponade?
a. Hypotension
b. Soft or absent heart sounds
c. Fixed wide Split S2
d. Jugular venous distention with a prominent x descent and absent y descent
3. You were making rounds at the wards when the nurse referred a patient to you.
Apparently, the patient lost consciousness. What should you NOT do?
a. Patient should be placed supine with the head turned to the side
b. Clothing that fits tightly around the neck or waist should be loosened
c. Sprinkling cold water on the face may be helpful
d. Have the patient drink cold water
4. While on your duty, you were referred a patient with infective endocarditis. He has
been admitted for 5 weeks already but today, he became unstable complaining of
shortness of breath. There was a new murmur according to the intern. Your resident is
thinking the patient might need urgent heart surgery. Which of the following is not an
indication for cardiac surgical intervention in patients with endocarditis?
a. Moderate to severe congestive heart failure due to valve dysfunction
b. Persistent bacteremia despite multiple anti microbial therapy
c. Presence of effective but expensive microbicidal therapy (e.g. fungal or brucella
endocarditis)
d. S.aureus prosthetic valve endocarditis with intracardiac complication
5. 69/F with dizziness, on Imipramine for depression for a few years now. BP of
140/90, RR 22/min, HR variable rhythm. Cardiac monitor shows: what will you do?
a. Immediately start IV lidocaine bolus and drip
b. Cardiovert the patient with 200 J
c. Administer IV amiodarone
d. Give IV magnesium sulfate bolus and drip
6. You are making rounds with your OB consultant. In front of you is a 35 week AOG
pregnant patient that has been having elevated blood pressure since 2 days ago. Right
in front of you, the patient had a seizure. Blood pressure was taken and BP =
200/110mmHg. What do you do?
a. Give captepril25mg tab SL stat q15 minutes
b. Send the patient to the ICU for further monitoring and start her on Duvadilan drip
c. Give magnesium sulfate to the patient
d. Start oral ACE inhibitors
7. 60 year old male suddenly had dizziness and diaphoresis. BP is 80/50mmHg with
cold clammy extremities. ECG shows the following. What to do?
a. IV streptokinase
b. Give verapamil 5mg IV stat
c. Send to cathlab immediately
d. Provide electrical cardioversion
8. 45/M, hypotesive , cold and clammy with EKG below:
True about this patient’s condition:
a. If right-sided V3 and V4 leads show 1mm ST depression, it indicates RV infarction
from proximal occlusion of the night coronary artery
b. Elevated JVP with kussmaul’s sign and clear lungs are consistent with RV infarction.
IV fluids dobutamine and reperfusion therapy are appropriate
c. ST-elevation or right-sided V3 and V4 leads, elevated JVP and pulmonary ,
congestion should be treated with IV fluids, dobutamine and reperfusion therapy
d. Intraaortic ballon counterpulsation is contraindicated

9.
1. administer CPR
2. check for responsiveness
3. call for help,activate EMS
4. survey the scene
5. check pulse and breathing
a. 3-4-5-2-1 F
b. 4-2-1-3-5 F
c. 4-2-3-5-1 T
d. 1-2-3-4-5 F
e. 2-4-3-1-5 F

10. A 58/M, smoker, hypertensive , diabetic patient with dyslipidemia consulted at your
clinic. Based on the NCEP guidelines, the LDL target for this patient should be:
a. <200mg/dL F
b. <150mg/dL F
c. <100mg/dL F
d. <70mg/dL T
e. <50mg/dL F
11. 60/F w sudden dyspnea 2 hrs after BK amputation for diabetic foot ulcer. BP 150/100,
HR 96/min, RR 22/min soft S1, cold, clammy extremities. ECG shows:
Best treatment option is:
a. IV r-tPA thrombolysis
b. IV streptokinase thrombolysis
c. Cardiac catherization and angioplasty/stenting
d. IV streptokinase followed by cardiac catherization
7. Which of the following statements is/are TRUE of connective tissue diseases?
F a. Systemic lupus erythematosis and dermatomyocitis can be differentiated through skin
biopsy and direct immunofluorescence findings
T b. red plaques on the knuckles of the hands are characteristic of dermatomyocitis
F c. Presence of malar rash distinguishes lesions of SLE from dermatomyocytis
F d. perifungual erythema are common in scleroderma
F e. Scleroi…is
7. Which of the following is NOT characteristic of rosacea?
a. Flushing
b. Comedones
c. Red papules
d. Telangiectasia
9. Which of the following xanthomas is the LEAST specific cutaneous sign of
hyperlipidemia?
a. Xanthelasma
b. Planar xanthoma
c. Eruptive xanthoma
d. Tuberous xanthoma
Oral antihyperglycemic agents that reduce postprandial hyperglycemia
a. Alpha glucocidase inhibitor
b. Arrythin agonist
c. Bile and sequestrant
d. Megitnide
e. Thiazolidne
Antidiabetic agents contraindicated in patients with elevated plasma triglycerides
a. Alpha gluccosidase inhibitor
b. Bile acid sequestrants
c.
d. GLP-1 receptor agonist
e. Thiamidinadione
Differentiates HHS from DKA
a. Presence of nausea and vomiting
b. Anion gap
c. Serum potassium
d. More pronounced volume
e. Higher mortality rate
Microvascular complications of diabetes
a. Cataracts
b. Diabetic skin spots
c. Macular edema
d.
e. Sexual dysfunction
Mechanism for the development of chronic DM complications
a. Epignhetic charges in the affected cells
b. Non erythmatic formation of advanced glycosylation and products
c. Increased glucose metabolism in the sorbital pathway
d. Increased
e. Formation of diacylglycerol leading to activation of protein phase C
True of diabetic nephropathy
a. Diabetic patients with nephropathy usually have concomittant neuropathy
b. Only
c.
True of diabetic neuropathy
a. Smoking is a risk factor
b. Is a diagnosis of exclusion
c. Most common form is distal symmetric polyneuropathy
d. For polyneuropathy pain is usually present at most and worse at night
e. is a FDA-approved treatment
True according hypoglycemia
a. Most common cause are drugs
b. During
Associated with low radioiodine update
a. grave’s disease
b. Plummer’s disease
c.
d.
e. hypothyroidism
True of myeoderma coma
a. Still has highmortality rate despite treatment
b. Almost always occurs in the elderly
c. Hyperventilation plays a major
d. External
e.
Characteristics of thyroiditis
a. Acute thyroiditis is rare and due to
b. Diagnosis of
c. Silent thyroiditis occurs in patients with underlying autoimmune thyroid disease
d. Cytoxine induced thyroiditis is more common in women with TPD antibodies
e. Thyroid dysfunction is
Presentation of amodarone-induced
a.
b.
c.
d.
e.
Characteristics of goiters
a.
b.
c.
d.
e.
Increases risk for osteoporosis
a. Dementia
b. Alcoholism
c. Anorexia-nervosa
d. Type 1 DM
e. Rheumatoid arthritis
Causes of hypocalcemia in the critically ill patients
a. Acidosis
b. Blood transfusion
c. Heparin
d. Hypoalbuminuria
e. Pan
Patients with assymptomatic hyperparathyroidism whoare candidates for surgery
a. 55 year old male
b. BMD T score of -2.0
c. Creatinine
d. 24H urinary calcium 3 g/day
e. Serum calcium 16 mg/dl
Finding consistent with phaeochromocytomas
a. Highly vascular and tumors
b. Most all
c. Palpitations, headaches and produce sweating
d. Documentation of
e. Liberal salt intake and hydration are necessary treatments
Features of adrenal insufficiency
a. Most common cause is autoimmune adrenalties
b. Secondary adrenal insufficiency presents with loss of glucoccorticoid and androgen
secretion
c. Distinguishing feature of primary adrenal insufficiency is hyperpigmentation in
areas of friction
d. Hypercalcemia is found at presentation in 80% of patients with primary adrenal
insufficiency
e. Random serum cortisol measurements are limited diagnostic value
Features of acromegaly
a. Due to GH and AOF -1 hypersecretion the after epiphysis has cloased
b. Calorie
c. Prolcatin is elevated in 21%
d. Screening is done by getting random iOF-1 hormone levels
e.
Differentiates ectopic ACTH secretion from ACTH-secreting pituitary tumor
a. Gender of patient
b. Rapidly of onset
c. Levels of 24H urine free cortisol
d. Marked hypokalemia
e. High dose dexamethasone test
Characteristics of diabetes insipidus
a. Due to a decreased secretion of action of AVP
b. Overt dehydration on physical exam
c. “Bright spot” noted on T1-weighted midsagital MRI images
d. Weight loss occurs with water deprivation
e. Desmopresssin is appropriate for D1
Treatment of SIADH
a. Fluid intake that is 500 cc less than the urine output increases serum sodium by 12% per day
b. IV infusion of 3% hypertonic saline connects the sodium deficiency and removes
excess water by solute diureses
c. Demeclocycline produces a reversible form of nephrogenic diabetes insipidus
d. Blocks the antidiuretic effect of AVP on V2 and V1s receptors
e. Too rapid correction can be complicated by cerebral ionizing
Definite benefits of HRT
a. Cognitive dysfunction
b. Diabetes mellitus
c. sypmtoms
d. Osteoporosis
e.
Insulin biosynthesis secretion and action
a. C peptide is cleared more slowly than insulin and is a useful in differentiating
endogenous from ecogenous insulin occurs
b.
Diagnosis and screening for diabetes
a. Diabetes is defined as the level at which diabetic specific complications occur
b. FPG is the most reliable and convenient test for identifying assymptomatic
individuals HPG or HBsAg
c. Diagnosis of diabetes can be withdrawn when the glucose incidence reverts back
to normal
d. Screening is recommended for both type 2 DM
Factors that confer a very high risk of progression from prediabetes to diabetes
a. Age < 60 years
b. Family history of DM in any relative
c. Hypertension
d. Elevated total cholesterol
Pathogenesis of type 1 DM
a. Patients will have evidence of islet directed autoimmunity
b. In the majority, immunologic markers appear the triggering event and before diabetes
appears
c. Major susceptibility gene for type 1 DM is located in the HLA region of chromosome
d. Most individuals with diabetes will not have a relative with this disorder
Treatment goals for adults with type 2 DM
a. HBA 1c < 6.9% in most patients
b. Postgrandial capillary glucose < 110mg/dl
c. Triglyceride < 160 mg/dl
d. BP < 120/80 mmHg
Nutrition and physical activity in patients with diabetes
a. The components of optimal medical nutrition therapy are generally similar for T1 DM,
T2 DM and the general population
b. Nonnutrient sweeteners are allowed
c. Insulin and SUg patients are very prone to hypoglycemia during exercise
d. Proliferative diabetic retinopathy is a contraindication to vigorous exercise
Rapid acting insulin
a. Aspart
b. Detemir
c. Glargine
d. NPH
Explains the absence of ketosis in HHS
a. Insulin deficiency
b. Lower levels of
Differentiates DKA from single hyperglycemia
a. Glucose levels
b. Ketonemia
c. Ketonuria
d. Plasma osmolarity
Features of DKA
a. Insulin deficiency is necessary for diabetic
Laboratory abnormalities of DKA
a. total body stores of sodium, potassium, chloride and phosphorus are total body by
their serum levels
b. Elevated serum creatinine
c. Elevated serum amylase is indicative of salivary in origin
Metabolic acidosis is from ketone bodies and lactic acid
Possible unifying mechanism in the theories regarding the development of chronic
diabetic
a. Epignetic charges in affected cells
b. Formation of advanced glycosylation
c. Increased production of growth factors
d. Increased production of reactive oxygen appear in the melochondria
Correct conclusions from trials on glycemic control and development of chronic
complications
a. In the Diabetes Control and Complications Trial (DCCT) reduction in chronic
hyperglycemia can prevent microvicular complications in patients with type 2 DM
b. Benefits from improved glycemic control during the trials persisted even after the
study concluded and glycemic control worsened in both the DCCT and in the
United Kingdom Prospective Diabetes Study (UKPDS)
c. Benefits of an improved glycemic control occurred only in the tighter ranges of A1c
values
d. In UKPDS strict blood pressure control significantly reduced microvascular
complications but had no effect on microvascular complications
Features of opthalmologic complications of diabetes
a. Blindness is primarily the result of progressive diabetic retinopathy and the
development of cataracts
b. Proliferative diabetic retinopathy is characterized by retinal vascular
microaneurysm, biot hemorrhages and cotton-wool spots
c. Hypertension is also a risk for the development rethropathy
d. Genetic susceptibility
Management of diabetic retinopathy
a. There is in established diabetic nephropathy in the first 6 – 12 months of improved
glycemic control transient worsening
b. Toutine nondilated eye examination by the primary care provider or diabetes
specialist is detect diabetic eye disease opthalmologist needed
c. Exercise has not been conclusively shown to worsen proliferative diabetic
nephropathy
d. Aspirin therapy 650 mg/day
Differentiates nephropathy in T1 DM from nephropathy of T2 DM
a. Microalbuminuria may be present on diagnosis
b. More commonly with accompanying hypertension
c. Presence of microalbuminuria is predictive of progression of microalbiminuria
d. Albuminuria may be secondary to factors unrelated to the
Cardiovascular morbidity and mortality in patients with diabetes
a. Screening for coronary heart disease in assypmtomatic individuals with diabetes
controversial have not yet shown clinical benefit
b. In both DCCT and UKPDS, cardiovascular events were reduced during the trial and
remained low at follow up 10-17 years later trend for benefit in the first part of the
trial
c. Current recommendations target A1 c levels near to reduce cardiovascular events
d. Aspirin therapy (75-162 mg/day) is primarily recommended for secondary
prevention
Characterisitc of diabetic dyslipidema
a. Most common pattern is high triglyceride, low HDL c, small dose LDL
b. DM does not increase levels of LDL
c. Beneficial effects of LDL reduction are similar in diabetic and nondiabetic
populations
d. High dose niacin may worsen glycemic control and insulin resistance
Antihypertensive agents which can increase insulin resistance
a. Ace inhibitor
b. Calcium channel blockers
c. Central adrenergic antagonists
d. Thiazide
Features of infectious & dermatologic complications in patients with diabetes
a. Organisms that cause pulmonary infections are similar from that found in the
general population
b. Infections seen almost exclusively in the diabetic population include malignant
otitis externa
c. “diabetic skin spots” result from minor trauma in the pretibial region and are
more common in elderly
d. Necrobiosis tipodice diabeticorum predominantly affects young females with type
1 DM
Which of the following emerging therapies is an important therapeutic option for
patients with T1DM
a. Whole pancreas transplantation
b. Panccreatic islet cell transplantation
c. SGLT-2 inhibitors
d. Banatric surgery
True regarding DM management in special situations
a. Target blood glucose levels in critically ill patients is 140-180 mg/dl
b. Total parenteral nutrition increases insulin requirements
c. Oral antihyperglycemia agents be will not efficacious for patients on
glucoccorticoids with FPG > 200 mg/dl
d. The most crucial period of glycemic control during pregnancy is during the second
term
Primary phyysiologic defense of the body against hypoglycemia
a. Decrease in insulin
b. Increase in glucagon
c. Increase in epinephrine
d. Decrease in cortisol
Treatment of hypoglycemia
a. Reasonable initial dose of glucose is 20g
b. Glucagon is effective for of hyoglycemia ineffective for glycogen-depleted individuals
also less useful for t2dm
c. Medical therapy with diazonide orr octreotinide is useful for endogenous
hyperinsulinemia from beta-cell tumors also for nontumor beta cell disorders
d. Administrations of uncooked cornstarch at bedtime may be necessary in some patients
Laboratory test for thyroid dysfunction
a. Exception, TSH alone is sufficient to exclude a primary abnormality of thyroid fx
b. Thyroxine-binding globulin are increased by estrogen androgens decrease TBG
c. TSI antibodies are mainly used to predict thyrotoxicosis fetal
d. Salicylates can transiently increase free thyroid hormone levels
Features of thyroid hormone resistance syndrome
a. Autosomal disorder due to a mutation in the TRB gene dominant
b. Usuallyasypmtomatic
c. Can present with tachycardia
d. Most patients not require levothyroxine replacement

Management of hypothyroidism
a. FT3 determination in diagnosis & management 25% with normal FT3 reflecting
adaption deiodenase responses
b. Liothyronine has no place for long term replacement
c. Patients who develop hypothyroidism after treatment of Grave’s disease require
replacement doses of LT4
d. Full relief from symptoms may take 3-6 months after normalization of TSH
Special treatment considerations of hypothyroidism
a. Levothyroxine treatment of clinically euthyroid patients with thyroid autoantibodies can
reduce the risk of preterm delivery
b. Levothyroxine dose requirements even in pregnancy increased by ≥ 50%
c. Elderly patients require 20% less levothyroxine than younger patients
d. Emergency surgery until euthyroidism is achieved generally safe
Features of Autoimmune hypothyroidism
a. More common in populations with chronic exposure to a low iodine diet
b. TPO and thyroiglobulin antibodies have a primary pathogenetic role and are
clinically useful markers of thyroid autoimmunity
c. Pericardial effusions occur in up to 30% and commonly leads to cardiac
compromise
d. Thyroid-associated opthalmopathy occurs in 5% of patients
Causes of secondary hyperthyroidism
a. Gestational thyrotoxicosis
b. Silent thyroiditis
c. Struma ovani
d. Iodine excess
Features of Graves disease
a. Minor for grave’s disease, major for the opthalmopathy smoking is a risk factor for
Grave’s disease
b. Worsens in 3-6 mos., plateaus in 6-12 mos. Climical course of the opthalmopathy
that of the thyroid disease
c. TSI antibodies do not correlate directly with thyroid hormone levels
d. May present with pretibial myxedema
Treatment of Graves disease
a. All antithyroid drugs inhibit the function of TPO reduce thyroid antibody levels,
and enhance remission rates
b. FT4 is used as a basis to the titrate doses of the antithyroid drugs
c. PTU has a prolonged radioprotective effect and must be stopped several weeks
before radioactive treatment
d. It is often possible antithyroid drugs during the first term of pregnancy
Characteristic of sick euthyroid syndrome
a. Most common pattern is low T3 syndrome
b. Major causes is cytokines
c. Progressive HIV disease is associated with a decrease in T3
d. Magnitude of fall in T3 levels correlates with the severity of the illness
Factors altering thyroid function in pregnancy
a. Rise of circulating hCG in the first term is accompanied by a reciprocal fall in TSH
b. Estrogen induced rise in TBG during the first trimester
c. Increased thyroid hormone metabolism by the placenta
d. Increases urinary iodide excretion
Characteristics of thyroid cancer
a. Papillary thyroid cancer has a propensity for spread
b. Follicular thyroid cancer be diagnosed by thyroid FNAB
c. Medullary thyroid cancer is sensitive to RAI
d. Anaplastic thyroid cancer is always stage IV
Most common cause of drug-induced osteoporosis
a. Alcohol
b. Glucoccorticoids
c. Heparin
d. levethyroxine
Management of Osteoporosis
a. Primary use of biochemical markers is to predict fracture risk independently of bone
density monitoring to reaponse
b. Preferred source of calcium is dietary sources
c. Weight-bearing exercises in postmenopausal women prevents bone loss but does not
appear to result in substantial gain of bone mass
d. Supplementation with vitamin K are helpful
Pharmacologic therapy for osteoporosis that promotes bone formation
a. Bisphosphonates
b. Calcitonin
c. Denosumab
d. Teriparitide
Causes hypercalcemia and low serum PTH levels
a. granulomulous disease
b. Lithium toxicity
c. Parathyroid hyperplasia
d. Tertiary hyperparathyroidism
Features of hyperparathyroidism
a. Hereditary hyperparathyroidism may be found in MEN-2A and MEN-1
b. Most common cause is parathyroid adenoma
c. More than half are asymptomatic
d. Incidence peaks between the 3rd and 4th decade
Approach to a patient with an adrenal incidentaloma
a. Diagnostic evaluation should be done for > 1 cm adrenal masses
b. For the diferentation of benign from malignant adrenal masses, imaging is relatively
sensitive though specificity is suboptimal
c. FNA to rule out malignancy, to establish malignancy need to demonstrate mets may
cause needle canal metastase of ACC
d. Adrenal masses with confirmed hormone excess and usually treated surgically
Features of adrenocortical cancer
a. 60-70% overproduce hormones and produce clinically syndromes, usually inapparent
due to defective synthesis
b. Metastasis most commonly occurs to the liver
c. Tumor size > 8 cm is associated with a high risk of recurrence
d. Five years survival rates are 30-40%
Principles of estrogen therapy in menopause
a. Oral estrogen is better than transdermal estrogen in managing hot flushes
b. It is useful in the management of hypertriglyceridemia
c. It is useful in patients with thromboembolic disease
d. It increases the risk of endometrial cancer
Characteristics of infertility
a. Defined as inability to conceive after 12 months of unprotected sexual intercourse
b. Female factors account for 58% of infertility
c. Modifiable risk factors include alcohol caffeine, obesity, and smoking
d. Time for evaluation, correction and expectant movement can be longer in women > 35
years.
Who has diabetes?
a. 40F with fasting blood sugar of 126 mg/dL taken once
b. 35M with prebreakfast cbg 130 mg/dL
c. 28F with 50 g OGCT 150 mg/dL
d. 18M with a standardized HBA 1c of 6.8%
29F was admitted for decrease
30F is for fertility work up. She is assymptomatic and PE is unremarkable. Her TSH is 7
uU/L, (NV 0.4 – 4 uU/L) and FT4 is 13 uU/L (NV 12 – 20 uU/L) TPO antibodies are
positive. What should you do next?
a. Request for a total T3
b. Request for thyroid scan
c. Repeat TSH, FT4 after 3 months
d. Start levothyroxine
26M with weakness of both thighs was noted to have a TSH 0.01 uU/L (NV 0.4 – 4
uU/L) and FT4 20 uU/L (NV 12 – 22 uU/L). Physical examination is unremarkable. What
should you do next?
a. Request for total T4
b. Request for free T3
c. Start PTU 50 mg TID PO
d. Reassure the patient and ask him to follow up after 6 – 12 months
45M has 2 cm firm nodule on the right thyroid lobe on palpation THS is uU/L (NV 0.4-4
uU/L) FNAB shows follicular tumor. What should you do next?
a. Do a thyroid scan
b. Do an ultrasound guided FNAB
c. Monitor by ultrasound
d. Send the patient for surgery
Who should be given medical therapy for osteoporosis?
a. Z score -2.0
b. 30F with bronchial asthma who was given Prednisone 30 mg OD PO x 7 days
c. 60F with BMI 16 and T score -1
d. 62M with family history of leg fracture
25F with BP 160/100 consulted for work up. She is on losartan 50 mg OD PO and
amlodipine 5 mg OD PO. Serum creatinine is 90 ug/dL. Na 140 meq/L, K 3.2 meq/L. what
should you do need?
a. Request for plasma aldosterone-renin ratio
b. Get on abdominal CT scan
c. Correct serum potassium first
d. Start spirinolactone 50 mg OD PO
28F complains ofamenorrhea in 6 months. Work up showed a fasting serum orolactin
150 ug/L. She has a history of thyroid surgery 2 years ago. Currently she is not taking
any medications. Physical examination is unremarkable except for dry thick skin. What
should you do advice the patient? (HPIM 18th and chapter 139)
a. Repeat prolactin levels using sample dilution
b. Request for THS and FT4
c. Request for a sellar FT4
d. Start bromocriptiine 2.5 mg/tab ½ tab OD PO
MTF 1. Features of Erythropoiesis
F a. the first morphologically recognizable erythroid precursor in the bone marrow is
themyeloblast
F b. In the absence of Granulocyte Colony Stimulating Factor (GCSF), erythroid
precursors willnundergo programmed cell death (apoptosis)
T c. EPO governs day-to-day production of red cells
T d. Average life span of red cells is 100-120 days
T e. The fundamental stimulus for EPO production is the availability of O₂ for tissue
metabolic needs
MTF 2. Oxygen affinity og hemoglobin in different situations
T a. Hemoglobin has a lower oxygen affinity among acidotic septic patients
F b. Hemoglobin has a lower oxygen affinity at higher pH (Bohr effect)
T c. Immediately after transfusion of PRBC, there may be decrease in O2 release by the
hemoglobin
F d. Increase in 2,3 BPG increases hemoglobin oxygen affinity
F e. Better oxygen delivery is noted ay higher attitudes
MTF 3. Differential diagnosis of microcytic hypochromic anemia
T a. Thalassemia
T b. Lead Poisoning
T c. Iron deficiency Anemia
F d. Diphyllobotrium latum infestation
T e. Anemia of chronic disease
MTF 4. Compatible with Iron deficiency anemia
F a. Microcytic hypochromic red cells low serum iron, normal ferritin level
T b. Microcytic hypochromic red cells, low serum iron, low ferritin level
F c. Microcytic hypochromic red cells, normal serum iron, increased ferritin
F d. Microcytic hypochromic red cells, normal serum iron and TIBC
T e. Microcytic hypochromic red cells, low serum iron, high
MTF 5. Microcytic hypochromic cells with low serum iron and normal or increased
bone marrow iron stores may be found in the following:
F a. 32 y/o female with alpha thalassemia trait
F b. 19 year old heavily menstruating woman with iron deficiency anemia
T c. 55 year old male with rheumatoid arthritis
F d. 28 y/o male with chronic lead positioning
T e. 49 year old female with systemic lupus erythematosus
MTF 6. Features of hypoproliferative anemias
T a. Acute and chronic inflammation is the most common of these
F b. Anemia of inflammation, like iron deficiency, is related in part to abnormal
erythropoietin response
T c. Renal disease, cancer and hypometabolic states are characterized by an abnormal
erythropoietin response to the anemia
T d. These abenia are assocciated with normocytic and normochromic red calls and an
inapppropriately low reticulocyte response
F e. Most commonly associated with exposure to benzene derva
MTF 7. Diseases associated with intravascular hemolysis
T a. Paroxysmal nocturnal hemoglobinuria
F b. Hemolytic uremic syndrome
T c. Hemoglobinupathies
F d. Hypersplenism
F e. Microangiopathic hemolytic anemias
MTF 8. Characteristic of Unsatable Hemoglobin Variants
T a. Should be suspected in patients with nonimmune hemolytic anemia, jaundice,
splenomegaly, or premature billiary tract disease. Or leg ulcers
F b.Severe hemolysis usually presents in adults
F c. Spontaneous mutation ucommon
T d. The peripheral blood smear often show anisocytosis, abundant cells with punctate
inclusions, and irregular shapes (i.e., poikilocytosis).
T e. Best test for diagnosing unstable hemoglobins are the Heinz body preparation and
the isopropanol or heart stability test.
MTF 9. Guidelines on splenectomy in patients with hemolytic diseases
T a. Avoid splenectomy in mild cases
F b. delay splenectomy until at least 10 years of age
T c. Antipneumococcal vaccination before splenectomy is imperative
F d. Anticoagulation may be required.
T e. Hereditary Spherocytosis patients often may require cholescystectomy
MTF 10. Characteristic of Vitamin B12 and associated deficiency resulting to
Megaloblastic Anemia
T a. Cobalamin (Vitamin B12) is synthesized solely by microorganisms.
T b. Only source for humans is food of animal origin, e.g. meat, fish, and dairy
products.
F c. Body stores are of the order of 2-3 mg, sufficient for 3-4 months if supplies are
completely cut off
F d. Vegetables, fruits and other foods of non-animal origin also contain cobalamin
T e. Most common among individuals of Hindu religion
MTF 11. The M Component noted in Serum Protein Electrophoresis may be seen in ff
condition/s
T a. multiple Myeloma
T b. Chronic lymphocytic leukemia
F c. Thalassemia
F d. Chronic renal disease
T e. Breast Cancer
MTF 12. Characteristics of Multiple myeloma
T a. Malignant proliferation of plasma cells derived from a single clone
T b. Overexpression of myc or ras genes hass been noted in some cases
F c. Inversion of chromosome 16 was also observed in certain varieties
T d. Management with Thalidomide and Dexamethasone have shown better survival
F e. Translocation of chromosome 9:22 is associated with worst prognosis
MTF 13. The following conditions predispose a patient to development of Acute
Myelogenous Leukemia
T a. Myelodysplastic Syndrome
T b. Polycythemia Vera
F c. Thalassemia
T d. Paroxysmal Nocturnal Hemoglobinuria
F e. Iron Deficiency Anemia
MTF 14. The following are part of the initial laboratory evaluation of adult patients
with acute myeloid leukemia
T a. CBC with manual differential cell count
F b. Ultrasound of whole abdomen
T c. Bone marrow aspirate and biopsy (morphology cytogenetics, flow
cytometry, molecular studies)
T d. HLA typing of patient, siblings, and parents for potential allogeneic SCT
F e. Urinalysis
MTF 15. Ccharacteristic of myelophthisic anemia
T a. A common finding in the peripheral blood smear is a shift to the left of the granulocytic series and
presence of nucleated red cells, these findings are suggestive of the diagnosis
T b. Usually the infectious or malignant underlying processes are obvious in secondary causes
F c. Leukopenia
F d.
F e.
MTF 16. Conditions with pancytopenia and cellular bone marrow:
F a. Aplastic anemia
T b. Paroxysmal nocturnal hemoglobinuria
T c. Myelodysplasia
F d. Dyskeratosis congenita
T e. Hairy cell leukemia
MTF 17. Drugs definitely reported to cause isolated thrombocytopenia
F a. Acetyl salicylic acid
T b. Trimethoprim/sulfamethoxazole
T c. Danazol
T d. Paracetamol
F e. Phenylpropanolamine
MTF 18. Consequences of Neutropenia
MTF 19. treatment of Immune Thrombocytopenic Purpura (ITP)
F a. Patients with platelet counts greater than 30,000/L should be started on oral
steroids
T b. Traditional OPD management of ITP this has been prednisone at 1 mg/kg
T c. Rh₀(D) immune globulin must be used only in Rh-positive patients
T d. Refractory ITP may benefit from anti-CD 20 agents
T e. Oral eltrombopag shows potential benefits for acute ITP
MTF 20. The following principles should be applied in the management of bleeding
disorders
F a. Rest
T b. Ice or cold compress over affected areas
F c. Warm compress over affected areas
T d. Elevation of affected extremity
F e. Aspiration of bleeding joints
MTF 21. Features of venous thrombosis
F a. Majority of cases are secondary to hereditary causes
F b. Females are at greater risk for venous thrombosis than are males
T c. High prevalence of Factor V Leiden was reported in Arabs
F d. factors 8 and vWF are decreased during pregnancy thus predisposing to venous
thrombosis
T e. LMWH is recommended for acute thrombotic events
MTF 22. Conditions associated with both arterial and venous thrombosis
T a. Hyperhocystenemia
T b. Paroxysmal nocturnal hemoglobinuria
F c. Immobilization
F d. Activated protein C
T e. Polycythemia vera
MTF 23. In the setting of acute thrombosis or anticoagulation, working up for a
hypercoagulable state should take into consideration:
T a. Coumadin reduces protein C and S levels
T b. Heparin can reduce antithrombin levels
F c. Heparin and Coumadin have no effect on testing for lupus anticoagulant and APC
T d. Sepsis is associated with reduction in levels of protein C, Protein S,, antithrombin
F e. Work ups may be done anytime
MTF 24. Indications forHematopoietic Stem cell transplantation (HSCT)
T a. Acute Myelogenous Leukemia
T b. Breast Ca
T c. Severe combined immunodeficiency
T d. Very Severe Aplastic Anemia
F e. Diffuse B Cell Lymphoma, newly diagnosed
1.
a.
b.
c.
d.

The world Health Organization (WHO) defines anemia as
Hemoglobin level <130g/L (13 g/dL) in men
Hemoglobin level <120g/L (12 g/dL) in men
Hemoglobin level <130g/L (13 g/dL) in women
Hemoglobin level < 120g/L (13 g/dL) in pregnant women
3. The main iron transport protein
a. Transferrin
b. Apoferritin
c. Albumin
d. globulin
4. The “gold standard” for iron stores determination
a. Bone marrow iron stain
b. Serum ferritin
c. Serum iron
d. Total Iron Binding Capacity (TIBC)
5. Definitive management may be achieved through splenectomy
a. Hereditary spherocytosis
b. G6PD deficiency
c. Thalassemia
d. Sickle cell hemoglobinopathy
6. History of pallor and tea colored urine after being given anti malarials and eating
legumes or beans best fits this disease:
a. Pyruvate kinase deficiency
b. GCPD deficiency
c. Spherocytosis
d. Fancom’s anemia
7. Most effective management of G6PD
a. Splenectomy
b. Gene therapy
c. Blood transfusion
d. Avoidance of drugs which may induce hemolysis
8. Disease almost always associated with positive direct Coomb’s test:
a. PNH
b. G6PD deficiency
c. Acute hemolytic reaction sec to ABO incompatibility
d. Idiopathic autoimmune hemolysis
9. Hallmark of the common forms of alpha thalassemia:
a. Elevated Hgb A
b. Elevated Hgb A2
c. Elevated HgbF
d. Low HgbA2
11. Hemoglobin levels in the elderly population
a. Almost all the elderly population have normal hemoglobin level
b. The female gender is more prone to nutritional type of anemia
c. About 25% of the elderly without underlying disease will have below normal
hemoglobin levels
d. About 30% of the male gender with anemia would be diagnosed with underlying
malignancy
12. Vegetarians are prone to anemia secondary to which of the following deficiency
a. Folic acid
b. Cyanocobalamin
c. Niacin
d. Zinc
13. Fundamental abnormality in pernicious anemia
a. Short bowel syndrome
b. Atrophic gastritis
c. Lack of Vitamin B12 in diet
d. malabsorption
14. Chronic renal diseases where erythropoietin production is spared
a. Chronic glomerulonephritis
b. Polycystic kidney disease
c. Chronic pyelonephritis
d. Diabetic nephropathy
15. Compatible with Anemia of Chronic disease
a. Microcytic hypochromic red cells, low serum iron, increased ferritin level
b. Microcytic hypochromic red cells, normal serum iron, low ferritin
c. Microcytic hypochromic red cells, normal serum iron and TIBC
d. Microcytic hypochromic red cells, low serum iron, high TIBC
16. Agent
17. Mechanisms of action of cyclosporine in aplastic anemia
a. Reduce cytotoxic T cells
b. Inhibits IL 2 production by T lyhmphocytes
c. Modulates adverse reactions to ATG
d. Stimulate production of erythropoietin
18. Specific treatment of marrow aplasia in severe aplastic anemia
a. Allogeneic stem cell transplantation
b. Autologous bone marrow transplantation
c. Cyclosporine
d. Blood transfusion
19. Hallmark clinical feature PNH
a. Ineffective eryhtropoiesis
b. Chronic intravascular hemolysis
c. Bone marrow failure
d. Bleeding diathesis
20. Clinical featured of idiopathic Thrombocytopenic Purpura in adults:
a. More common in men than women
b. Onset is acute
c. Spontaneous remission in 2%
d. A and C
21. Treatment for assymptomatic patients newly diagnosed with ITP with platelet
count of 50,000:
a. Oral prednisone 1 to 2 mg/kg/day
b. IVIg
c. IV hydrocortisone
d. Observation
22. Mild Von Willebrand’s disease type 2 patient for surgery is best managed with
a. DDAVP
b. Cryoprecipitate
c. Factor VIII/VWF concentrates
d. Factor VIIa
23. 19 year old male with 3% Factor 8 assay activity is compatible with
a. glanzmann’s thrombasthenia
b. Hemophilia A, mild
c. Hemophilia A, moderate
d. Hemophilia B, mild
24. What is the probability that a hemophilia A male married to a carrier will have a
carrier female offspring (Chapter 116)?
a. 25%
b. 50%
c. 75%
d. 100%
25. Treatments of choice for hemophilia A patients with inhibitors (Chapter 116)
a. Factor Vlla
b. Factor VIII, recombinant
c. Cryoprecipitate
d. Tranexamic acid
26. Central mechanism of disseminated intravascular coagulation (Chapter 116):
a. Massive platelet activation
b. Uncontrolled plasmin generation
c. Excessive thrombin generation
d. hyperfibrinolysis
27. Most sensitive test for disseminated intravascular coagulation
a. Platelet count
b. Fibrinogen level
c. D-dimer
d. Fibrinogen degradation products
28. Post partum hemorrhage is a major cause of maternal mortality with disseminated
intravascular coagulation. With consumption of fibrinogen as sequelae resulting to
bleeding, this may be replaced with (Chapter 113)
a. Whole Blood
b. Cryoprecipitate
c. Cryosupernate
d. Tranexamic acid
29. The following have been observed among patients who inherited thrombophilias
a. Often present with unusual sites of thrombosis
b. In majority of cases, thrombosis is provoked by surgery, pregnancy, immobilization OCP,
HRT or old age
c. Those heterozygous for factor V Leiden and prothrombin gene mutation have been
shown to have a higher rate of recurrent venous thrombosis verses the general
population
d. B and C
30. The following situation does not predispose to venous thrombosis (Chapter 117)
a. Increasing age
b. Varicose veins
c. Intake of melphalan in multiple myeloma
d. Trauma
31. trousseau’s syndrome may be noted in which of the following (Chapter 117):
a. Gastric Ca
b. Prolonged best rest
c. Trauma to blood vessels
d. Intake of Coumadin
32. Recommended anticoagulant prophylaxis for high risk activities like prolonged air travels
etc., for patients with thrombophilia (Chapter 118):
a. Aspirin
b. Warfarin
c. UFH
d. LMWH
34. Standard initial treatment for deep vein thrombosis (Chapter 118):
a. UFH
b. LMWH
c. Argatroban
d. Fondaparinux
35. Pharmacologic characteristics of warfarin (Chapter 110):
a. The onset of action is immediate
b. The antithrombotic effect depends on the reduction of the functional levels of the
clotting factors with longest half lives which are Factors II and X.
c. It is rapidly and almost completely absorbed from the kidney
d. Anticoagulation effect best for acute DVT
36. Treatment of choice for thrombotic thrombocytopenic Purpura (Chapter 175)
a. TpE
b. Cryosupernate
c. Steroids
d. Aspirin
37. Associated with ADAMTS 13 deficiency
a. Hemolytic uremic syndrome
b. Thrombotic thrombocytopenic purpura
c. Essential thrombocythemia
d. Heparin induced thrombocytopenia
39. The following is not a major indication for splenectomy in primary myelofibrosis
(Chapter 108)
a. Massively enlarged spleen
b. Excessive transfusion requirements
c. Portal hypertension
d. Severe thrombocytopenia
40. Molecular pathology of CML (Chapter 104)
41. Characteristics of CML compared with the other Chronic Myeloproliferative
disorder (P.vera, Idiopathic Myelofibrosis, Essential Thrombocythemia) (Chapter
108, 109)
a. Total WBC count > 30,000/ul in >90%
b. Platelet count > 750,000/ul in >50%
c. Increased red cell mass and plethora
d. JAK 2 mutation
42. The tyrosine kinase inhibitor Imatinib Mesylate (Chapter 109)
a. Initial therapy in almost all CML, chronic phase
b. Improved survival outcomes among CML patients in accelerated phase
c. Patients treated with imatinib after IFN failure had better survival than those who
continued receiving IFN
d. May be given as an alterative to autologous hematopoietic stem cell
transplantation
43. Best treatment option for accelerated and blast crisis CML (chapter 109)
a. Chemotherapy
b. Allogeneic stem cell transplant
c. Autologous stem cell transplant
d. Imatinib mesylate, higher dose
44. Most common site of relapse in ALL
a. CNS
b. Ovary
c. Bone marrow
d. Testes in males
45. Classic triad of multiple myeloma
a. Marrow plasmacytosis (>10%), lytic bone lesions, and serum and/or urine M
component
b. Anemia, lytic bone lesions and serum and. Urine M component
c. Marrow plasmacytosis (>10%), reverse albumin/globulin ratio, and serum and/or urine
M component
d. Anemia, marrow plasmacytosis (>20%), reverse albumin/globulin ratio
46. Major specific pathophysiologic mechanism in MDS (Chapter 107)
a. Defective maturation and death of marrow precursor cells
b. Fibrosis associated with PDGFR
c. Bone marrow suppression which may be associated with increased cytokine production
d. Bone marrow infiltration
47. Treatment of choice for elderly patients with MDS, RAEB (Chapter 107)
a. Lenolidamide
b. Intensive chemotherapy
c. Azacytidine
d. Arsenic trioxide
48. Management of platelet refractoriness in alloimmunized bleeding patient with severe
thrombocytopenia,with sepsis (Chap 113)
a. Random donor platelets at 1 unit/10 kg body weight, treat sepsis
b. Apheresed Platelets, steroids, treat sepsis
c. HLA matched Apheresed Platelets treat sepsis
d. Apheresed platelets and IV IG
A 1. A 26 year old female was stabbed with an ice pick several times over the right
anterior chest wall and was brought to the Emergency Room by her boyfriend. Vital
signs: BP=80 palpatory, HR=128/min, weak thready pulse RR=28/min. patient
appeared paper white with blood oozing over puncture wounds. For blood transfusion
requirements, whole blood (WB) was requested however only blood components
were available at the blood bank. What components can be combined in place of
whole blood?
a. Packed red blood cells (PRBC) + fresh frozen plasma (FFP) + platelets
b. PRBC + FFP
c. PRBC + Cryosupernate
d. PRBC + Cryosupernate + platelets
A 2. the above patient’s laboratory CBC hemoglobin-42 g/L, wbc-12.3, platelet count142,000; PT = 13.2 secs control 12.9, PTT = 40 secs control=36, Blood type was A Rh
positive. The patient is best managed with:
a. Crystalloids/colloids + A Rh positive PRBC
b. Crystalloids/colloids + A Rh positive PRBC and platelet count
c. Crystalloids/colloids + A Rh positive FWB
d. Crystalloids/colloids + O Rh positive FWB
A 3. A 38 year old male presents with hgb 76 g/L, MCV of 105fl, platelet count
80,000/ul, low level of serum folate ands a normal serum level of vitamin B12. this is
most compatible with:
a. Pregnancy
b. Alcoholism
c. Pernicious anemia
d. Resection of ileum
A 4. A 56 year old male, diabetic, was admitted with peripheral arterial occlusive
disease affecting his toes on both lower extremities. A few days after starting
Enoxaparine subcutaneously, his platelet count was noted to be 20 x 109/L. what
would be the best management for this patient?
a. Continue Enoxaparine and observe patient
b. Discontinue Enoxaparine and observe patient
c. Discontinue enoxaparine and start Unfrationated Heparin IV with monitoring of
INR
d. Discontinue Enoxaparine and start heparinoids
A 5.An Assymtomatic 72 y/o Multiple Myeloma patient with renal insufficiency with
hemoglobin at 94 g/dl, platelet count =102,000 is best managed with
a. PRBC transfusion
b. Leukocyte depleted PRBC transfusion
c. Whole Blood
d. Erythropoietin
A 6. An assymptomatic 22 year old woman was seen at the OPD requesting clearance
for work, history revealed regular monthly cycles, wherein she utilizes 12-14 maternity
napkins, most fully soaked. Physical examination (except forpallor)was unremarkable.
Labs: CBC-Hgb=72 g/L, hct ==0.22, wbc=7.5 (seg 68%, lympho=30%), plt ct = 450,
MCV=65, mch=18, RDW=21Serum ferritin =6.7
What is best management for the anemia?
a. Oral iron supplement
b. Intravenous iron
c. Blood transfusion
d. Improve diet
A 7. A 36 year old male, asymptomatic consulted because of incidental finding of
platelet count at 850,000 on CBC (Hgb=138, wbc=18.5)during annual medical. Pas
medical history, other labs and PE unremarkable. If the patient was further worked up
and showed still elevated plt count at 600,000 mild leukocytosis, positive JAK 2
mutation assay and negative bcr-abl, what would be best management for the
patient?
a. Observation and monitoring
b. Start anagrelide and hydroxyurea
c. Start imatinib
d. allogeneic bone marrow transplant
A 8. A 62/M, previous total gastrectomy for NHL 7 years ago, came in because of easy
fatigability of 4 months duration. CBC: Hb 5.2 HCT 25 MCV 125 WBC 3.5 Seg 54 Ly 46
Platelets 78; Retic count 1.2% PBS: Macricytosis, aisopoikilocytosis
WHAT CLINICAL SITUATION COULD EXPLAIN THE FINDINGS?
A. Iron deficiency anemia
B. Vitamin B12 deficiency
C. Sideroblastic anemia
D. Myelopthisic anemia
2. In a patient with metabolic alkalosis from vomiting, this urine parameter serves as an
accurate indicator of volume status:
a. Sodium
b. Chloride
c. Potassium
d. Specific gravity
6. This chinese herb present in slimming tea preparations is also believed to be the cause of
Balkan nephropathy
a. Aristolochic acid
b. Ochratoxin A
c. Melamine
d. cadmium
8. Components involved in providing gastroduodenal defense against peptic ulcers
• T A. Unstirred layer of mucus and bicarbonate
• T B. Surface epithelial cells that generate heart shock proteins
• T C. Cell renewal from mucosal progenitor cells
• T D. Submucosal vascular system that generates alkaline tide
• T E.Prostaglandins that promote epithelial cell restitution
42. Primary prophylaxis of esophageal variceal hemorrhage
• T A. Propanolol
• T B. Nadolol
• T C. Variceal band ligation
• F D.Transjugular intrahepatic portosystemic shunt (TIPS)
• F E. Surgical esophageal transection
43. Radiographic findings present in acute intestinal obstruction
• T A. Stepladder pattern in small intestine
• T B. Absence of colonic gas
• T C. Coffee bean-shaped mass
• F D. Free air in the peritoneum
• F E. “Thumbprinting”
44. Diagnosis of diverticulitisis best made on CT with these findings
• T A. Sigmoid diverticula
• T B. Thickened thickened colonic wall >4mm
• T C. Inflammation within pericolic fat
• F D. String of beads sign
• F E. Bird’s beak sign
45. Treatment for stage II hemorrhoids, which are protrusions with spontaneous
reduction
• T A. Fiber supplementation
• T B. Cortisone suppository
• F C. Sclerotherapy
• F D. Banding
• F E. Operative hemorrhoidectomy
46. Colorectal cancer screening strategies for asymptomatic individuals ≥ 50 years of
age
• T A. Colonscopy every 10 years
• T B. Fecal DNA testing every 3 years
• T C. CT colonography every 5 years
• T D. Flexible sigmoidoscopy every 5 years
• T E. Double-contrast barium enema every 5 years
47. Epidemiologic characteristics of ulcerative colitis
• T A. Smoking is preventive
• F B. Oral contraceptive pills increase risk
• T C. Appendectomy is protective
• T D. More common in African Americans than asians
• T E. Has two peak ages of onset
48. Pathologic feature of Crohn’s disease
• T A. Can affect the mouth and anus
• T B. Cobblestone appearance
• T C. Perirectal fistulas and anal stenosis
• F D. Almost always involves the rectum
• F E. Disease process is limited to the submucosa
49. Pathologic features of ulcerative colitis
• F A. “Skip areas” or areas of histologically normal mucosa are common
• T B. Pseudopolyps may be present
• F C. Disease process is transmural
• F D. “Creeping fat” or thickened mesentry encasing bowel is a common feature
• F E. Granulomas are a pathognomonic feature
50. Extraintestinal manifestations of inflammatory bowel disease more commonly seen in
Crohn’s disease than in ulcerative colitis
• F A. Pyoderma gangrenosum
• T B. Ankylosing sspondylatis
• F C. Primary sclerosing cholangitis
• T D. Nephrolithiasis
• T E. Reactive amyloidosis
51. Indications for surgery of ulcerative colitis
• T A. Toxic megacolon
• F B. Refractory fistula
• T C. Colon cancer prophylaxis
• T D. Colon dysplasia
• F E. Perianal disease unresponsive to medical therapy
1.
•
•
•
•

Most important indication for urgent gastrointestinal endoscopy in the list below
A. Age>60 Years
B. Decrease in hemoglobin
C.Decrease in hematocrit
D. Orthostatic hypotension
2. What is the most sensitive and specific approach to testing for peptic ulcer disease?
• A. Double-contrast barium study
• B. Urea breath test
• Serum gastrin level
• D. Endoscopy
3. Which type of chronic atrophic gastritis is associated with pernicious anemia?
• A. Isolated granulomatous gastritis
• B. Antral predominant or type B gastritis
• C. Multifocal atrophic gastritis
• D. Autoimmune or type A gastritis
4. Risk of rebleed is lowest with this type of peptic ulcer
• A. ulcer with spurting
• B. ulcer with visible vessel
• C. ulcer with pigmented spot
• D. clean-based ulcer
5. Preferred endoscopic therapy for control of active esophageal variceal bleeding
• A. Band ligation
• B. Sclerotherapy
• C. Balloon tamponade
• D. Transjugular intrapatic portosystemic shunt (TIPS)
6. Large-caliber arteriole that runs immediately beneath the gastrointestinal mucosa and
bleeds through pinpoint mucosal erosion
• A. Gastroesophageal varix type I (GOV1)
• B. Mallory-Weiss tear
• C. Gastric antral vascular ectasis (GAVE)
• D. Dieulafoy’s lesion
7. Sight, smell and taste of food are components of this phase of stimulated gastric and
secretion
• A. cephalic
• B. Gastric
• C. Pancreatic
• D. Intestinal
8. This cell is responsible for gastric acid secretion
• A. Chief cell
• B. G cell
• C. Parietal cell
• D. ECL cell
9. Ulcers within 3cm of the pylorus, commonly accompanied by high gastric acid production
and duodenal ulcers
• A. Type I gastric ulcers
• B. Type II gastric ulcers
• C. Type III gastric ulcers
• D. Type IV gastric ulcers
10. This type of gastritis produced by chronic H. pylori infection may lead to development of
MALT lymphoma
• A. Antral-predominant gastritis
• B. Nonatrophic pangastritis
• C. corpus-predominant atrophic gastritis
• D. Fundal gastritis
11. Dose of aspirin deemed safe, as it does not cause peptic ulceration
• A. 75mg/day
• B. 81mg/day
• C. 325mg/day
• D. no dose is completely safe
12. Most commonly encountered diagnosis among patient’s seen for upper abdominal
discomfort
• A. Nonulcer dyspepsia (NUD)
• B. Gastroesophaegeal reflux disease (GERD)
• C. peptic ulcer disease (PUD)
• D. Nonerosive reflux disease (NERD)
13. Cyto preotective agent used in peptic ulcer disease that may lead to aluminum-induced
neurotoxicity in patients with chronic renal insufficiency
• A. Bismuth subsalicylate
• B. Misoprostol
• C. Rebamipide
• Sucralfate
14. Cytoprotective agent used in peptic ulcer disease contraindicated in women who may
be pregnant
• A. Sucralfate
• B. Colloidal bismuth subcitrate
• C. Misoprostol
• D. Amitriptyline
15. Proven benefit of H. pylori eradication in patients with peptic ulcer disease
• A. Decreases in ulcer recurrence
• B. Elimination of risk of ulcer perforation
• C. Prevention of gastric cancer
• D. Prevention of gastric lymphoma
16. Most feared complication with amoxicillin use in triple therapy of peptic ulcer disease
• A. Black tongue
• B. Pseudomembranous colitis
• C. Allergic reaction
• D. Hepatotoxicity
17. Most common cause of treatment failure in compliant patients on triple therapy
for peptic ulcer disease
• A. underdosing
• B. Antibiotic-resistant strains
• Drug side effects leading to discontinuation
• Lower bioavailability of generic drugs
18. Test of choice for documenting H. pylori eradication
• A. Stool antigen test
• B. Urea breath test
• C. Serologic testing
• D. Histopathologic exam
19.Definition of a refractory peptic ulcer
• A. Gastric ulcer that fails to heal after 12 weeks
• B. gastric ulcer that fails to heal after 6 weeks
• C. Duodenal ulcer that fails to heal after 4 weeks
• D. Duodenal ulcer that fails to heal after 2 weeks
20. Surgical treatment of choice for a gastric antral ulcer
• A. Antrectomy, intraoperative ulcer biopsy, vagotomy
• B. Subtotal gastrectomy with Roux-en-Y
• C. Antrectomy with a Billroth I anastomosis
• D. Ulcer excision with vagotomy and drainage
21. Cornerstone of therapy for patients with dumping syndrome after surgery for peptic
ulcer disease
• A. Antidiarrheal medications
• B. Octreotide
• C. Anticholinergic agents
• Dietary modification
22. Physical exam finding which indicates the presence of a severe necrotizing pancreatitis
a. Abdominal distenstion
b. Jaundice
c. Pleural effusion
d. Blue discoloration around umbilicus
23. Lab value consistent with acute pancreatitis
a. Threefold elevated serum lipase
b. Fourfold elevated salivary-type amytase
c. Hypoglycemia
d. hypercalcemia
24. Type of viral hepatitis for which no vaccine exists
a. HAV
b. HCV
c. HDV
d. HEV
25. Endovascular stenting is a management option for this type of acute intestinal
ischemia
• A. Aterial embolus
• B. Arterial thrombosis
• C. Venous thrombosis
• D. Non-occlusive mesenteric ischemia
26. This is the gold standard for the diagnosis and management of acute intestinal arterial
occlusive ischemia
• A. mesenteric duplex scan
• B. CT angiography
• C. Lapatoramy
• D. Ablation of renin-angiotensin axis

27. This is the most common extrauterine condition requiring abdominal operation during
pregnancy
• A. neprolithiasis
• B. Small-bowel obstruction
• C. perinephric abscesses
• D. Appendicitis
28. Most common cause of hematocheza in patients >60 years
• A. inflammatory bowel disease
• B. Colon Cancer
• C. Colonic diverticulitis
• D. hemorrhoids
29. Which laboratory test is a highly sensitive and specific marker for detecting intestinal inflammation
in inflammatory bowel disease?
• A. Fecal lactoferin
• B. C-reactive protein
• C. Erythrocyte sedimentation rate
• D. Platelet count
• E. Fecalysis
30. In which variant of inflammatory bowel disease does total parenteral nutrition play a major role in
inducing remission?
• A. Distal ulcerative colitis
• B. Extensive ulcerative colitis
• C. Inflammatory Crohn’s disease
• D. Crohn’s disease with TB ileitis
• E. Fistulizing Crohn’s disease
1.
•
•
•
•

A 40-year-old male with melena is found to have a gastric ulcer with a visible vessel on endoscopy.
Which treatment strategy is recommended?
A. Discharge home with oral PPI therapy
B. ward admission, no IV PPI therapy, no endoscopic therapy
C. ICU admission, IV PPI therapy, endoscopic therapy
D. Ward admission, IV PPI therapy, endoscopic therapy
2. A 22-year-oldman had hepatitis B serologies done as part of his seafarer’s preemployment workup. Which serologic pattern assures that his childhood vaccination series
remain effective?
a. HBsAg(+), anti-HBc(+), anti-HBs(-)
b. HBsAg(+), anti-HBc(+), anti-HBs(+)
c. HBsAG(-), anti-HBc(-), anti-HBs(-)
d. HBsAG(-), anti-HBc(-),anti-HBs(+)
3. Your 50-year old uncle is in the hospital for hepatitis. His serologies are (+)HBsAG, (+)IgM
anti-HAV, (+)IgM anti-HBc, (-) anti-HCV. What is your interpretation?
a. Acute hepatitis B
b. Acute hepatitis A and B
c. Acute hepatitis A superimposed on chronic hepatitis B
d. Acute hepatitis C
4. A 48-year-old man with HBeAg-negative chronic hepatitis B asks to be treated. His viral
load is 1,500 IU/mL and his ALT is normal. This would be a reasonable recommendation:
• A. Treatment with pegylated interferon
• B. treatment with entecavir
• C. referral for liver transplantation
• D. No treatment
5. A 40-year-old man was admitted from the ER with the following test
results, prothrombin time 18 (N=12), serum bilirubin 12mg?dL. Which is the best
treatment for his alcoholic hepatitis?
• A. High dose thiamine
• B. Silymarin
• C. IV multivitamins
• D. Prednisolone
6. A 50-year-old first degree relative of patient diagnosed with colon cancer at age 65
should undergo colonoscopy this often.
• A. every 10 years
• B. every 5 years
• C. every 3 years
• D. every year
6. Deficiencies of the Adaptive Immune System:
T a. Severe combined immune deficiency (SCID)
T b. Hyper-IgE syndrome (autosomal dominant)
T c. Hyper IgM syndrome
T d. Common variable immunodeficiency (CVID)
F e. Chronic granulomatous disease (CGD)
7. Pathogenesis of Drug Reactions
T a. Untoward cutaneous responses can arise as a result of immunologic mechanisms.
T b. Immunologic reactions require toward activation of host immunologic pathways
and are designated as drug allergy.
T c. Drug reactions occurring through nonimmunologic mechanisms may be due to
activation of effector pathways, ovedosage, cumulative toxicity, side effects, drug
interactions, metabolic alterations, exacerbation of preexisting dermatologic
conditions or inherited protein or enzyme deficiencies.
T d. Increase in molecular size is associated with increased immunogenicity.
F e. Anaphylaxis is more common among oral medications than IV medications.
15. Extent of epidermal detachment in Stevens-Johnson Dyndrome:
a. <5% body surface area
b. <10% body surface area
c. <15% body surface area
d. <20% body surface area

1. A 60 year old male with a 50 pack year smoking history consults for chronic cough,
hemophylis, weight loss and exertional dysnea. You consider lung cancer. Which of
the following would be consistent with his case?
T a. A chest x-ray would likely reveal a lung lesion that is central in location.
T b. The most likely histologic subtype is going to be squamouss cell carcinoma.
F c. The histologic subtype plays in the most important prognostic factor in non-small
cell lung cancer.
F d. This patient will most likely benefit from anti-angiogenic agents added to systemic
therapy.
T e. The main etiology factor associated with every histologic subtype of lung cancer is
cigarette smoking.
2. Clinical manifestations of lung cancer.
T a. Hemoptysis is a manifestations of endobronchial tumor growth.
F b. Symptoms of cough, hemoptysis, and stridor are examples of symptoms caused by
peripherally growing lesions.
T c. Shoulder pain, destruction of the 1st snd 2nd ribs, and involvement odf the 8th cervical
and 1st and 2nd thoracic nerves are features of a Pancoast’s syndromes.
F d. A Cushing-like sundrome is an uncommon paraneoplastic syndrome associated with
adenocarcinomas
T e. Non-small cell lung cancers have a predilection for causing metastatis but these don’t
cause adrenal insufficiency.
3. A 66 year old female with adenocarcinoma of the lung is found to have a 6 x 5 cm tumor
a the lower lobe of the right lung on the CT scan. Mediastinal adenopathy is detected as
well as an enlarged right supraclavicular node. Liver and adrenals are free of tumor. Bone
scan reveals multiple enhancing lesions at the thoracic and lumbar vertebrae.
T a. The tumor may be labelled as T2
F b. Mediastenal adenopathy makes this an N1 lesion.
T c. The presence of palpable supraclavicular nodes makes this a N3 staged disease.
F d. Because the liver and adrenals are free of tumor this can be labelled as MO.
F e. A 5-year survival rate of > 40% is expected with systemic therapy even in the presence
of abnormal bone scan.
4. Critical dates in a woman’s life that have a major impact on the incidence of breast
cancer:
T a. Age of menarche
F b. Age of marriage
T c. Age of first full-term pregnancy
F d. Age of first lactation
T e. Age of menopause
5. In the evaluation of breast masses in women, which of the following is applicable?
T a. In the absence of certain risk factors cannot be used to avoid the need for biopsy in
women with palpable breast masses.
F b. In a woman with palpable breast mass, MRI features are diagnostic and may avoid the
need for biopsy.
T c. Diagnostic mammography defines the procedure done after the discovery of a palpable
breast mass in order to evaluate the rest of the breast prior to a biopsy.
F d. In pregnant women, the occurrence of a dominant breast mass is most likely due to
hormonal changes, thus avoiding the need for a biopsy.
T e. Women whose biopsies reveal benign lesions without atypia have little risk of
developing cancer.
6. Prognostic factors in breast cancer.
T a. Tumor stage
T b. ER and PR status
T c. Her 2 status
T d. S phase function
T e. Histologic grade
7. Risk factors in colorectal cancers.
T a. Diet high in animal fat
F b. Low fiber diet
T c. Inflammatory bowel disease
F d. Female sex
T e. Lynch syndrome
8. Recommended screening procedures for colorectal cancer.
F a. Digital rectal exam
T b. Feccal occult blood test
T c. Flexible sigmoidoscopy
F d. Abdominal CT scan
F e. Whole body PET scan
9. Predictors of prognosis in patients with colorectal cancer.
F a. Age
T b. Grade of the tumor
T c. Depth of tumor invasion
F d. EGFR mutation status
T e. Lymph node involvement
10. Clinical features of Gastric cancer.
T a. Elderly patients have a history of chronic ulceration with the histopathology most
likely an intestinal type of adenocarcinoma.
F b. H. pylori increases the risk for gastric cancer by causing mutations in a critical set
of tumour suppressor genes.
F c. Dysphagia and early satiety are symptoms due to intestinal types of lesions that
may involve the entire stomach.
T d. Intestinal type lesions occur most frequently in the antrum and lesser curvative of
the stomach.
F e. The lung is the most common site of hematogenous spread.
11. Etiologic factors believed to be associated with esophageal cancer.
T a. Chronic alcohol intake
T b. Radiation induced strictures
F c. Vitamin C deficiency
F d. Chronic intake of cold beverages
T e. Fungal toxins in pickled vegetables
12. Risk factors associated with Hepatocellular cancer.
T a. Poor hepatitic cirrhosis
T b. Ingestion of poorly stored grains
F c. Cigarette smoking
F d. History of Hepatitis A
F e. Juvenile diabetes mellitus
13. A 60 year oldmale alcoholic with a 2 week history of jaundice is noted to have a hard
nodular mass on liver palpation. Serum AFP is 500 IU/L. with regards to his management
, which of the following isapplicable?
T a. The best procedure for evaluating tumor vascularity and size is a triphasis CT scan of
the abdomen.
T b. Pain is the most common presenting symptom in high risk areas of liver cancer.
F c. an alpha feto protein level of > 1000 IU/L is diagnostic for HCC.
F d. Radiofrequency ablation or ethanol injection can be very useful for tumor near the
main portal areas.
F e. The multikinase inhibitor Erlotinib has been found to improve median survival by 3
months in patients with advanced diseas.
14. The clinical presentation of pancreatic cancer.
F a. Courvorsier’s sign involves a dilated portal vein in a patient with carcinoma of the
pancreatic head
T b. Pain is the most frequent symptom in patient’s with tumors of the pancreatic body
F c. Elevated CA 19-9 levels are diagnostic of pancreatic cancer but carry no prognostic
implications.
F d. A detailed clinical staging evaluation provides important therapeutic and prognostic
information in patients with advanced disease.
F e. 5-fluorouracil + Folinic acid remain the initial combination of choice in patients with
metastatic disease.
15. Clinical characteristics of renal cell carcinoma.
T a. the proximal tubules are the most common sites of origin of clear cell carcinomas.
F b. the strongest risk factor associated with tumor development remains a positive family
history.
F c. the classic triad of hematuria, flank pain, and flank mass can be seen in more than half
of patients.
F d. clear cell carcinoma is associated with the inhibition of the epidermal growth factor
receptor gene resulting in tumor angiogenesis.
T e. agents that inhibit angiogenesis may be useful in the management of metastatic
disease.
16. Clinical characteristics of transitional cell cancers of the genito-urinary tract.
T a. Polychromatropism, the ability to recur in new sites along the urothelial tract, is a
common clinical manifestation
T b. Smoking increases risk for developing urothelial carcinomas.
T c. Once node positive disease is detected, the patientis labelled as having stage IV disease.
F d. The bladder should be suspected as the most common site of microscopic hematuria.
F e. In stage IV disease, cure may still be possible in the presence of visceral metastase.
17. Clinical manifestations of prostate cancer.
F a. PSA screening is the main reason behind the marked decline in mortality rates.
F b. Bicalumatide has been shown to improve survival in the chemo-prevention of
prostate cancer.
T c. PSA screening has resulted in treatment of many low-grade tumors that would
otherwise not have been clinically significant.
F d. Elevated PSA is diagnostic for the presence of prostatic carcinoma.
T e. the main function of trans-rectal ultrasound is directing prostate biopsies rather
than staging.
18. Risk factors for head and neck cancers.
T a. Cigarette smoking
F b. Age > 60 years
T c. Low consumption of fruits and vegetables
T d. Human papilloma virus
F e. Eat Asian race
19. Management of head and neck cancers.
T a. Localized (T1 and T2) squamous cell cancer can be treated with surgery of RT with
curative intent
T b. Despite toxicity, concomitant chemo-RT has a better survival rate compared to
sequential therapy in advanced disease.
T c. Concomitant chemo-RT produces better survival rates and allows for organ preservation
in advanced head and neck cancers compared to RT alone.
F d. Concomitant chemo RT can produce similar benefits in patients with metastatic disease
compared to chemotherapy alone squamous cell cancers.
F e. Cis-retinoic acid can result in regression of leukoplakia and prevents occurrence of
second primary sites.
20. Clinical features of ovarian cancer.
T a. the use of oral contraceptives has been associated with a decreased risk of epithelial
ovarian cancers.
T b. pelvic pain is a common symptom of ovarian cancer and may help identify early stage
disease.
F c. screening with annual CA-125 determination is expected to improve survival and to
identify early stage disease.
T d. the most common type of epithelial ovarian tumors is the serous type.
T e. prognostic factors for ovarian cancer include tumor stage, tumor grade and extent of
residual disease after surgery
21. Clinical features of cervical cancer.
T a. the most important etiologic agent is the venereal transmission of the human
papilloma virus
T b. PAP smear is capable of detecting up to 95% of early cervical lesions
F c. the presence of a mass in the cervix is the most common sign of cervical cancer
F d. the presence of hydronephrosis immediately classifies a patient as having stage IV
disease
T e. Tumors that extend beyond the area of the true pelvis are labelled as stage IV disease
22. Paraneoplastic metabolic syndromes and their associated malignancies.
F a. Hypercalcemia - Osteosarcomas
T b. Cushing’s syndrome – Carcinoid tumors
T c. SIADH – Squamous cell CA, lung
F d. Hypoglycemia –Adrenal medulla tumors
T e. Diarrhea – Pancreatic cancer
23. Paraneoplastic hemotologic syndromes and their associated malignancies.
T a. Erythrocytosis – Renal cell cancer
T b. Thrombocytosis – Lung cancer
T c. Granulocytosis – Hodgkin’s lymphoma
F d. Eosinophilia – Neuroendocrine tumors
T e. Thrombophlebitis – Breast cancer
24. Clinical features of malignant melanoma.
T a. Adults have a higher risk for development compared to children.
F b. well-defined boarders are more characteristic of melanoma than benign nevi.
F c. The most important prognostic factor for melanoma is tumor size.
T d. of the characteristics that can differentiate melanoma from other pigmented
lesions, the weakest is diameter of the lesion
F e. uniformly dark pigmentation is associated more than benign nevi.
2. What procedure would you advise to a 50 year old female who would want to
undergo screening for breast cancer?
a. Breast MRI
b. Mammography
c. Self-breast examination
d. Self-breast + annual clinical breast exam
3. A 65 year old woman with breast cancer post-MRM was discovered on pathologic
analysis to have a 4 x 3 cm tumor in her left breast. 7 of the 14 isolated axillary nodes
are positive for malignancy. Liver US, chest x-ray and bone scan are all normal. What
would her stage be?
a. I
b. II
c. III
d. IV
4. In which of the following conditions would a biologic immune therapy be indicated
as part of the adjuvant treatment of early stage breast cancer?
a. Estrogen receptor positive
b. Progesterone receptor positive
c. Her2/neu receptor positive
d. Triple negative
5. Which of the following signs or symptoms is the result of a peripherally located tumor in
lung cancer?
a. Clubbing
b. Pleuritis chest pain
c. Hemoptysis
d. Facial edema
6. Which situation would argue strongly for proceeding with the resection of a
asymptomatic pulmonary nodule?
a. Male sex
b. Presence of calcification on chest x-ray
c. Stable findings on serial chest x-ray
d. Presence of atelectasis
7. In which of the following situations is curative surgery or radiotherapy no longer possible
in a patient with non-small cell lung cancer?
a. A 5 cm tumor 4 cm from the carina
b. N1 nodal involvement
c. Ipsilateral mediastinal node involvement
d. Paralysis of the vocal cords
8. Which of the following is an acceptable strategy for chemotherapy in advanced non-small
cell lung cancer?
a. Patients with good performance status should be treated with single agents
b. Non-platinum doublets are the new standard of care in these situations
c. Cisplatin or carboplatin remains a standard component of doublet chemotherapy
d. The concomittant administration of tyrosine kinase inhibitors with standard
chemotherapy is superior to either used alone
9. Which of the following is a recognized risk factor of carcinogen for salivary gland cancer?
a. Cigarette smoking
b. Alcohol intake
c. Human papilloma virus
d. None identified
10. Which of the following is recognized as an acute toxicity of RT in head and neck
cancers?
a. Mucositis
b. Immobilization of the tongue
c. Loss of taste
d. xerostomia
11. Which of the following subtypes is the most aggressive in melanoma?
a. Desmoplastic
b. Acral lentiginous
c. Nodular
d. Lentigo maligna
12. Which of the following is considered a standard form of therapy in metastatic
melanoma?
a. Dacarbazine
b. Interferon
c. Cisplatinum
d. Interleukein-2
13. Which of the following is consistent with a more aggressive type of basal cell cancer?
a. Superficial subtype
b. Small lesions
c. Located on the scalp
d. Pigmented lesions
14. What is an expected response of tumor blood vessels to treatment with bevacizumab, a
potenta anti-VEGF monoclonal antibody?
a. Inhibition of dendritic cell action
b. Inhibition of vascular permeability
c. Growth of dilated and leaky vessels
d. Sprouting of endothelial cells from host vessels
15. Which of the following is important in enhancing the efficacy of radiation against cancer
cells?
a. Cold
b. Heat
c. Oxygen
d. Iron
16. In which of the following tumors is cure still possible with chemotherapy even in an
advanced stage of the disease?
a. Endometrial cancer
b. Testicular cancer
c. Breast cancer
d. Follicular lymphomas
17. Which of the following is an example of a biologic form of therapy?
a. Aromastase inhibitors
b. Interferon
c. Anti-tumor antibiotics
d. gefitinib
18. In which of the following conditions would routine G-CSF prophylaxis against febrile
neutropenia be required during the first cylce of chemotherapy?
a. Conventional dose regimen
b. Performance status of 3
c. Age of 50 years
d. Adjuvant chemotherapy for breast cancer
19. Hyponatremia as a paraneoplastic syndrome may be seen in which of the following?
a. Renal cell cancer
b. Rhabdomyosacoma
c. Small cell lung cancer
d. Non-Hodgkin’s lymphoma
20. Eosinophilia is a paraneoplastic syndrome usually associated with which of the
following?
a. Gastric cancer
b. Breast cancer
c. Lymphomas
d. Paancreatic cancer
21. Which of the following sarcomas arises from the interstitial cells of Cajal and are
dependent on the c-kit oncogene for malignant transformation?
a. Leiomyorsarcome
b. Primitive neuroectodermal tumor
c. Gastrointestinal stromal tumor
d. Ewing’s sarcoma
22. Which of the following is a known characteristic of metastatic bone lesions?
a. Osteolytic lesions have a higher incidence of hypercalcemia
b. Osteolytic lesions produce higher levels of serum alkaline phosphatase
c. Osteoblastic lesions are best diagnosed by plain radiographs
d. Osteoblastic lesions are associated with the elaboration of parathyroid hormone related
peptides
23. An 80 year old female is diagnosed to have peritoneal carcinomatosis from an epithelial
ovarian cancer. Which of the following is the best treatment option?
a. Paclitaxel-platinum combination
b. Intraperitoneal chemotherapy
c. Pelvic exanteration followed by chemo-RT
d. Targeted agents
24. Which of the following is a known risk factor for endometrial cancer?
a. Multiple partners
b. Multiparity
c. Obesity
d. Late menarche
25. Which of the following is the molecular target of the E& protein of HPV 16 which may
results in the development of cervical cancer?
a. P53
b. Rb
c. Bcl-2
d. K-ras
26. According to the American Cancer Society guidelines, when should PAP smear
screening begin?
a. Menarche
b. After the 1st pregnancy
c. After the 1st live birth
d. Start of sexual activity
27. In which situation should SVC syndrome be considered a true medical emergency?
a. Facial edema
b. Jugular vein engorgement
c. Dyspnea
d. dysphagia
28. What is the most common location of esophageal adenocarcinomas?
a. Oropharyngeal portion
b. Upper third
c. Middle third
d. Lower third
29. A 75 year old male, chronic smoker, is diagnosed to have an unresectable esophageal
cancer and complains of severe dysphagia and anorexia. What is the most promising
management approach?
a. Radiotherapy
b. Chemotherapy + RT
c. Endoscopic laser therapy
d. Repeated endoscopic dilatation
30. Which of the following is consistent with the intestinal type of gastric
adenocarcinomas?
a. Found mostly in the cardia
b. Presents as linitis plastica
c. Ulcerative in presentation
d. Usually seen in younger patients
31. Which of the following represents a pre-malignant lesion of colorectal cancer?
a. Hamartoma
b. Hyperplastic polyp
c. Adenomatous polyp
d. Juvenile polyp
ICU ventilator patient management options
ICU ventilator patient management options
ICU ventilator patient management options
ICU ventilator patient management options
ICU ventilator patient management options
ICU ventilator patient management options
ICU ventilator patient management options
ICU ventilator patient management options
ICU ventilator patient management options
ICU ventilator patient management options

More Related Content

What's hot

03.sabiston surgery questions 17th ed
03.sabiston surgery questions 17th ed03.sabiston surgery questions 17th ed
03.sabiston surgery questions 17th edLucia Rosales
 
Krok 1 - 2015 (Physiology)
Krok 1 - 2015 (Physiology)Krok 1 - 2015 (Physiology)
Krok 1 - 2015 (Physiology)Eneutron
 
Krok 2 - 2014 (Therapy)
Krok 2 - 2014 (Therapy)Krok 2 - 2014 (Therapy)
Krok 2 - 2014 (Therapy)Eneutron
 
Krok 1 - 2015 (Anatomy)
Krok 1 - 2015 (Anatomy)Krok 1 - 2015 (Anatomy)
Krok 1 - 2015 (Anatomy)Eneutron
 
Krok 2 - 2015 Question Paper (General Medicine)
Krok 2 - 2015 Question Paper (General Medicine)Krok 2 - 2015 Question Paper (General Medicine)
Krok 2 - 2015 Question Paper (General Medicine)Eneutron
 
Krok 1 - 2014 (Path-Physiology)
Krok 1 - 2014 (Path-Physiology)Krok 1 - 2014 (Path-Physiology)
Krok 1 - 2014 (Path-Physiology)Eneutron
 
Krok 2 - 2013 (Pediatrics)
Krok 2 - 2013 (Pediatrics)Krok 2 - 2013 (Pediatrics)
Krok 2 - 2013 (Pediatrics)Eneutron
 
Krok 2 - 2013 Question Paper (General Medicine)
Krok 2 - 2013 Question Paper (General Medicine)Krok 2 - 2013 Question Paper (General Medicine)
Krok 2 - 2013 Question Paper (General Medicine)Eneutron
 
Krok 2 Medicine - 2016 Question Paper
Krok 2 Medicine - 2016 Question PaperKrok 2 Medicine - 2016 Question Paper
Krok 2 Medicine - 2016 Question PaperEneutron
 
Krok 2 Medicine - 2018 Question Paper
Krok 2 Medicine - 2018 Question PaperKrok 2 Medicine - 2018 Question Paper
Krok 2 Medicine - 2018 Question PaperEneutron
 
Krok 2 Medicine - 2017 Question Paper
Krok 2 Medicine - 2017 Question PaperKrok 2 Medicine - 2017 Question Paper
Krok 2 Medicine - 2017 Question PaperEneutron
 
Krok 2 - 2007 Question Paper (General Medicine)
Krok 2 - 2007 Question Paper (General Medicine)Krok 2 - 2007 Question Paper (General Medicine)
Krok 2 - 2007 Question Paper (General Medicine)Eneutron
 
Krok 1 - 2010 Question Paper (General medicine)
Krok 1 - 2010 Question Paper (General medicine)Krok 1 - 2010 Question Paper (General medicine)
Krok 1 - 2010 Question Paper (General medicine)Eneutron
 
Krok 1 - 2006 Question Paper (General medicine)
Krok 1 - 2006 Question Paper (General medicine)Krok 1 - 2006 Question Paper (General medicine)
Krok 1 - 2006 Question Paper (General medicine)Eneutron
 
Krok 1 - 2014 (Physiology)
Krok 1 - 2014 (Physiology)Krok 1 - 2014 (Physiology)
Krok 1 - 2014 (Physiology)Eneutron
 
Krok 1 - 2014 (Biochemistry)
Krok 1 - 2014 (Biochemistry)Krok 1 - 2014 (Biochemistry)
Krok 1 - 2014 (Biochemistry)Eneutron
 
Krok 2 - 2012 Question Paper (General Medicine)
Krok 2 - 2012 Question Paper (General Medicine)Krok 2 - 2012 Question Paper (General Medicine)
Krok 2 - 2012 Question Paper (General Medicine)Eneutron
 
Krok 1 - 2015 (Pharmacology)
Krok 1 - 2015 (Pharmacology)Krok 1 - 2015 (Pharmacology)
Krok 1 - 2015 (Pharmacology)Eneutron
 
Krok 2 - 2014 (Surgery)
Krok 2 - 2014 (Surgery)Krok 2 - 2014 (Surgery)
Krok 2 - 2014 (Surgery)Eneutron
 
Krok 1 - 2015 (Microbiology)
Krok 1 - 2015 (Microbiology)Krok 1 - 2015 (Microbiology)
Krok 1 - 2015 (Microbiology)Eneutron
 

What's hot (20)

03.sabiston surgery questions 17th ed
03.sabiston surgery questions 17th ed03.sabiston surgery questions 17th ed
03.sabiston surgery questions 17th ed
 
Krok 1 - 2015 (Physiology)
Krok 1 - 2015 (Physiology)Krok 1 - 2015 (Physiology)
Krok 1 - 2015 (Physiology)
 
Krok 2 - 2014 (Therapy)
Krok 2 - 2014 (Therapy)Krok 2 - 2014 (Therapy)
Krok 2 - 2014 (Therapy)
 
Krok 1 - 2015 (Anatomy)
Krok 1 - 2015 (Anatomy)Krok 1 - 2015 (Anatomy)
Krok 1 - 2015 (Anatomy)
 
Krok 2 - 2015 Question Paper (General Medicine)
Krok 2 - 2015 Question Paper (General Medicine)Krok 2 - 2015 Question Paper (General Medicine)
Krok 2 - 2015 Question Paper (General Medicine)
 
Krok 1 - 2014 (Path-Physiology)
Krok 1 - 2014 (Path-Physiology)Krok 1 - 2014 (Path-Physiology)
Krok 1 - 2014 (Path-Physiology)
 
Krok 2 - 2013 (Pediatrics)
Krok 2 - 2013 (Pediatrics)Krok 2 - 2013 (Pediatrics)
Krok 2 - 2013 (Pediatrics)
 
Krok 2 - 2013 Question Paper (General Medicine)
Krok 2 - 2013 Question Paper (General Medicine)Krok 2 - 2013 Question Paper (General Medicine)
Krok 2 - 2013 Question Paper (General Medicine)
 
Krok 2 Medicine - 2016 Question Paper
Krok 2 Medicine - 2016 Question PaperKrok 2 Medicine - 2016 Question Paper
Krok 2 Medicine - 2016 Question Paper
 
Krok 2 Medicine - 2018 Question Paper
Krok 2 Medicine - 2018 Question PaperKrok 2 Medicine - 2018 Question Paper
Krok 2 Medicine - 2018 Question Paper
 
Krok 2 Medicine - 2017 Question Paper
Krok 2 Medicine - 2017 Question PaperKrok 2 Medicine - 2017 Question Paper
Krok 2 Medicine - 2017 Question Paper
 
Krok 2 - 2007 Question Paper (General Medicine)
Krok 2 - 2007 Question Paper (General Medicine)Krok 2 - 2007 Question Paper (General Medicine)
Krok 2 - 2007 Question Paper (General Medicine)
 
Krok 1 - 2010 Question Paper (General medicine)
Krok 1 - 2010 Question Paper (General medicine)Krok 1 - 2010 Question Paper (General medicine)
Krok 1 - 2010 Question Paper (General medicine)
 
Krok 1 - 2006 Question Paper (General medicine)
Krok 1 - 2006 Question Paper (General medicine)Krok 1 - 2006 Question Paper (General medicine)
Krok 1 - 2006 Question Paper (General medicine)
 
Krok 1 - 2014 (Physiology)
Krok 1 - 2014 (Physiology)Krok 1 - 2014 (Physiology)
Krok 1 - 2014 (Physiology)
 
Krok 1 - 2014 (Biochemistry)
Krok 1 - 2014 (Biochemistry)Krok 1 - 2014 (Biochemistry)
Krok 1 - 2014 (Biochemistry)
 
Krok 2 - 2012 Question Paper (General Medicine)
Krok 2 - 2012 Question Paper (General Medicine)Krok 2 - 2012 Question Paper (General Medicine)
Krok 2 - 2012 Question Paper (General Medicine)
 
Krok 1 - 2015 (Pharmacology)
Krok 1 - 2015 (Pharmacology)Krok 1 - 2015 (Pharmacology)
Krok 1 - 2015 (Pharmacology)
 
Krok 2 - 2014 (Surgery)
Krok 2 - 2014 (Surgery)Krok 2 - 2014 (Surgery)
Krok 2 - 2014 (Surgery)
 
Krok 1 - 2015 (Microbiology)
Krok 1 - 2015 (Microbiology)Krok 1 - 2015 (Microbiology)
Krok 1 - 2015 (Microbiology)
 

Viewers also liked

500 single best answers in medicine
500 single best answers in medicine500 single best answers in medicine
500 single best answers in medicinehamadadodo
 
Mcq 1060 questions
Mcq 1060 questionsMcq 1060 questions
Mcq 1060 questionsadrioz
 
High-Yield Internal Medicine Board Review Pearls
High-Yield Internal Medicine Board Review PearlsHigh-Yield Internal Medicine Board Review Pearls
High-Yield Internal Medicine Board Review PearlsKnowmedge
 
Philippine physician licensure exam primer
Philippine physician licensure exam primerPhilippine physician licensure exam primer
Philippine physician licensure exam primerEx D
 
Compilation of previous board examination questions
Compilation of previous board examination questionsCompilation of previous board examination questions
Compilation of previous board examination questionsRhem Rick Corpuz
 
Internal Medicine Board Review - General Internal Medicine Flashcards - by Kn...
Internal Medicine Board Review - General Internal Medicine Flashcards - by Kn...Internal Medicine Board Review - General Internal Medicine Flashcards - by Kn...
Internal Medicine Board Review - General Internal Medicine Flashcards - by Kn...Knowmedge
 
Learning and teaching methods
Learning and teaching methodsLearning and teaching methods
Learning and teaching methodsHidayah Ismawi
 
Vision for a More Effective Medical Curriculum in the Philippines
Vision for a More Effective Medical Curriculum in the PhilippinesVision for a More Effective Medical Curriculum in the Philippines
Vision for a More Effective Medical Curriculum in the PhilippinesReynaldo Joson
 
Future Medical Education
Future Medical EducationFuture Medical Education
Future Medical EducationBen Williams
 
Introduction of Medical Education Domain
Introduction of Medical Education Domain Introduction of Medical Education Domain
Introduction of Medical Education Domain Md Yunus
 
Gina pocket april20
Gina pocket april20Gina pocket april20
Gina pocket april20raissa_09
 
Medical education in india-current status
Medical education in india-current statusMedical education in india-current status
Medical education in india-current statusMitasha Singh
 
August 2011 Physician Licensure Exam
August 2011 Physician Licensure ExamAugust 2011 Physician Licensure Exam
August 2011 Physician Licensure ExamDJ CrissCross
 
Handbook on adult_immunization_2009_(contents)
Handbook on adult_immunization_2009_(contents)Handbook on adult_immunization_2009_(contents)
Handbook on adult_immunization_2009_(contents)raissa_09
 
Teaching learning method
Teaching learning methodTeaching learning method
Teaching learning methodSamira Rahmdel
 

Viewers also liked (20)

500 single best answers in medicine
500 single best answers in medicine500 single best answers in medicine
500 single best answers in medicine
 
Mcq 1060 questions
Mcq 1060 questionsMcq 1060 questions
Mcq 1060 questions
 
High-Yield Internal Medicine Board Review Pearls
High-Yield Internal Medicine Board Review PearlsHigh-Yield Internal Medicine Board Review Pearls
High-Yield Internal Medicine Board Review Pearls
 
Philippine physician licensure exam primer
Philippine physician licensure exam primerPhilippine physician licensure exam primer
Philippine physician licensure exam primer
 
Compilation of previous board examination questions
Compilation of previous board examination questionsCompilation of previous board examination questions
Compilation of previous board examination questions
 
Internal Medicine Board Review - General Internal Medicine Flashcards - by Kn...
Internal Medicine Board Review - General Internal Medicine Flashcards - by Kn...Internal Medicine Board Review - General Internal Medicine Flashcards - by Kn...
Internal Medicine Board Review - General Internal Medicine Flashcards - by Kn...
 
Mcq exam 2011
Mcq exam 2011Mcq exam 2011
Mcq exam 2011
 
Curriculum development in medical education
Curriculum development in medical educationCurriculum development in medical education
Curriculum development in medical education
 
Learning and teaching methods
Learning and teaching methodsLearning and teaching methods
Learning and teaching methods
 
Vision for a More Effective Medical Curriculum in the Philippines
Vision for a More Effective Medical Curriculum in the PhilippinesVision for a More Effective Medical Curriculum in the Philippines
Vision for a More Effective Medical Curriculum in the Philippines
 
Future Medical Education
Future Medical EducationFuture Medical Education
Future Medical Education
 
Medical education in India
Medical education in IndiaMedical education in India
Medical education in India
 
Introduction of Medical Education Domain
Introduction of Medical Education Domain Introduction of Medical Education Domain
Introduction of Medical Education Domain
 
Anatomy & Surgery
Anatomy & SurgeryAnatomy & Surgery
Anatomy & Surgery
 
Gina pocket april20
Gina pocket april20Gina pocket april20
Gina pocket april20
 
Medical education in india-current status
Medical education in india-current statusMedical education in india-current status
Medical education in india-current status
 
August 2011 Physician Licensure Exam
August 2011 Physician Licensure ExamAugust 2011 Physician Licensure Exam
August 2011 Physician Licensure Exam
 
Physiology
Physiology Physiology
Physiology
 
Handbook on adult_immunization_2009_(contents)
Handbook on adult_immunization_2009_(contents)Handbook on adult_immunization_2009_(contents)
Handbook on adult_immunization_2009_(contents)
 
Teaching learning method
Teaching learning methodTeaching learning method
Teaching learning method
 

Similar to ICU ventilator patient management options

Medical surgical nursing quiz
Medical surgical nursing quizMedical surgical nursing quiz
Medical surgical nursing quizNursing Path
 
pulmonary embolism.pptx
pulmonary embolism.pptxpulmonary embolism.pptx
pulmonary embolism.pptxghadeereideh
 
Infective Endocarditis as a Complication of Ventriculoatrial Shunting for Hyd...
Infective Endocarditis as a Complication of Ventriculoatrial Shunting for Hyd...Infective Endocarditis as a Complication of Ventriculoatrial Shunting for Hyd...
Infective Endocarditis as a Complication of Ventriculoatrial Shunting for Hyd...asclepiuspdfs
 
Medical surgical nursing quiz
Medical surgical nursing quizMedical surgical nursing quiz
Medical surgical nursing quizNursing Path
 
Congenital heart disease
Congenital heart diseaseCongenital heart disease
Congenital heart diseasesurendra sharma
 
11. Pulmonary Embolism.2.pptx
11. Pulmonary Embolism.2.pptx11. Pulmonary Embolism.2.pptx
11. Pulmonary Embolism.2.pptxAmos Brighton
 
Central venous catheter complications
Central venous catheter complicationsCentral venous catheter complications
Central venous catheter complicationsRanjita Pallavi
 
Hemophilias Case based study
Hemophilias Case based study Hemophilias Case based study
Hemophilias Case based study Akshat Jain M.D.
 
Respiratory emergencies
Respiratory emergenciesRespiratory emergencies
Respiratory emergenciesFatma Elbadry
 
Krok 2 - 2008 Question Paper (General Medicine)
Krok 2 - 2008 Question Paper (General Medicine)Krok 2 - 2008 Question Paper (General Medicine)
Krok 2 - 2008 Question Paper (General Medicine)Eneutron
 
Previous year question on pneumothorax based on neet pg, usmle, plab and fmge...
Previous year question on pneumothorax based on neet pg, usmle, plab and fmge...Previous year question on pneumothorax based on neet pg, usmle, plab and fmge...
Previous year question on pneumothorax based on neet pg, usmle, plab and fmge...Abhishek Gupta
 
Guillain–Barré syndrome after acute myocardial infarction: A rare presentation
Guillain–Barré syndrome after acute myocardial infarction: A rare presentationGuillain–Barré syndrome after acute myocardial infarction: A rare presentation
Guillain–Barré syndrome after acute myocardial infarction: A rare presentationApollo Hospitals
 
Dr shaf3y cardiology revision
Dr shaf3y cardiology revisionDr shaf3y cardiology revision
Dr shaf3y cardiology revisionyasseryasserahmed
 
EPIGNOSIS-CARDIOLOGY QUIZ-PRELIMS.pptx
EPIGNOSIS-CARDIOLOGY QUIZ-PRELIMS.pptxEPIGNOSIS-CARDIOLOGY QUIZ-PRELIMS.pptx
EPIGNOSIS-CARDIOLOGY QUIZ-PRELIMS.pptxSyedFurqan30
 
Pulmonary embolism - Diagnosis and management
Pulmonary embolism - Diagnosis and managementPulmonary embolism - Diagnosis and management
Pulmonary embolism - Diagnosis and managementDr Vivek Baliga
 
Massive hemoptysis
Massive hemoptysisMassive hemoptysis
Massive hemoptysisgagsol
 
Guillain - Barre syndrome after acute myocardial infarction: A rare presentat...
Guillain - Barre syndrome after acute myocardial infarction: A rare presentat...Guillain - Barre syndrome after acute myocardial infarction: A rare presentat...
Guillain - Barre syndrome after acute myocardial infarction: A rare presentat...Apollo Hospitals
 

Similar to ICU ventilator patient management options (20)

Medical surgical nursing quiz
Medical surgical nursing quizMedical surgical nursing quiz
Medical surgical nursing quiz
 
pulmonary embolism.pptx
pulmonary embolism.pptxpulmonary embolism.pptx
pulmonary embolism.pptx
 
Infective Endocarditis as a Complication of Ventriculoatrial Shunting for Hyd...
Infective Endocarditis as a Complication of Ventriculoatrial Shunting for Hyd...Infective Endocarditis as a Complication of Ventriculoatrial Shunting for Hyd...
Infective Endocarditis as a Complication of Ventriculoatrial Shunting for Hyd...
 
Medical surgical nursing quiz
Medical surgical nursing quizMedical surgical nursing quiz
Medical surgical nursing quiz
 
Congenital heart disease
Congenital heart diseaseCongenital heart disease
Congenital heart disease
 
Taponamiento cardica covid
Taponamiento cardica covidTaponamiento cardica covid
Taponamiento cardica covid
 
11. Pulmonary Embolism.2.pptx
11. Pulmonary Embolism.2.pptx11. Pulmonary Embolism.2.pptx
11. Pulmonary Embolism.2.pptx
 
Central venous catheter complications
Central venous catheter complicationsCentral venous catheter complications
Central venous catheter complications
 
Hemophilias Case based study
Hemophilias Case based study Hemophilias Case based study
Hemophilias Case based study
 
Respiratory emergencies
Respiratory emergenciesRespiratory emergencies
Respiratory emergencies
 
Krok 2 - 2008 Question Paper (General Medicine)
Krok 2 - 2008 Question Paper (General Medicine)Krok 2 - 2008 Question Paper (General Medicine)
Krok 2 - 2008 Question Paper (General Medicine)
 
Previous year question on pneumothorax based on neet pg, usmle, plab and fmge...
Previous year question on pneumothorax based on neet pg, usmle, plab and fmge...Previous year question on pneumothorax based on neet pg, usmle, plab and fmge...
Previous year question on pneumothorax based on neet pg, usmle, plab and fmge...
 
Guillain–Barré syndrome after acute myocardial infarction: A rare presentation
Guillain–Barré syndrome after acute myocardial infarction: A rare presentationGuillain–Barré syndrome after acute myocardial infarction: A rare presentation
Guillain–Barré syndrome after acute myocardial infarction: A rare presentation
 
5 Embolie Pulmonaire.pdf
5 Embolie Pulmonaire.pdf5 Embolie Pulmonaire.pdf
5 Embolie Pulmonaire.pdf
 
Dr shaf3y cardiology revision
Dr shaf3y cardiology revisionDr shaf3y cardiology revision
Dr shaf3y cardiology revision
 
EPIGNOSIS-CARDIOLOGY QUIZ-PRELIMS.pptx
EPIGNOSIS-CARDIOLOGY QUIZ-PRELIMS.pptxEPIGNOSIS-CARDIOLOGY QUIZ-PRELIMS.pptx
EPIGNOSIS-CARDIOLOGY QUIZ-PRELIMS.pptx
 
Pulmonary embolism - Diagnosis and management
Pulmonary embolism - Diagnosis and managementPulmonary embolism - Diagnosis and management
Pulmonary embolism - Diagnosis and management
 
Diary of Practical Training
Diary of Practical Training Diary of Practical Training
Diary of Practical Training
 
Massive hemoptysis
Massive hemoptysisMassive hemoptysis
Massive hemoptysis
 
Guillain - Barre syndrome after acute myocardial infarction: A rare presentat...
Guillain - Barre syndrome after acute myocardial infarction: A rare presentat...Guillain - Barre syndrome after acute myocardial infarction: A rare presentat...
Guillain - Barre syndrome after acute myocardial infarction: A rare presentat...
 

Recently uploaded

College Call Girls Vyasarpadi Whatsapp 7001305949 Independent Escort Service
College Call Girls Vyasarpadi Whatsapp 7001305949 Independent Escort ServiceCollege Call Girls Vyasarpadi Whatsapp 7001305949 Independent Escort Service
College Call Girls Vyasarpadi Whatsapp 7001305949 Independent Escort ServiceNehru place Escorts
 
Call Girl Service Bidadi - For 7001305949 Cheap & Best with original Photos
Call Girl Service Bidadi - For 7001305949 Cheap & Best with original PhotosCall Girl Service Bidadi - For 7001305949 Cheap & Best with original Photos
Call Girl Service Bidadi - For 7001305949 Cheap & Best with original Photosnarwatsonia7
 
call girls in green park DELHI 🔝 >༒9540349809 🔝 genuine Escort Service 🔝✔️✔️
call girls in green park  DELHI 🔝 >༒9540349809 🔝 genuine Escort Service 🔝✔️✔️call girls in green park  DELHI 🔝 >༒9540349809 🔝 genuine Escort Service 🔝✔️✔️
call girls in green park DELHI 🔝 >༒9540349809 🔝 genuine Escort Service 🔝✔️✔️saminamagar
 
Call Girls Service Noida Maya 9711199012 Independent Escort Service Noida
Call Girls Service Noida Maya 9711199012 Independent Escort Service NoidaCall Girls Service Noida Maya 9711199012 Independent Escort Service Noida
Call Girls Service Noida Maya 9711199012 Independent Escort Service NoidaPooja Gupta
 
Hematology and Immunology - Leukocytes Functions
Hematology and Immunology - Leukocytes FunctionsHematology and Immunology - Leukocytes Functions
Hematology and Immunology - Leukocytes FunctionsMedicoseAcademics
 
Call Girls Jayanagar Just Call 7001305949 Top Class Call Girl Service Available
Call Girls Jayanagar Just Call 7001305949 Top Class Call Girl Service AvailableCall Girls Jayanagar Just Call 7001305949 Top Class Call Girl Service Available
Call Girls Jayanagar Just Call 7001305949 Top Class Call Girl Service Availablenarwatsonia7
 
Call Girls Service Chennai Jiya 7001305949 Independent Escort Service Chennai
Call Girls Service Chennai Jiya 7001305949 Independent Escort Service ChennaiCall Girls Service Chennai Jiya 7001305949 Independent Escort Service Chennai
Call Girls Service Chennai Jiya 7001305949 Independent Escort Service ChennaiNehru place Escorts
 
Call Girl Nagpur Sia 7001305949 Independent Escort Service Nagpur
Call Girl Nagpur Sia 7001305949 Independent Escort Service NagpurCall Girl Nagpur Sia 7001305949 Independent Escort Service Nagpur
Call Girl Nagpur Sia 7001305949 Independent Escort Service NagpurRiya Pathan
 
Call Girl Lucknow Mallika 7001305949 Independent Escort Service Lucknow
Call Girl Lucknow Mallika 7001305949 Independent Escort Service LucknowCall Girl Lucknow Mallika 7001305949 Independent Escort Service Lucknow
Call Girl Lucknow Mallika 7001305949 Independent Escort Service Lucknownarwatsonia7
 
Call Girls ITPL Just Call 7001305949 Top Class Call Girl Service Available
Call Girls ITPL Just Call 7001305949 Top Class Call Girl Service AvailableCall Girls ITPL Just Call 7001305949 Top Class Call Girl Service Available
Call Girls ITPL Just Call 7001305949 Top Class Call Girl Service Availablenarwatsonia7
 
Call Girls Kanakapura Road Just Call 7001305949 Top Class Call Girl Service A...
Call Girls Kanakapura Road Just Call 7001305949 Top Class Call Girl Service A...Call Girls Kanakapura Road Just Call 7001305949 Top Class Call Girl Service A...
Call Girls Kanakapura Road Just Call 7001305949 Top Class Call Girl Service A...narwatsonia7
 
Noida Sector 135 Call Girls ( 9873940964 ) Book Hot And Sexy Girls In A Few C...
Noida Sector 135 Call Girls ( 9873940964 ) Book Hot And Sexy Girls In A Few C...Noida Sector 135 Call Girls ( 9873940964 ) Book Hot And Sexy Girls In A Few C...
Noida Sector 135 Call Girls ( 9873940964 ) Book Hot And Sexy Girls In A Few C...rajnisinghkjn
 
Call Girls Hebbal Just Call 7001305949 Top Class Call Girl Service Available
Call Girls Hebbal Just Call 7001305949 Top Class Call Girl Service AvailableCall Girls Hebbal Just Call 7001305949 Top Class Call Girl Service Available
Call Girls Hebbal Just Call 7001305949 Top Class Call Girl Service Availablenarwatsonia7
 
High Profile Call Girls Jaipur Vani 8445551418 Independent Escort Service Jaipur
High Profile Call Girls Jaipur Vani 8445551418 Independent Escort Service JaipurHigh Profile Call Girls Jaipur Vani 8445551418 Independent Escort Service Jaipur
High Profile Call Girls Jaipur Vani 8445551418 Independent Escort Service Jaipurparulsinha
 
Hemostasis Physiology and Clinical correlations by Dr Faiza.pdf
Hemostasis Physiology and Clinical correlations by Dr Faiza.pdfHemostasis Physiology and Clinical correlations by Dr Faiza.pdf
Hemostasis Physiology and Clinical correlations by Dr Faiza.pdfMedicoseAcademics
 
Pharmaceutical Marketting: Unit-5, Pricing
Pharmaceutical Marketting: Unit-5, PricingPharmaceutical Marketting: Unit-5, Pricing
Pharmaceutical Marketting: Unit-5, PricingArunagarwal328757
 
VIP Call Girls Mumbai Arpita 9910780858 Independent Escort Service Mumbai
VIP Call Girls Mumbai Arpita 9910780858 Independent Escort Service MumbaiVIP Call Girls Mumbai Arpita 9910780858 Independent Escort Service Mumbai
VIP Call Girls Mumbai Arpita 9910780858 Independent Escort Service Mumbaisonalikaur4
 
Russian Call Girl Brookfield - 7001305949 Escorts Service 50% Off with Cash O...
Russian Call Girl Brookfield - 7001305949 Escorts Service 50% Off with Cash O...Russian Call Girl Brookfield - 7001305949 Escorts Service 50% Off with Cash O...
Russian Call Girl Brookfield - 7001305949 Escorts Service 50% Off with Cash O...narwatsonia7
 
Call Girls Jp Nagar Just Call 7001305949 Top Class Call Girl Service Available
Call Girls Jp Nagar Just Call 7001305949 Top Class Call Girl Service AvailableCall Girls Jp Nagar Just Call 7001305949 Top Class Call Girl Service Available
Call Girls Jp Nagar Just Call 7001305949 Top Class Call Girl Service Availablenarwatsonia7
 
Housewife Call Girls Hsr Layout - Call 7001305949 Rs-3500 with A/C Room Cash ...
Housewife Call Girls Hsr Layout - Call 7001305949 Rs-3500 with A/C Room Cash ...Housewife Call Girls Hsr Layout - Call 7001305949 Rs-3500 with A/C Room Cash ...
Housewife Call Girls Hsr Layout - Call 7001305949 Rs-3500 with A/C Room Cash ...narwatsonia7
 

Recently uploaded (20)

College Call Girls Vyasarpadi Whatsapp 7001305949 Independent Escort Service
College Call Girls Vyasarpadi Whatsapp 7001305949 Independent Escort ServiceCollege Call Girls Vyasarpadi Whatsapp 7001305949 Independent Escort Service
College Call Girls Vyasarpadi Whatsapp 7001305949 Independent Escort Service
 
Call Girl Service Bidadi - For 7001305949 Cheap & Best with original Photos
Call Girl Service Bidadi - For 7001305949 Cheap & Best with original PhotosCall Girl Service Bidadi - For 7001305949 Cheap & Best with original Photos
Call Girl Service Bidadi - For 7001305949 Cheap & Best with original Photos
 
call girls in green park DELHI 🔝 >༒9540349809 🔝 genuine Escort Service 🔝✔️✔️
call girls in green park  DELHI 🔝 >༒9540349809 🔝 genuine Escort Service 🔝✔️✔️call girls in green park  DELHI 🔝 >༒9540349809 🔝 genuine Escort Service 🔝✔️✔️
call girls in green park DELHI 🔝 >༒9540349809 🔝 genuine Escort Service 🔝✔️✔️
 
Call Girls Service Noida Maya 9711199012 Independent Escort Service Noida
Call Girls Service Noida Maya 9711199012 Independent Escort Service NoidaCall Girls Service Noida Maya 9711199012 Independent Escort Service Noida
Call Girls Service Noida Maya 9711199012 Independent Escort Service Noida
 
Hematology and Immunology - Leukocytes Functions
Hematology and Immunology - Leukocytes FunctionsHematology and Immunology - Leukocytes Functions
Hematology and Immunology - Leukocytes Functions
 
Call Girls Jayanagar Just Call 7001305949 Top Class Call Girl Service Available
Call Girls Jayanagar Just Call 7001305949 Top Class Call Girl Service AvailableCall Girls Jayanagar Just Call 7001305949 Top Class Call Girl Service Available
Call Girls Jayanagar Just Call 7001305949 Top Class Call Girl Service Available
 
Call Girls Service Chennai Jiya 7001305949 Independent Escort Service Chennai
Call Girls Service Chennai Jiya 7001305949 Independent Escort Service ChennaiCall Girls Service Chennai Jiya 7001305949 Independent Escort Service Chennai
Call Girls Service Chennai Jiya 7001305949 Independent Escort Service Chennai
 
Call Girl Nagpur Sia 7001305949 Independent Escort Service Nagpur
Call Girl Nagpur Sia 7001305949 Independent Escort Service NagpurCall Girl Nagpur Sia 7001305949 Independent Escort Service Nagpur
Call Girl Nagpur Sia 7001305949 Independent Escort Service Nagpur
 
Call Girl Lucknow Mallika 7001305949 Independent Escort Service Lucknow
Call Girl Lucknow Mallika 7001305949 Independent Escort Service LucknowCall Girl Lucknow Mallika 7001305949 Independent Escort Service Lucknow
Call Girl Lucknow Mallika 7001305949 Independent Escort Service Lucknow
 
Call Girls ITPL Just Call 7001305949 Top Class Call Girl Service Available
Call Girls ITPL Just Call 7001305949 Top Class Call Girl Service AvailableCall Girls ITPL Just Call 7001305949 Top Class Call Girl Service Available
Call Girls ITPL Just Call 7001305949 Top Class Call Girl Service Available
 
Call Girls Kanakapura Road Just Call 7001305949 Top Class Call Girl Service A...
Call Girls Kanakapura Road Just Call 7001305949 Top Class Call Girl Service A...Call Girls Kanakapura Road Just Call 7001305949 Top Class Call Girl Service A...
Call Girls Kanakapura Road Just Call 7001305949 Top Class Call Girl Service A...
 
Noida Sector 135 Call Girls ( 9873940964 ) Book Hot And Sexy Girls In A Few C...
Noida Sector 135 Call Girls ( 9873940964 ) Book Hot And Sexy Girls In A Few C...Noida Sector 135 Call Girls ( 9873940964 ) Book Hot And Sexy Girls In A Few C...
Noida Sector 135 Call Girls ( 9873940964 ) Book Hot And Sexy Girls In A Few C...
 
Call Girls Hebbal Just Call 7001305949 Top Class Call Girl Service Available
Call Girls Hebbal Just Call 7001305949 Top Class Call Girl Service AvailableCall Girls Hebbal Just Call 7001305949 Top Class Call Girl Service Available
Call Girls Hebbal Just Call 7001305949 Top Class Call Girl Service Available
 
High Profile Call Girls Jaipur Vani 8445551418 Independent Escort Service Jaipur
High Profile Call Girls Jaipur Vani 8445551418 Independent Escort Service JaipurHigh Profile Call Girls Jaipur Vani 8445551418 Independent Escort Service Jaipur
High Profile Call Girls Jaipur Vani 8445551418 Independent Escort Service Jaipur
 
Hemostasis Physiology and Clinical correlations by Dr Faiza.pdf
Hemostasis Physiology and Clinical correlations by Dr Faiza.pdfHemostasis Physiology and Clinical correlations by Dr Faiza.pdf
Hemostasis Physiology and Clinical correlations by Dr Faiza.pdf
 
Pharmaceutical Marketting: Unit-5, Pricing
Pharmaceutical Marketting: Unit-5, PricingPharmaceutical Marketting: Unit-5, Pricing
Pharmaceutical Marketting: Unit-5, Pricing
 
VIP Call Girls Mumbai Arpita 9910780858 Independent Escort Service Mumbai
VIP Call Girls Mumbai Arpita 9910780858 Independent Escort Service MumbaiVIP Call Girls Mumbai Arpita 9910780858 Independent Escort Service Mumbai
VIP Call Girls Mumbai Arpita 9910780858 Independent Escort Service Mumbai
 
Russian Call Girl Brookfield - 7001305949 Escorts Service 50% Off with Cash O...
Russian Call Girl Brookfield - 7001305949 Escorts Service 50% Off with Cash O...Russian Call Girl Brookfield - 7001305949 Escorts Service 50% Off with Cash O...
Russian Call Girl Brookfield - 7001305949 Escorts Service 50% Off with Cash O...
 
Call Girls Jp Nagar Just Call 7001305949 Top Class Call Girl Service Available
Call Girls Jp Nagar Just Call 7001305949 Top Class Call Girl Service AvailableCall Girls Jp Nagar Just Call 7001305949 Top Class Call Girl Service Available
Call Girls Jp Nagar Just Call 7001305949 Top Class Call Girl Service Available
 
Housewife Call Girls Hsr Layout - Call 7001305949 Rs-3500 with A/C Room Cash ...
Housewife Call Girls Hsr Layout - Call 7001305949 Rs-3500 with A/C Room Cash ...Housewife Call Girls Hsr Layout - Call 7001305949 Rs-3500 with A/C Room Cash ...
Housewife Call Girls Hsr Layout - Call 7001305949 Rs-3500 with A/C Room Cash ...
 

ICU ventilator patient management options

  • 1. 4. A 45-yesr-old male vehicular accident patient was admitted at the ICU for ventilator support after he was brought to the ER unconscious. Prior to the accident, he was previously well. He was stable until the 4th ICU day when he had fever and yellowish secretions. Chest x-ray showed new infiltrates on the left mid-lung field. Recommended initial antibiotic treatment (HPIM p. 2139): a. IV ampicillin-sulbactam b. IV linezoid c. Vancomycin d. Ceftazimide + IV levofloxacin 5. A 57-year-old male was admitted because of hemoptysis amounting to 1 ½ cups. He had a history of recurrent hemoptysis over the last 10 years. Chest x-ray showed bilateral basal cystic lucencies. Chest CT scan showed bilateral dilated airways at the bases. Fiberoptic bronchoscopy showed bleeding coming from both basal air passages. Appropriate management for this patient (HPIM p. 2144) a. Bronchial arterial embolization b. Surgical resection c. Forgarty catheter insertion d. Pulmonary physiotherapy
  • 2. 6. A 67-year-old male, non-smoker consulted at the OPD because of occasional shortness of breath and chest wheezing 1-2x a month. On PE, patient had clear breath sounds . Chest x-ray was unremarkable and pulmonary function test showed more than 15% improvement post-bronchodilator in FEV₁ and FEV₁ %. Management for this patient is (HPIM p. 2113): a. ICS low-dose + β₂ agonist prn b. LABA + ICS low-dose + β₂ agonist prn c. Inhaled β₂ agonist d. Oral β₂ agonist 7. A 55-year-old male was admitted because of low-grade fever of 3 weeks associated with cough productive of greenish foul-smelling phlegm. PE showed rales at the right lung base and clubbing. Chest x-ray showed cavity with an air fluid level at the right base . The patient was given 6 weeks of clindamycin but symptoms persisted and x-ray findings remained the same. Management of this patient is (HPIM p. 2146): a. 4-6 weeks of another antibiotic b. TB treatment c. Surgery on lesion d. Anti-fungal treatment
  • 3. Gynecology consultant plan was to do surgery for the myoma, but only after the medical problem was managed appropriately. Your management of the patient prior to surgery is (HPIM p. 2176): a. Low molecular weight heparin b. Fibrinolysis with rTPA (tissue plasminogen activator) c. IVC filter insertion d. Fresh frozen plasma transfusion 1. The following condition/s can result to hypokinetic pulses: T a. Left ventricular failure T b. Shock from massive blood loss T c. Large perincardial effusion F d. Mitral regurgitation F e. Aortic regurgitation
  • 4. 2. Proposed mechanism/s of cardiac arrhythmias: a. Alterations in impulse initiation b. Automacity c. Early after depolarizations and triggered automaticity d. Abnormal impulse conduction e. Phase 4 depolarization 3. Causes of heart failure T a. Myocardial infarction F b. Pericarditis T c. Thyrotoxicosis F d. Dissecting aortic aneurysm T e. Chronic anemia
  • 5. 4. Which of the following statement/s is/are true regarding diagnostic examinations in heart failure? T a. New-onset heart failure warrants a complete blood count, urinalysis, BUN, creatinine, liver enzymes and electrolytes. F b. The most useful index of LV function is the EF calculated by the end-diastolic volume divided by stroke volume. T c. the gold standard for assessing LV mass and volume is the cardiac MRI T d. Both B-type natriuretic peptide (BNP) and N-terminal pro-BNP are sensitive markers for heart failure even in those with preserved EF F e. A normal ECG virtually excludes LV systolic dysfunction. 5. Features of peripartum cardiomyopathy T a. Prognosis depends on whether the heart size returns to normal after the first episode of CHF T b. May develop during the last trimester of pregnancy or within 6 months of delivery F c. Typical patients are nulliparous and >30 years F d. clinical outcome is dependent on whether the delivered via caesarian section or spontaneous normal vaginal delivery T e. the cause is unknown
  • 6. 6. Cardinal manifestations of acute pericarditis: T a. Pain T b. Pericardial friction rub T c. Pericardial effusion and tamponade T d. Paradoxical pulse F e. Shortness of breath 8. Which of the following is/are risk of factor/s for stroke in a patient with atrial fibrillation? F a. Obesity F b. Mitral Regurgitation T c. Diabetes Mellitus T d. Spontaneous echo contrast F e. Market right atrial enlargement 9. Unstable angina is defined as defined as angina pectoris or equavalent chest discomfort with the following features: T a. it occurs at rest (or with )usually fasting >10 min T b. it is severe and of T c. it occurs with a crescendo pattern (i.e. distinctly more severe prolonged or frequent than previously) F d. positive cardiac F e. BP <90/ 60mmhg requiring inotropic support
  • 7. 10. Which of the following is/are ECG clue/s supporting the diagnosis of ventricular tachycardia? T a. Concordance of QRS complex in all precordial leads F b. Frontal plane axis + 90 to 110 degrees T c. QRS duration >140ms for RBBB type V1 morphology T d. AV dissociation (atrial capture, fusion beats) T e. RS dominant S in V6 for RBBB VT 11. Patients with the following congenital heart disease/s should be advised to avoid pregnancy: T a. VSD with Eisenmengerization T b. Congenital aortic stenosis T c. Coarctation of the aorta F d. Large patent ductus arteriosus T e. Marfan syndrome
  • 8. 12. Features of cardiogenic shock: T a. Prompt reperfusion, efforts to reduce infarct size, ad treatment of ongoing ischemia and other complications of MI appear to have reduced its incidence from 20% to about 7% F b. Inferior location of the myocardial infarction increases the risk T c. typically , at least 40% of the myocardium is damaged by old scars and new infarcts F d. Plans for percutaneous transcoronary intervention should be delayed until patient is stabilized via inotropics T e. typically, patients who develop cardiogenic shock have severe multivessel coronary artery disease with evidence of “placemeal” necrosis extending outward from the original infarct zone 13. Which of the following is/are true about the patterns and causes of hypertension? T a. Blunting of the day-night BP pattern is seen in sleep apnea and autonomic dysfunction T b. white coat hypertension is associated with sustained hypertension and target organ damage F c. BP tends to be higher at noontime until late in the afternoon T d. thyrotoxicosis leads to systolic hypertension while hypothyroidism causes diastolic hypertension F e. Night time “dips” in BP has been shown to increased cardiovascular disease risk
  • 9. 14. Cause/s of systolic hypertension with widened pulse pressure: T a. Hyperkinetic heart syndrome F b. Hypothermia F c. Hypothyroidism T d. Aortic regurgitation F e. Mitral regurgitation 15. Features of resistant hypertension: F a. Abrupt increase of BP in a patient with underlying hypertension F b. More common in young patients T c. May be related to nonadherence to therapy, obesity and excessive alcohol intake, and use of any number of nonprescription and prescription drugs T d. refers to patients with BPs persistently >140/90 mmHg despite taking three or more antihypertensive agents, including a diuretic, in reasonable combination and at full doses. T e. Evaluation of patients include home BP monitoring to determine if office Bpare representative of the usual BP.
  • 10. 16. Features of aortic aneurysms: T a. Many patients with aortic aneurysms have co-existing risk factors for atherosclerosis F b. Cystic medial necrosis characteristically affects the distal aorta T c. Familial clustering of aortic aneurysms occur in 20% of patients, suggesting a hereditary basis for the disease T d. the infectious causes of aortic aneurysms include syphilis, tuberculosis, and other bacterial infections. F e. vasculitides associated with aortic aneurysm include Takayasu’ s arteritis and giant cell arteritis, which may cause proximal ascending aorta and descending thoracic aorta 17. Major manifestations of Rheumatic Fever based on Revised Jones Criteria: T a. Carditis F b. Monoarthritis F c. Myaglia T d. Subcutaneous nodules T e. Chorea
  • 11. 18. Clinical variables that increase the likehood of deep venous thrombosis: T a. Active cancer F b. Bilateral calf swelling T c. Pitting edema T d. Surgery within 4 weeks F e. Non-healing wound at the toes 19. Organisms commonly involved in infective endocarditis: T a. Staphylococcus F b. Pseudomonas aeruginosa T c. Enterococci F d. Corynebacterium T e. Streptococcus
  • 12. 20. Clinical presentation of STEMI: T a. In up to ½ of cases, a precipitating factor is present F b. Pain is the most common presenting complaint and is uniformly present T c. The precordium is usually quiet T d. Paradoxic spitting of 52 T e. Fever 21. Clinical manifestations of acute rheumatic fever: T a. There is a latent period of -3 weeks (1-5 weeks) between the precipitating group A streptococcal infection and the appearance of the clinical features F b. Chorea and indolent carditis may follow prolonged latent periods lasting up to 12 months T c. A streptococcal infection is commonly subclinical, in these cases it can only be confirmed using streptococcal antibody testing F d. Carditis is more commonly present than polyarthritis T e. Erythema marginatum and subcutaneous nodules are now rare, being found in <5% of cases
  • 13. 22. Clinical features of systemic arterial hypertension: F a. cardiac compensation is by eccentric left ventricular hypertrophy F b. the ECG is more sensitive than 2D echocardiography for detecting left ventricular hypertrophy F c. it is an independent predisposing factor for coronary artery disease, stroke, renal disease and peripheral vascular disease but not heart failure T d. Opthalmoscopic examination provides the opportunity to observe the progress of vascular effects of hypertension T e. systolic blood pressure has a greater effect on morbidity and mortality than diastolic blood pressure 23. Features of mitral valve prolapse: F a. In most patients, the cause is unknown, but in some it appears to be genetically determined elastin disorder T b. A frequent finding in patients with heritable disorders of connective tissue F c. the anterior leaflet is usually more affected than the posterior leaflet T d. may occur as a sequel to acute rheumatic fever T e. More common in females and occurs most commonly between the ages of 15 and 30 years
  • 14. 2.What is cause of falsely negative treadmill exercise test? a. Abnormal serumpotassium b. Use of cardioactive drugs (e.g. digitalis, anti-arrhythmic agents) c. Obstruction limited to the circumflex coronary artery d. Asymptomatic men under the age of 40 3. Most common site of focal spasm in Prinzmetal’s variant angina a. Proximal left anterior descending coronary artery b. Distal left anterior descending coronary artery c. Left circumflex artery d. Right coronary artery 4. Gold standard in the assessment of myocardial viability a. SPECT b. Cardiac MRI c. PET Scan d. Stress Echocardiography
  • 15. 6. a. b. c. Not a feature of atrial septal defects (ASD): Occurs more frequently in females Ostium primum defects type are common in Down’s syndrome Surgical closure should be carried out in patients small defects and trivial left to right shunts and in those with severe pulmonary vascular disease wihtout a significant left-to-right shunt d. ASD of the sinus venosis or ostium secundum types rarely die before the fifth decade. 7. Not a feature of ventricular septal defect (VSD): a. Patients with large VSD and pulmonary hypertension are those at greatest risk for developing pulmonary vascular obstruction b. Large defects should be corrected surgically early in life when pulmonary vascular disease is still reversible or not yet developed c. In patients with severe pulmonary vascular obstruction (Eisenmenger syndrome), symptoms in adult life consist of exertional dyspnea, chest pain, syncope, and hemoptysis d. Spontaneous closure is common even in moderate-sized defects early in adulthood
  • 16. 8. NOT a pharmacologic property of amiodarone: a. Class III agent delays repolarization due to inhibition of potassium current or activation of depolarizing current b. Useful in atrial fibrilliation, ventricular tachycardia and sinus bradycardia c. Potential toxic effects include both hypothyroidism, chronic lung disease, and liver function test abnormalities d. Non-inferior to automatic implantable cardioverter defibrillator indecreasing arrhythmetic recurrence and deaths 9. In patients with hypertrophic cardiomyopathy, this type of drug amellorates angina pectoris and syncope in 1/3 to ½ of patients: a. Nondihydropyridine calcium channel blockers b. Ace inhibitors c. Beta blockers d. Diuretics
  • 17. 10. A major feature of metabolic syndrome: a. FBS ≥ 110mg/dL b. HDL in men < 50mg/dL c. Blood pressure > 130/90 d. Triglycerides > 200mg/dL 11. Hallmark of the restrictive cardionyopathies a. Systolic dysfunction b. Myocardial fibrosis c. Abnormal diastolic function d. Concomittant endocardial involvment 12. Not a treatment of Prinzmetal’s angina: a. ASA b. Nitrates c. Calcium channel blockers d. Alpha-adrenoceptor blockers
  • 18. 13. Not an absolute contraindication to the use of fibrinolytics in STEMI: a. History of cerebrovascular hemorrhage at any time b. Nonhemorrhagic stroke 5 years ago c. BP > 180/110mmHg d. Active bleeding excluding menstruation 14. Most frequent arrythmia seen in alcoholic cardiotoxicity or “holiday heart syndrome” a. Ventricular Tachycardia b. Frequent PVCs c. Atrial flutter d. Atrial fibrillation 15.Not cardinal manifestation of severe aortic stenosis a. Chest pain b. Syncope c. Edema d. Congestive heart failure
  • 19. 16. Most common congenital cardiovascular cause of hypertension: a. Bicuspid aortic Valve b. Coarctication of the Aorta c. Patent Ductus Arteriosus d. Coronary Arteriovenous Fistula 17. Best diagnostic modality in pericardial effusion: a. Chest x-ray b. Myocardial perfusion imaging c. 2D echocardiogram d. Cardiac catherization 20. Hormone found to be helpful in differentiating dyspnea caused by cardiac vs other etiologies a. BUN b. BNP c. ANP d. D-Dimer
  • 20. 20. Pharmacologic agent found to reduce mortality in heart failure patients: a. Digoxin b. ACE inhibitors c. Salt restriction d. Isosorbide mononitrate 21. Management of acute myocardial infarction: a. Aspirin is effective across the entire spectrum of acute coronary syndromes b. Nitrates increase preload, thus decreasing myocardial oxygen demand c. Morphine causes venous pooling leading to increased cardiac output d. Supplemental oxygen is helpful in decreasing the size of the infarcted myocardium even in patients with normal arterial O2 saturation 22. What is the EKG diagnosis? a. First degree atrioventricular block b. 2nd degree atrioventricular block Type 1 c. 2nd degree atrioventricular block Type 2 d. 3rd degree atrioventricular block
  • 21. 23. A feature of mitral stenosis: a. VT is most common cause of death b. Atherosclerosis remains the leading cause of mitral stenosis c. The normal mitral valve orifice is 2-4cm2 d. Percutaneous valvuloplasty is treatment of choice for symptomatic patients, unless technically not amenable 25. Not present in significant right ventricular infarction: a. Jugular venous distention b. Kussmaul’s sign c. Bilateral mid to basal rales d. Hepatomegaly 26. Drug of choice for medical management of aortic aneurysms a. Beta blocker b. Ace inhibitors c. Calcium channel blocker d. nitrates
  • 22. 27. Mechanisms of action of thienopyridines: a. Inhibition of thromboxane synthesis b. Inhibition of the clotting factor Xa c. Inhibition of the P2Y12 ADP receptor d. Inhibition of the IIIB/IIIA receptor 28. A principle of treadmill exercise testing: a. Sensitivity for coronary disease is the same as coronary angiogram b. Specificity for coronary disease is unaffected by the presence of LVH, digoxin use, or resting ST-T abnormality c. Contraindications include active myocarditis , symptomatic severe aortic stenosis and early acute myocardial infarction d. Routine screening is recommended for all adults over 30 years of age
  • 23. 29. Not a clinical manifestation of orthostatic hypotension: a. Polypharmacy with antihypertensive or antidepressant drugs is often noncontributory b. After physical deconditioning such as after prolonged illness with recumbency, especially in elderly individuals with reduced muscle tone c. Occurs in hypovolemic states from diuretics, excessive sweating, diarrhea, vomiting, hemorrhage or adrenal insufficiency d. Falls in 10 mmHg systolic or 29 mmHg diastolic blood pressure on standing is diagnostic when due to autonomic dysfunction from various neurologic causes 30. Feature of Atrial Flutter: a. Atrial flutter does not require anticoagulation because of low thromboembolic risk. b. Cardioversion is not effective in terminating hemodynamically tolerated tachyarrhythmias, and amiodarone is therefore preffered c. Atrial flutter will not cause tachycardia-induced severe left ventricular dysfnction d. Rate control with calcium antagonist, betablockers or lanoxin maybe difficult
  • 24. 31. Pathophysiology of hypertension: a. Both hypertropic (increased cell number increased cell size and increased deposition of intercellular matrix) and eutrophic (no change in the amount of material in the vessel wall) vascular remodelling result in decreased lumen size and contribute to increased peripheral resistance b. The renin-angiotensin-aldosterone system contributes to arterial pressure regulation via angiotensin II which causes vasoconstriction, and aldosterone which causes natriuresis. c. Stiffer arteries in hypertension, particularly in arteriosclerotic patients results in high systolic blood pressures and narrow pulse pressures. d. Pheochromocytoma is an example of hypertension related to increased salt retention. 32. A characteristics of secondaryhypertension: a. Hyperthyroidism causes systolic hypertension with a narrow pulse pressure b. Beta blockers are contraindicated in bilateral renal artery stenosis, or unilateral stenosis in s solitary kidney c. Surgical excision is the definitive treatment of pheochromcytoma and results in cure in-90% of patients d. Pheochromocytoma is the most common cause
  • 25. 33. A feature of hypertension in pregnancy: a. For women with severe preeclamsia , BP >160/110 must be aggressively treated; and delaying the delivery of the fetus and placenta as much as possible improves maternal outcomes. b. During pregnancy, a blood pressure of 140/90mmHg is considered to be abnormally elevated and is associated with an increase in perinatal morbidity and mortality c. Magnesium sulfate is no longer used for the prevention and treatment of eclamptic seizures d. Methyldopa, ARBs and hydralazine are used most often because they have no known adverse effect on the fetus. 34. A feature of aortic aneurysms: a. If asymptomatic, surgery is indicated if the diameter is > 6 cm in thoracic aneurysms and > 5.5 abdominal aneurysms. b. Cystic medial necrosis is the most common cause of all aortic aneurysms. c. Atherosclerosis is the condition not frequently associated with aneurysms of the aortic archand descending thoracic aorta. d. Abdominal aortic aneurysms occur more frequently in females.
  • 26. 35. Management of aortic dissections: a. MPI with sestamibi is the most sensitive test. b. Ct and MRI are accurate tests in this condition, while transesophangeal echocardiography is generally unreliable for this coondition. c. Systemic hypertension, cystic medial necrosis, Marfan’s syndrome are rare predisposing factors. d. Medical management is the preferred initial therapy for proximal Type A aortic dissection. 36.A feature of neurogenic syncope: a. beta-blockers are popular for the condition because trials have shown it to be clearly effective b. ICD implantation are indicated in these cases c. Tilt table testing has a poorer sensitivity (20-75%) compared to its specificity (nearly 90%), but specificity decreases with pharmacologic provocation d. Usually accompanied by tonic clonic episodes
  • 27. 37. An indication for infective endocarditis prophylaxis: a. Atrial septal defects deemed uncomplicated b. s/p prosthetic mitral valve replacement c. LV aneurysms d. Patients with permanent pacemakers 38. Pulsus paradoxicus consist of a greater than normal inspiratory decline in systolic arterial pressure by: a. 10mmHg b. 15mmHg c. 20mmHg d. 25mmHg 39. The significant ankle bachial index to suspect PAD: a. <1 b. <1.5 c. <2 d. <2.5
  • 28. 40. Not a cause of aortic regurgitation: a. Congenital b. Marfan’s syndrome c. Syphilis d. Carcinoid 41. Cornerstones of modern therapy for heart failure with a depressed ejection fraction to prevent disease progression a. Diuretics and Beta-blockers b. Beta blockers and ACE-inhibitors c. Ace-inhibitors or Angiotensin receptor blockers d. Diuretics and ACE-inhibitors 42. A late sign in cor pulmonale resulting from low cardiac output with systemic vasoconstriction and ventilation-perfusion mismatches in the lung: a. carvallo’s sign b. Dyspnea c. Clubbing d. cyanosis
  • 29. 43. Most common maalignant cardiac tumor: a. Sarcoma b. Myxoma c. Hemangioma d. Lymphoma 44. Not a failure of cardiovascular syndrome caused by thiamine deficiency: a. Characterized by high-output heart failure, tachycardia, and often elevated left and right ventricular filling pressures b. Major cause of the high-output state is vasomotor depression leading to reduced response to thiamin and may take 1-2 weeks before effects are clinically apparent c. Reduced systemic vascular resistance d. Cardiac examination reveal a wide pulse pressure, tachycardia, a third heart sound, and frequently, an apical systolic murmur.
  • 30. 45. Marks a turning point in the course of a patient with a mitral stenosis, associated with acceleration of the rate at which symptoms progress: a. Infective Endocarditis b. LV dilatation c. Permanent Atrial Fibrillation d. MV orifice of <1.5 cm2 46. Not an indication for coronary angiography in chronic stable angina: a. Patients with chronic stable angina pectoris who are severely symptomatic despite medical therapy and who are being considered for revascularization, i.e., a percutaneous coronary artery bypass grafting (CABG) b. Patients with troublesome symptoms that present diagnostic difficulties in whom there is a need to confirm or rule out the diagnosis of IHD c. Patients with no evidence of ischemia on noninvasive testing and no-clinical or laboratory evidence of ventricular dysfunction d. Patients with known or possible angina pectoris who have survived cardiac arrest
  • 31. 47. Advantageof bioprosthetic and mechanical valve replacement: a. Bioprosthetic valves are superior to mechanical prosthetic valves in durability andthrombogenicity b. Lifetime anicoagulation is warranted for mechanical prosthetic valves c. Valve replacement is indicated for asymptomatic moderate mitral or aortic regurgitation with well-preserved left ventricular systolic function d. Prosthetic valves pose a low risk for infective endocarditis and antibiotic prophylaxis is not indicated prior to dental procedures that may cause transient bacteremia 1. A 55 year old female was admitted due to shortness of breath with associated bipedal edema. Your resident saw the patient earlier and told you that the present working impression is cor pulmonale. Which of the following point in the history and PE does not contribute to the diagnosis? a. Chronic productive cough for 3 months in each of two consecutive years b. 30 pack year smoker up to present c. Normal JVP d. She has been on warfarin regimen for the past year with irregular intake for history of deep venous thrombosis
  • 32. 2. The cardiology fellow was called to do a pericardiocentesis procedure on one of the patients at the emergency room. As he was doing the procedure, he asks you, which of the ff is not a feature of cardiac tamponade? a. Hypotension b. Soft or absent heart sounds c. Fixed wide Split S2 d. Jugular venous distention with a prominent x descent and absent y descent 3. You were making rounds at the wards when the nurse referred a patient to you. Apparently, the patient lost consciousness. What should you NOT do? a. Patient should be placed supine with the head turned to the side b. Clothing that fits tightly around the neck or waist should be loosened c. Sprinkling cold water on the face may be helpful d. Have the patient drink cold water
  • 33. 4. While on your duty, you were referred a patient with infective endocarditis. He has been admitted for 5 weeks already but today, he became unstable complaining of shortness of breath. There was a new murmur according to the intern. Your resident is thinking the patient might need urgent heart surgery. Which of the following is not an indication for cardiac surgical intervention in patients with endocarditis? a. Moderate to severe congestive heart failure due to valve dysfunction b. Persistent bacteremia despite multiple anti microbial therapy c. Presence of effective but expensive microbicidal therapy (e.g. fungal or brucella endocarditis) d. S.aureus prosthetic valve endocarditis with intracardiac complication 5. 69/F with dizziness, on Imipramine for depression for a few years now. BP of 140/90, RR 22/min, HR variable rhythm. Cardiac monitor shows: what will you do? a. Immediately start IV lidocaine bolus and drip b. Cardiovert the patient with 200 J c. Administer IV amiodarone d. Give IV magnesium sulfate bolus and drip
  • 34. 6. You are making rounds with your OB consultant. In front of you is a 35 week AOG pregnant patient that has been having elevated blood pressure since 2 days ago. Right in front of you, the patient had a seizure. Blood pressure was taken and BP = 200/110mmHg. What do you do? a. Give captepril25mg tab SL stat q15 minutes b. Send the patient to the ICU for further monitoring and start her on Duvadilan drip c. Give magnesium sulfate to the patient d. Start oral ACE inhibitors 7. 60 year old male suddenly had dizziness and diaphoresis. BP is 80/50mmHg with cold clammy extremities. ECG shows the following. What to do? a. IV streptokinase b. Give verapamil 5mg IV stat c. Send to cathlab immediately d. Provide electrical cardioversion
  • 35. 8. 45/M, hypotesive , cold and clammy with EKG below: True about this patient’s condition: a. If right-sided V3 and V4 leads show 1mm ST depression, it indicates RV infarction from proximal occlusion of the night coronary artery b. Elevated JVP with kussmaul’s sign and clear lungs are consistent with RV infarction. IV fluids dobutamine and reperfusion therapy are appropriate c. ST-elevation or right-sided V3 and V4 leads, elevated JVP and pulmonary , congestion should be treated with IV fluids, dobutamine and reperfusion therapy d. Intraaortic ballon counterpulsation is contraindicated 9. 1. administer CPR 2. check for responsiveness 3. call for help,activate EMS 4. survey the scene 5. check pulse and breathing
  • 36. a. 3-4-5-2-1 F b. 4-2-1-3-5 F c. 4-2-3-5-1 T d. 1-2-3-4-5 F e. 2-4-3-1-5 F 10. A 58/M, smoker, hypertensive , diabetic patient with dyslipidemia consulted at your clinic. Based on the NCEP guidelines, the LDL target for this patient should be: a. <200mg/dL F b. <150mg/dL F c. <100mg/dL F d. <70mg/dL T e. <50mg/dL F 11. 60/F w sudden dyspnea 2 hrs after BK amputation for diabetic foot ulcer. BP 150/100, HR 96/min, RR 22/min soft S1, cold, clammy extremities. ECG shows: Best treatment option is: a. IV r-tPA thrombolysis b. IV streptokinase thrombolysis c. Cardiac catherization and angioplasty/stenting d. IV streptokinase followed by cardiac catherization
  • 37. 7. Which of the following statements is/are TRUE of connective tissue diseases? F a. Systemic lupus erythematosis and dermatomyocitis can be differentiated through skin biopsy and direct immunofluorescence findings T b. red plaques on the knuckles of the hands are characteristic of dermatomyocitis F c. Presence of malar rash distinguishes lesions of SLE from dermatomyocytis F d. perifungual erythema are common in scleroderma F e. Scleroi…is 7. Which of the following is NOT characteristic of rosacea? a. Flushing b. Comedones c. Red papules d. Telangiectasia 9. Which of the following xanthomas is the LEAST specific cutaneous sign of hyperlipidemia? a. Xanthelasma b. Planar xanthoma c. Eruptive xanthoma d. Tuberous xanthoma
  • 38. Oral antihyperglycemic agents that reduce postprandial hyperglycemia a. Alpha glucocidase inhibitor b. Arrythin agonist c. Bile and sequestrant d. Megitnide e. Thiazolidne Antidiabetic agents contraindicated in patients with elevated plasma triglycerides a. Alpha gluccosidase inhibitor b. Bile acid sequestrants c. d. GLP-1 receptor agonist e. Thiamidinadione Differentiates HHS from DKA a. Presence of nausea and vomiting b. Anion gap c. Serum potassium d. More pronounced volume e. Higher mortality rate
  • 39. Microvascular complications of diabetes a. Cataracts b. Diabetic skin spots c. Macular edema d. e. Sexual dysfunction Mechanism for the development of chronic DM complications a. Epignhetic charges in the affected cells b. Non erythmatic formation of advanced glycosylation and products c. Increased glucose metabolism in the sorbital pathway d. Increased e. Formation of diacylglycerol leading to activation of protein phase C True of diabetic nephropathy a. Diabetic patients with nephropathy usually have concomittant neuropathy b. Only c.
  • 40. True of diabetic neuropathy a. Smoking is a risk factor b. Is a diagnosis of exclusion c. Most common form is distal symmetric polyneuropathy d. For polyneuropathy pain is usually present at most and worse at night e. is a FDA-approved treatment True according hypoglycemia a. Most common cause are drugs b. During
  • 41. Associated with low radioiodine update a. grave’s disease b. Plummer’s disease c. d. e. hypothyroidism True of myeoderma coma a. Still has highmortality rate despite treatment b. Almost always occurs in the elderly c. Hyperventilation plays a major d. External e. Characteristics of thyroiditis a. Acute thyroiditis is rare and due to b. Diagnosis of c. Silent thyroiditis occurs in patients with underlying autoimmune thyroid disease d. Cytoxine induced thyroiditis is more common in women with TPD antibodies e. Thyroid dysfunction is
  • 42. Presentation of amodarone-induced a. b. c. d. e. Characteristics of goiters a. b. c. d. e. Increases risk for osteoporosis a. Dementia b. Alcoholism c. Anorexia-nervosa d. Type 1 DM e. Rheumatoid arthritis
  • 43. Causes of hypocalcemia in the critically ill patients a. Acidosis b. Blood transfusion c. Heparin d. Hypoalbuminuria e. Pan Patients with assymptomatic hyperparathyroidism whoare candidates for surgery a. 55 year old male b. BMD T score of -2.0 c. Creatinine d. 24H urinary calcium 3 g/day e. Serum calcium 16 mg/dl
  • 44. Finding consistent with phaeochromocytomas a. Highly vascular and tumors b. Most all c. Palpitations, headaches and produce sweating d. Documentation of e. Liberal salt intake and hydration are necessary treatments Features of adrenal insufficiency a. Most common cause is autoimmune adrenalties b. Secondary adrenal insufficiency presents with loss of glucoccorticoid and androgen secretion c. Distinguishing feature of primary adrenal insufficiency is hyperpigmentation in areas of friction d. Hypercalcemia is found at presentation in 80% of patients with primary adrenal insufficiency e. Random serum cortisol measurements are limited diagnostic value
  • 45. Features of acromegaly a. Due to GH and AOF -1 hypersecretion the after epiphysis has cloased b. Calorie c. Prolcatin is elevated in 21% d. Screening is done by getting random iOF-1 hormone levels e.
  • 46. Differentiates ectopic ACTH secretion from ACTH-secreting pituitary tumor a. Gender of patient b. Rapidly of onset c. Levels of 24H urine free cortisol d. Marked hypokalemia e. High dose dexamethasone test Characteristics of diabetes insipidus a. Due to a decreased secretion of action of AVP b. Overt dehydration on physical exam c. “Bright spot” noted on T1-weighted midsagital MRI images d. Weight loss occurs with water deprivation e. Desmopresssin is appropriate for D1
  • 47. Treatment of SIADH a. Fluid intake that is 500 cc less than the urine output increases serum sodium by 12% per day b. IV infusion of 3% hypertonic saline connects the sodium deficiency and removes excess water by solute diureses c. Demeclocycline produces a reversible form of nephrogenic diabetes insipidus d. Blocks the antidiuretic effect of AVP on V2 and V1s receptors e. Too rapid correction can be complicated by cerebral ionizing Definite benefits of HRT a. Cognitive dysfunction b. Diabetes mellitus c. sypmtoms d. Osteoporosis e.
  • 48. Insulin biosynthesis secretion and action a. C peptide is cleared more slowly than insulin and is a useful in differentiating endogenous from ecogenous insulin occurs b. Diagnosis and screening for diabetes a. Diabetes is defined as the level at which diabetic specific complications occur b. FPG is the most reliable and convenient test for identifying assymptomatic individuals HPG or HBsAg c. Diagnosis of diabetes can be withdrawn when the glucose incidence reverts back to normal d. Screening is recommended for both type 2 DM Factors that confer a very high risk of progression from prediabetes to diabetes a. Age < 60 years b. Family history of DM in any relative c. Hypertension d. Elevated total cholesterol
  • 49. Pathogenesis of type 1 DM a. Patients will have evidence of islet directed autoimmunity b. In the majority, immunologic markers appear the triggering event and before diabetes appears c. Major susceptibility gene for type 1 DM is located in the HLA region of chromosome d. Most individuals with diabetes will not have a relative with this disorder Treatment goals for adults with type 2 DM a. HBA 1c < 6.9% in most patients b. Postgrandial capillary glucose < 110mg/dl c. Triglyceride < 160 mg/dl d. BP < 120/80 mmHg Nutrition and physical activity in patients with diabetes a. The components of optimal medical nutrition therapy are generally similar for T1 DM, T2 DM and the general population b. Nonnutrient sweeteners are allowed c. Insulin and SUg patients are very prone to hypoglycemia during exercise d. Proliferative diabetic retinopathy is a contraindication to vigorous exercise
  • 50. Rapid acting insulin a. Aspart b. Detemir c. Glargine d. NPH Explains the absence of ketosis in HHS a. Insulin deficiency b. Lower levels of Differentiates DKA from single hyperglycemia a. Glucose levels b. Ketonemia c. Ketonuria d. Plasma osmolarity
  • 51. Features of DKA a. Insulin deficiency is necessary for diabetic Laboratory abnormalities of DKA a. total body stores of sodium, potassium, chloride and phosphorus are total body by their serum levels b. Elevated serum creatinine c. Elevated serum amylase is indicative of salivary in origin Metabolic acidosis is from ketone bodies and lactic acid Possible unifying mechanism in the theories regarding the development of chronic diabetic a. Epignetic charges in affected cells b. Formation of advanced glycosylation c. Increased production of growth factors d. Increased production of reactive oxygen appear in the melochondria
  • 52. Correct conclusions from trials on glycemic control and development of chronic complications a. In the Diabetes Control and Complications Trial (DCCT) reduction in chronic hyperglycemia can prevent microvicular complications in patients with type 2 DM b. Benefits from improved glycemic control during the trials persisted even after the study concluded and glycemic control worsened in both the DCCT and in the United Kingdom Prospective Diabetes Study (UKPDS) c. Benefits of an improved glycemic control occurred only in the tighter ranges of A1c values d. In UKPDS strict blood pressure control significantly reduced microvascular complications but had no effect on microvascular complications Features of opthalmologic complications of diabetes a. Blindness is primarily the result of progressive diabetic retinopathy and the development of cataracts b. Proliferative diabetic retinopathy is characterized by retinal vascular microaneurysm, biot hemorrhages and cotton-wool spots c. Hypertension is also a risk for the development rethropathy d. Genetic susceptibility
  • 53. Management of diabetic retinopathy a. There is in established diabetic nephropathy in the first 6 – 12 months of improved glycemic control transient worsening b. Toutine nondilated eye examination by the primary care provider or diabetes specialist is detect diabetic eye disease opthalmologist needed c. Exercise has not been conclusively shown to worsen proliferative diabetic nephropathy d. Aspirin therapy 650 mg/day Differentiates nephropathy in T1 DM from nephropathy of T2 DM a. Microalbuminuria may be present on diagnosis b. More commonly with accompanying hypertension c. Presence of microalbuminuria is predictive of progression of microalbiminuria d. Albuminuria may be secondary to factors unrelated to the
  • 54. Cardiovascular morbidity and mortality in patients with diabetes a. Screening for coronary heart disease in assypmtomatic individuals with diabetes controversial have not yet shown clinical benefit b. In both DCCT and UKPDS, cardiovascular events were reduced during the trial and remained low at follow up 10-17 years later trend for benefit in the first part of the trial c. Current recommendations target A1 c levels near to reduce cardiovascular events d. Aspirin therapy (75-162 mg/day) is primarily recommended for secondary prevention Characterisitc of diabetic dyslipidema a. Most common pattern is high triglyceride, low HDL c, small dose LDL b. DM does not increase levels of LDL c. Beneficial effects of LDL reduction are similar in diabetic and nondiabetic populations d. High dose niacin may worsen glycemic control and insulin resistance
  • 55. Antihypertensive agents which can increase insulin resistance a. Ace inhibitor b. Calcium channel blockers c. Central adrenergic antagonists d. Thiazide Features of infectious & dermatologic complications in patients with diabetes a. Organisms that cause pulmonary infections are similar from that found in the general population b. Infections seen almost exclusively in the diabetic population include malignant otitis externa c. “diabetic skin spots” result from minor trauma in the pretibial region and are more common in elderly d. Necrobiosis tipodice diabeticorum predominantly affects young females with type 1 DM
  • 56. Which of the following emerging therapies is an important therapeutic option for patients with T1DM a. Whole pancreas transplantation b. Panccreatic islet cell transplantation c. SGLT-2 inhibitors d. Banatric surgery True regarding DM management in special situations a. Target blood glucose levels in critically ill patients is 140-180 mg/dl b. Total parenteral nutrition increases insulin requirements c. Oral antihyperglycemia agents be will not efficacious for patients on glucoccorticoids with FPG > 200 mg/dl d. The most crucial period of glycemic control during pregnancy is during the second term
  • 57. Primary phyysiologic defense of the body against hypoglycemia a. Decrease in insulin b. Increase in glucagon c. Increase in epinephrine d. Decrease in cortisol Treatment of hypoglycemia a. Reasonable initial dose of glucose is 20g b. Glucagon is effective for of hyoglycemia ineffective for glycogen-depleted individuals also less useful for t2dm c. Medical therapy with diazonide orr octreotinide is useful for endogenous hyperinsulinemia from beta-cell tumors also for nontumor beta cell disorders d. Administrations of uncooked cornstarch at bedtime may be necessary in some patients Laboratory test for thyroid dysfunction a. Exception, TSH alone is sufficient to exclude a primary abnormality of thyroid fx b. Thyroxine-binding globulin are increased by estrogen androgens decrease TBG c. TSI antibodies are mainly used to predict thyrotoxicosis fetal d. Salicylates can transiently increase free thyroid hormone levels
  • 58. Features of thyroid hormone resistance syndrome a. Autosomal disorder due to a mutation in the TRB gene dominant b. Usuallyasypmtomatic c. Can present with tachycardia d. Most patients not require levothyroxine replacement Management of hypothyroidism a. FT3 determination in diagnosis & management 25% with normal FT3 reflecting adaption deiodenase responses b. Liothyronine has no place for long term replacement c. Patients who develop hypothyroidism after treatment of Grave’s disease require replacement doses of LT4 d. Full relief from symptoms may take 3-6 months after normalization of TSH Special treatment considerations of hypothyroidism a. Levothyroxine treatment of clinically euthyroid patients with thyroid autoantibodies can reduce the risk of preterm delivery b. Levothyroxine dose requirements even in pregnancy increased by ≥ 50% c. Elderly patients require 20% less levothyroxine than younger patients d. Emergency surgery until euthyroidism is achieved generally safe
  • 59. Features of Autoimmune hypothyroidism a. More common in populations with chronic exposure to a low iodine diet b. TPO and thyroiglobulin antibodies have a primary pathogenetic role and are clinically useful markers of thyroid autoimmunity c. Pericardial effusions occur in up to 30% and commonly leads to cardiac compromise d. Thyroid-associated opthalmopathy occurs in 5% of patients Causes of secondary hyperthyroidism a. Gestational thyrotoxicosis b. Silent thyroiditis c. Struma ovani d. Iodine excess
  • 60. Features of Graves disease a. Minor for grave’s disease, major for the opthalmopathy smoking is a risk factor for Grave’s disease b. Worsens in 3-6 mos., plateaus in 6-12 mos. Climical course of the opthalmopathy that of the thyroid disease c. TSI antibodies do not correlate directly with thyroid hormone levels d. May present with pretibial myxedema Treatment of Graves disease a. All antithyroid drugs inhibit the function of TPO reduce thyroid antibody levels, and enhance remission rates b. FT4 is used as a basis to the titrate doses of the antithyroid drugs c. PTU has a prolonged radioprotective effect and must be stopped several weeks before radioactive treatment d. It is often possible antithyroid drugs during the first term of pregnancy
  • 61. Characteristic of sick euthyroid syndrome a. Most common pattern is low T3 syndrome b. Major causes is cytokines c. Progressive HIV disease is associated with a decrease in T3 d. Magnitude of fall in T3 levels correlates with the severity of the illness Factors altering thyroid function in pregnancy a. Rise of circulating hCG in the first term is accompanied by a reciprocal fall in TSH b. Estrogen induced rise in TBG during the first trimester c. Increased thyroid hormone metabolism by the placenta d. Increases urinary iodide excretion Characteristics of thyroid cancer a. Papillary thyroid cancer has a propensity for spread b. Follicular thyroid cancer be diagnosed by thyroid FNAB c. Medullary thyroid cancer is sensitive to RAI d. Anaplastic thyroid cancer is always stage IV
  • 62. Most common cause of drug-induced osteoporosis a. Alcohol b. Glucoccorticoids c. Heparin d. levethyroxine Management of Osteoporosis a. Primary use of biochemical markers is to predict fracture risk independently of bone density monitoring to reaponse b. Preferred source of calcium is dietary sources c. Weight-bearing exercises in postmenopausal women prevents bone loss but does not appear to result in substantial gain of bone mass d. Supplementation with vitamin K are helpful Pharmacologic therapy for osteoporosis that promotes bone formation a. Bisphosphonates b. Calcitonin c. Denosumab d. Teriparitide
  • 63. Causes hypercalcemia and low serum PTH levels a. granulomulous disease b. Lithium toxicity c. Parathyroid hyperplasia d. Tertiary hyperparathyroidism Features of hyperparathyroidism a. Hereditary hyperparathyroidism may be found in MEN-2A and MEN-1 b. Most common cause is parathyroid adenoma c. More than half are asymptomatic d. Incidence peaks between the 3rd and 4th decade Approach to a patient with an adrenal incidentaloma a. Diagnostic evaluation should be done for > 1 cm adrenal masses b. For the diferentation of benign from malignant adrenal masses, imaging is relatively sensitive though specificity is suboptimal c. FNA to rule out malignancy, to establish malignancy need to demonstrate mets may cause needle canal metastase of ACC d. Adrenal masses with confirmed hormone excess and usually treated surgically
  • 64. Features of adrenocortical cancer a. 60-70% overproduce hormones and produce clinically syndromes, usually inapparent due to defective synthesis b. Metastasis most commonly occurs to the liver c. Tumor size > 8 cm is associated with a high risk of recurrence d. Five years survival rates are 30-40% Principles of estrogen therapy in menopause a. Oral estrogen is better than transdermal estrogen in managing hot flushes b. It is useful in the management of hypertriglyceridemia c. It is useful in patients with thromboembolic disease d. It increases the risk of endometrial cancer Characteristics of infertility a. Defined as inability to conceive after 12 months of unprotected sexual intercourse b. Female factors account for 58% of infertility c. Modifiable risk factors include alcohol caffeine, obesity, and smoking d. Time for evaluation, correction and expectant movement can be longer in women > 35 years.
  • 65. Who has diabetes? a. 40F with fasting blood sugar of 126 mg/dL taken once b. 35M with prebreakfast cbg 130 mg/dL c. 28F with 50 g OGCT 150 mg/dL d. 18M with a standardized HBA 1c of 6.8% 29F was admitted for decrease
  • 66. 30F is for fertility work up. She is assymptomatic and PE is unremarkable. Her TSH is 7 uU/L, (NV 0.4 – 4 uU/L) and FT4 is 13 uU/L (NV 12 – 20 uU/L) TPO antibodies are positive. What should you do next? a. Request for a total T3 b. Request for thyroid scan c. Repeat TSH, FT4 after 3 months d. Start levothyroxine 26M with weakness of both thighs was noted to have a TSH 0.01 uU/L (NV 0.4 – 4 uU/L) and FT4 20 uU/L (NV 12 – 22 uU/L). Physical examination is unremarkable. What should you do next? a. Request for total T4 b. Request for free T3 c. Start PTU 50 mg TID PO d. Reassure the patient and ask him to follow up after 6 – 12 months
  • 67. 45M has 2 cm firm nodule on the right thyroid lobe on palpation THS is uU/L (NV 0.4-4 uU/L) FNAB shows follicular tumor. What should you do next? a. Do a thyroid scan b. Do an ultrasound guided FNAB c. Monitor by ultrasound d. Send the patient for surgery Who should be given medical therapy for osteoporosis? a. Z score -2.0 b. 30F with bronchial asthma who was given Prednisone 30 mg OD PO x 7 days c. 60F with BMI 16 and T score -1 d. 62M with family history of leg fracture 25F with BP 160/100 consulted for work up. She is on losartan 50 mg OD PO and amlodipine 5 mg OD PO. Serum creatinine is 90 ug/dL. Na 140 meq/L, K 3.2 meq/L. what should you do need? a. Request for plasma aldosterone-renin ratio b. Get on abdominal CT scan c. Correct serum potassium first d. Start spirinolactone 50 mg OD PO
  • 68. 28F complains ofamenorrhea in 6 months. Work up showed a fasting serum orolactin 150 ug/L. She has a history of thyroid surgery 2 years ago. Currently she is not taking any medications. Physical examination is unremarkable except for dry thick skin. What should you do advice the patient? (HPIM 18th and chapter 139) a. Repeat prolactin levels using sample dilution b. Request for THS and FT4 c. Request for a sellar FT4 d. Start bromocriptiine 2.5 mg/tab ½ tab OD PO MTF 1. Features of Erythropoiesis F a. the first morphologically recognizable erythroid precursor in the bone marrow is themyeloblast F b. In the absence of Granulocyte Colony Stimulating Factor (GCSF), erythroid precursors willnundergo programmed cell death (apoptosis) T c. EPO governs day-to-day production of red cells T d. Average life span of red cells is 100-120 days T e. The fundamental stimulus for EPO production is the availability of O₂ for tissue metabolic needs
  • 69. MTF 2. Oxygen affinity og hemoglobin in different situations T a. Hemoglobin has a lower oxygen affinity among acidotic septic patients F b. Hemoglobin has a lower oxygen affinity at higher pH (Bohr effect) T c. Immediately after transfusion of PRBC, there may be decrease in O2 release by the hemoglobin F d. Increase in 2,3 BPG increases hemoglobin oxygen affinity F e. Better oxygen delivery is noted ay higher attitudes MTF 3. Differential diagnosis of microcytic hypochromic anemia T a. Thalassemia T b. Lead Poisoning T c. Iron deficiency Anemia F d. Diphyllobotrium latum infestation T e. Anemia of chronic disease MTF 4. Compatible with Iron deficiency anemia F a. Microcytic hypochromic red cells low serum iron, normal ferritin level T b. Microcytic hypochromic red cells, low serum iron, low ferritin level F c. Microcytic hypochromic red cells, normal serum iron, increased ferritin F d. Microcytic hypochromic red cells, normal serum iron and TIBC T e. Microcytic hypochromic red cells, low serum iron, high
  • 70. MTF 5. Microcytic hypochromic cells with low serum iron and normal or increased bone marrow iron stores may be found in the following: F a. 32 y/o female with alpha thalassemia trait F b. 19 year old heavily menstruating woman with iron deficiency anemia T c. 55 year old male with rheumatoid arthritis F d. 28 y/o male with chronic lead positioning T e. 49 year old female with systemic lupus erythematosus MTF 6. Features of hypoproliferative anemias T a. Acute and chronic inflammation is the most common of these F b. Anemia of inflammation, like iron deficiency, is related in part to abnormal erythropoietin response T c. Renal disease, cancer and hypometabolic states are characterized by an abnormal erythropoietin response to the anemia T d. These abenia are assocciated with normocytic and normochromic red calls and an inapppropriately low reticulocyte response F e. Most commonly associated with exposure to benzene derva
  • 71. MTF 7. Diseases associated with intravascular hemolysis T a. Paroxysmal nocturnal hemoglobinuria F b. Hemolytic uremic syndrome T c. Hemoglobinupathies F d. Hypersplenism F e. Microangiopathic hemolytic anemias MTF 8. Characteristic of Unsatable Hemoglobin Variants T a. Should be suspected in patients with nonimmune hemolytic anemia, jaundice, splenomegaly, or premature billiary tract disease. Or leg ulcers F b.Severe hemolysis usually presents in adults F c. Spontaneous mutation ucommon T d. The peripheral blood smear often show anisocytosis, abundant cells with punctate inclusions, and irregular shapes (i.e., poikilocytosis). T e. Best test for diagnosing unstable hemoglobins are the Heinz body preparation and the isopropanol or heart stability test.
  • 72. MTF 9. Guidelines on splenectomy in patients with hemolytic diseases T a. Avoid splenectomy in mild cases F b. delay splenectomy until at least 10 years of age T c. Antipneumococcal vaccination before splenectomy is imperative F d. Anticoagulation may be required. T e. Hereditary Spherocytosis patients often may require cholescystectomy MTF 10. Characteristic of Vitamin B12 and associated deficiency resulting to Megaloblastic Anemia T a. Cobalamin (Vitamin B12) is synthesized solely by microorganisms. T b. Only source for humans is food of animal origin, e.g. meat, fish, and dairy products. F c. Body stores are of the order of 2-3 mg, sufficient for 3-4 months if supplies are completely cut off F d. Vegetables, fruits and other foods of non-animal origin also contain cobalamin T e. Most common among individuals of Hindu religion
  • 73. MTF 11. The M Component noted in Serum Protein Electrophoresis may be seen in ff condition/s T a. multiple Myeloma T b. Chronic lymphocytic leukemia F c. Thalassemia F d. Chronic renal disease T e. Breast Cancer MTF 12. Characteristics of Multiple myeloma T a. Malignant proliferation of plasma cells derived from a single clone T b. Overexpression of myc or ras genes hass been noted in some cases F c. Inversion of chromosome 16 was also observed in certain varieties T d. Management with Thalidomide and Dexamethasone have shown better survival F e. Translocation of chromosome 9:22 is associated with worst prognosis
  • 74. MTF 13. The following conditions predispose a patient to development of Acute Myelogenous Leukemia T a. Myelodysplastic Syndrome T b. Polycythemia Vera F c. Thalassemia T d. Paroxysmal Nocturnal Hemoglobinuria F e. Iron Deficiency Anemia MTF 14. The following are part of the initial laboratory evaluation of adult patients with acute myeloid leukemia T a. CBC with manual differential cell count F b. Ultrasound of whole abdomen T c. Bone marrow aspirate and biopsy (morphology cytogenetics, flow cytometry, molecular studies) T d. HLA typing of patient, siblings, and parents for potential allogeneic SCT F e. Urinalysis
  • 75. MTF 15. Ccharacteristic of myelophthisic anemia T a. A common finding in the peripheral blood smear is a shift to the left of the granulocytic series and presence of nucleated red cells, these findings are suggestive of the diagnosis T b. Usually the infectious or malignant underlying processes are obvious in secondary causes F c. Leukopenia F d. F e. MTF 16. Conditions with pancytopenia and cellular bone marrow: F a. Aplastic anemia T b. Paroxysmal nocturnal hemoglobinuria T c. Myelodysplasia F d. Dyskeratosis congenita T e. Hairy cell leukemia MTF 17. Drugs definitely reported to cause isolated thrombocytopenia F a. Acetyl salicylic acid T b. Trimethoprim/sulfamethoxazole T c. Danazol T d. Paracetamol F e. Phenylpropanolamine
  • 76. MTF 18. Consequences of Neutropenia MTF 19. treatment of Immune Thrombocytopenic Purpura (ITP) F a. Patients with platelet counts greater than 30,000/L should be started on oral steroids T b. Traditional OPD management of ITP this has been prednisone at 1 mg/kg T c. Rh₀(D) immune globulin must be used only in Rh-positive patients T d. Refractory ITP may benefit from anti-CD 20 agents T e. Oral eltrombopag shows potential benefits for acute ITP MTF 20. The following principles should be applied in the management of bleeding disorders F a. Rest T b. Ice or cold compress over affected areas F c. Warm compress over affected areas T d. Elevation of affected extremity F e. Aspiration of bleeding joints
  • 77. MTF 21. Features of venous thrombosis F a. Majority of cases are secondary to hereditary causes F b. Females are at greater risk for venous thrombosis than are males T c. High prevalence of Factor V Leiden was reported in Arabs F d. factors 8 and vWF are decreased during pregnancy thus predisposing to venous thrombosis T e. LMWH is recommended for acute thrombotic events MTF 22. Conditions associated with both arterial and venous thrombosis T a. Hyperhocystenemia T b. Paroxysmal nocturnal hemoglobinuria F c. Immobilization F d. Activated protein C T e. Polycythemia vera
  • 78. MTF 23. In the setting of acute thrombosis or anticoagulation, working up for a hypercoagulable state should take into consideration: T a. Coumadin reduces protein C and S levels T b. Heparin can reduce antithrombin levels F c. Heparin and Coumadin have no effect on testing for lupus anticoagulant and APC T d. Sepsis is associated with reduction in levels of protein C, Protein S,, antithrombin F e. Work ups may be done anytime MTF 24. Indications forHematopoietic Stem cell transplantation (HSCT) T a. Acute Myelogenous Leukemia T b. Breast Ca T c. Severe combined immunodeficiency T d. Very Severe Aplastic Anemia F e. Diffuse B Cell Lymphoma, newly diagnosed 1. a. b. c. d. The world Health Organization (WHO) defines anemia as Hemoglobin level <130g/L (13 g/dL) in men Hemoglobin level <120g/L (12 g/dL) in men Hemoglobin level <130g/L (13 g/dL) in women Hemoglobin level < 120g/L (13 g/dL) in pregnant women
  • 79. 3. The main iron transport protein a. Transferrin b. Apoferritin c. Albumin d. globulin 4. The “gold standard” for iron stores determination a. Bone marrow iron stain b. Serum ferritin c. Serum iron d. Total Iron Binding Capacity (TIBC) 5. Definitive management may be achieved through splenectomy a. Hereditary spherocytosis b. G6PD deficiency c. Thalassemia d. Sickle cell hemoglobinopathy
  • 80. 6. History of pallor and tea colored urine after being given anti malarials and eating legumes or beans best fits this disease: a. Pyruvate kinase deficiency b. GCPD deficiency c. Spherocytosis d. Fancom’s anemia 7. Most effective management of G6PD a. Splenectomy b. Gene therapy c. Blood transfusion d. Avoidance of drugs which may induce hemolysis 8. Disease almost always associated with positive direct Coomb’s test: a. PNH b. G6PD deficiency c. Acute hemolytic reaction sec to ABO incompatibility d. Idiopathic autoimmune hemolysis
  • 81. 9. Hallmark of the common forms of alpha thalassemia: a. Elevated Hgb A b. Elevated Hgb A2 c. Elevated HgbF d. Low HgbA2 11. Hemoglobin levels in the elderly population a. Almost all the elderly population have normal hemoglobin level b. The female gender is more prone to nutritional type of anemia c. About 25% of the elderly without underlying disease will have below normal hemoglobin levels d. About 30% of the male gender with anemia would be diagnosed with underlying malignancy 12. Vegetarians are prone to anemia secondary to which of the following deficiency a. Folic acid b. Cyanocobalamin c. Niacin d. Zinc
  • 82. 13. Fundamental abnormality in pernicious anemia a. Short bowel syndrome b. Atrophic gastritis c. Lack of Vitamin B12 in diet d. malabsorption 14. Chronic renal diseases where erythropoietin production is spared a. Chronic glomerulonephritis b. Polycystic kidney disease c. Chronic pyelonephritis d. Diabetic nephropathy 15. Compatible with Anemia of Chronic disease a. Microcytic hypochromic red cells, low serum iron, increased ferritin level b. Microcytic hypochromic red cells, normal serum iron, low ferritin c. Microcytic hypochromic red cells, normal serum iron and TIBC d. Microcytic hypochromic red cells, low serum iron, high TIBC
  • 83. 16. Agent 17. Mechanisms of action of cyclosporine in aplastic anemia a. Reduce cytotoxic T cells b. Inhibits IL 2 production by T lyhmphocytes c. Modulates adverse reactions to ATG d. Stimulate production of erythropoietin 18. Specific treatment of marrow aplasia in severe aplastic anemia a. Allogeneic stem cell transplantation b. Autologous bone marrow transplantation c. Cyclosporine d. Blood transfusion 19. Hallmark clinical feature PNH a. Ineffective eryhtropoiesis b. Chronic intravascular hemolysis c. Bone marrow failure d. Bleeding diathesis
  • 84. 20. Clinical featured of idiopathic Thrombocytopenic Purpura in adults: a. More common in men than women b. Onset is acute c. Spontaneous remission in 2% d. A and C 21. Treatment for assymptomatic patients newly diagnosed with ITP with platelet count of 50,000: a. Oral prednisone 1 to 2 mg/kg/day b. IVIg c. IV hydrocortisone d. Observation 22. Mild Von Willebrand’s disease type 2 patient for surgery is best managed with a. DDAVP b. Cryoprecipitate c. Factor VIII/VWF concentrates d. Factor VIIa
  • 85. 23. 19 year old male with 3% Factor 8 assay activity is compatible with a. glanzmann’s thrombasthenia b. Hemophilia A, mild c. Hemophilia A, moderate d. Hemophilia B, mild 24. What is the probability that a hemophilia A male married to a carrier will have a carrier female offspring (Chapter 116)? a. 25% b. 50% c. 75% d. 100% 25. Treatments of choice for hemophilia A patients with inhibitors (Chapter 116) a. Factor Vlla b. Factor VIII, recombinant c. Cryoprecipitate d. Tranexamic acid
  • 86. 26. Central mechanism of disseminated intravascular coagulation (Chapter 116): a. Massive platelet activation b. Uncontrolled plasmin generation c. Excessive thrombin generation d. hyperfibrinolysis 27. Most sensitive test for disseminated intravascular coagulation a. Platelet count b. Fibrinogen level c. D-dimer d. Fibrinogen degradation products 28. Post partum hemorrhage is a major cause of maternal mortality with disseminated intravascular coagulation. With consumption of fibrinogen as sequelae resulting to bleeding, this may be replaced with (Chapter 113) a. Whole Blood b. Cryoprecipitate c. Cryosupernate d. Tranexamic acid
  • 87. 29. The following have been observed among patients who inherited thrombophilias a. Often present with unusual sites of thrombosis b. In majority of cases, thrombosis is provoked by surgery, pregnancy, immobilization OCP, HRT or old age c. Those heterozygous for factor V Leiden and prothrombin gene mutation have been shown to have a higher rate of recurrent venous thrombosis verses the general population d. B and C 30. The following situation does not predispose to venous thrombosis (Chapter 117) a. Increasing age b. Varicose veins c. Intake of melphalan in multiple myeloma d. Trauma 31. trousseau’s syndrome may be noted in which of the following (Chapter 117): a. Gastric Ca b. Prolonged best rest c. Trauma to blood vessels d. Intake of Coumadin
  • 88. 32. Recommended anticoagulant prophylaxis for high risk activities like prolonged air travels etc., for patients with thrombophilia (Chapter 118): a. Aspirin b. Warfarin c. UFH d. LMWH 34. Standard initial treatment for deep vein thrombosis (Chapter 118): a. UFH b. LMWH c. Argatroban d. Fondaparinux 35. Pharmacologic characteristics of warfarin (Chapter 110): a. The onset of action is immediate b. The antithrombotic effect depends on the reduction of the functional levels of the clotting factors with longest half lives which are Factors II and X. c. It is rapidly and almost completely absorbed from the kidney d. Anticoagulation effect best for acute DVT
  • 89. 36. Treatment of choice for thrombotic thrombocytopenic Purpura (Chapter 175) a. TpE b. Cryosupernate c. Steroids d. Aspirin 37. Associated with ADAMTS 13 deficiency a. Hemolytic uremic syndrome b. Thrombotic thrombocytopenic purpura c. Essential thrombocythemia d. Heparin induced thrombocytopenia 39. The following is not a major indication for splenectomy in primary myelofibrosis (Chapter 108) a. Massively enlarged spleen b. Excessive transfusion requirements c. Portal hypertension d. Severe thrombocytopenia 40. Molecular pathology of CML (Chapter 104)
  • 90. 41. Characteristics of CML compared with the other Chronic Myeloproliferative disorder (P.vera, Idiopathic Myelofibrosis, Essential Thrombocythemia) (Chapter 108, 109) a. Total WBC count > 30,000/ul in >90% b. Platelet count > 750,000/ul in >50% c. Increased red cell mass and plethora d. JAK 2 mutation 42. The tyrosine kinase inhibitor Imatinib Mesylate (Chapter 109) a. Initial therapy in almost all CML, chronic phase b. Improved survival outcomes among CML patients in accelerated phase c. Patients treated with imatinib after IFN failure had better survival than those who continued receiving IFN d. May be given as an alterative to autologous hematopoietic stem cell transplantation
  • 91. 43. Best treatment option for accelerated and blast crisis CML (chapter 109) a. Chemotherapy b. Allogeneic stem cell transplant c. Autologous stem cell transplant d. Imatinib mesylate, higher dose 44. Most common site of relapse in ALL a. CNS b. Ovary c. Bone marrow d. Testes in males 45. Classic triad of multiple myeloma a. Marrow plasmacytosis (>10%), lytic bone lesions, and serum and/or urine M component b. Anemia, lytic bone lesions and serum and. Urine M component c. Marrow plasmacytosis (>10%), reverse albumin/globulin ratio, and serum and/or urine M component d. Anemia, marrow plasmacytosis (>20%), reverse albumin/globulin ratio
  • 92. 46. Major specific pathophysiologic mechanism in MDS (Chapter 107) a. Defective maturation and death of marrow precursor cells b. Fibrosis associated with PDGFR c. Bone marrow suppression which may be associated with increased cytokine production d. Bone marrow infiltration 47. Treatment of choice for elderly patients with MDS, RAEB (Chapter 107) a. Lenolidamide b. Intensive chemotherapy c. Azacytidine d. Arsenic trioxide 48. Management of platelet refractoriness in alloimmunized bleeding patient with severe thrombocytopenia,with sepsis (Chap 113) a. Random donor platelets at 1 unit/10 kg body weight, treat sepsis b. Apheresed Platelets, steroids, treat sepsis c. HLA matched Apheresed Platelets treat sepsis d. Apheresed platelets and IV IG
  • 93. A 1. A 26 year old female was stabbed with an ice pick several times over the right anterior chest wall and was brought to the Emergency Room by her boyfriend. Vital signs: BP=80 palpatory, HR=128/min, weak thready pulse RR=28/min. patient appeared paper white with blood oozing over puncture wounds. For blood transfusion requirements, whole blood (WB) was requested however only blood components were available at the blood bank. What components can be combined in place of whole blood? a. Packed red blood cells (PRBC) + fresh frozen plasma (FFP) + platelets b. PRBC + FFP c. PRBC + Cryosupernate d. PRBC + Cryosupernate + platelets A 2. the above patient’s laboratory CBC hemoglobin-42 g/L, wbc-12.3, platelet count142,000; PT = 13.2 secs control 12.9, PTT = 40 secs control=36, Blood type was A Rh positive. The patient is best managed with: a. Crystalloids/colloids + A Rh positive PRBC b. Crystalloids/colloids + A Rh positive PRBC and platelet count c. Crystalloids/colloids + A Rh positive FWB d. Crystalloids/colloids + O Rh positive FWB
  • 94. A 3. A 38 year old male presents with hgb 76 g/L, MCV of 105fl, platelet count 80,000/ul, low level of serum folate ands a normal serum level of vitamin B12. this is most compatible with: a. Pregnancy b. Alcoholism c. Pernicious anemia d. Resection of ileum A 4. A 56 year old male, diabetic, was admitted with peripheral arterial occlusive disease affecting his toes on both lower extremities. A few days after starting Enoxaparine subcutaneously, his platelet count was noted to be 20 x 109/L. what would be the best management for this patient? a. Continue Enoxaparine and observe patient b. Discontinue Enoxaparine and observe patient c. Discontinue enoxaparine and start Unfrationated Heparin IV with monitoring of INR d. Discontinue Enoxaparine and start heparinoids
  • 95. A 5.An Assymtomatic 72 y/o Multiple Myeloma patient with renal insufficiency with hemoglobin at 94 g/dl, platelet count =102,000 is best managed with a. PRBC transfusion b. Leukocyte depleted PRBC transfusion c. Whole Blood d. Erythropoietin A 6. An assymptomatic 22 year old woman was seen at the OPD requesting clearance for work, history revealed regular monthly cycles, wherein she utilizes 12-14 maternity napkins, most fully soaked. Physical examination (except forpallor)was unremarkable. Labs: CBC-Hgb=72 g/L, hct ==0.22, wbc=7.5 (seg 68%, lympho=30%), plt ct = 450, MCV=65, mch=18, RDW=21Serum ferritin =6.7 What is best management for the anemia? a. Oral iron supplement b. Intravenous iron c. Blood transfusion d. Improve diet
  • 96. A 7. A 36 year old male, asymptomatic consulted because of incidental finding of platelet count at 850,000 on CBC (Hgb=138, wbc=18.5)during annual medical. Pas medical history, other labs and PE unremarkable. If the patient was further worked up and showed still elevated plt count at 600,000 mild leukocytosis, positive JAK 2 mutation assay and negative bcr-abl, what would be best management for the patient? a. Observation and monitoring b. Start anagrelide and hydroxyurea c. Start imatinib d. allogeneic bone marrow transplant A 8. A 62/M, previous total gastrectomy for NHL 7 years ago, came in because of easy fatigability of 4 months duration. CBC: Hb 5.2 HCT 25 MCV 125 WBC 3.5 Seg 54 Ly 46 Platelets 78; Retic count 1.2% PBS: Macricytosis, aisopoikilocytosis WHAT CLINICAL SITUATION COULD EXPLAIN THE FINDINGS? A. Iron deficiency anemia B. Vitamin B12 deficiency C. Sideroblastic anemia D. Myelopthisic anemia
  • 97. 2. In a patient with metabolic alkalosis from vomiting, this urine parameter serves as an accurate indicator of volume status: a. Sodium b. Chloride c. Potassium d. Specific gravity 6. This chinese herb present in slimming tea preparations is also believed to be the cause of Balkan nephropathy a. Aristolochic acid b. Ochratoxin A c. Melamine d. cadmium 8. Components involved in providing gastroduodenal defense against peptic ulcers • T A. Unstirred layer of mucus and bicarbonate • T B. Surface epithelial cells that generate heart shock proteins • T C. Cell renewal from mucosal progenitor cells • T D. Submucosal vascular system that generates alkaline tide • T E.Prostaglandins that promote epithelial cell restitution
  • 98. 42. Primary prophylaxis of esophageal variceal hemorrhage • T A. Propanolol • T B. Nadolol • T C. Variceal band ligation • F D.Transjugular intrahepatic portosystemic shunt (TIPS) • F E. Surgical esophageal transection 43. Radiographic findings present in acute intestinal obstruction • T A. Stepladder pattern in small intestine • T B. Absence of colonic gas • T C. Coffee bean-shaped mass • F D. Free air in the peritoneum • F E. “Thumbprinting”
  • 99. 44. Diagnosis of diverticulitisis best made on CT with these findings • T A. Sigmoid diverticula • T B. Thickened thickened colonic wall >4mm • T C. Inflammation within pericolic fat • F D. String of beads sign • F E. Bird’s beak sign 45. Treatment for stage II hemorrhoids, which are protrusions with spontaneous reduction • T A. Fiber supplementation • T B. Cortisone suppository • F C. Sclerotherapy • F D. Banding • F E. Operative hemorrhoidectomy
  • 100. 46. Colorectal cancer screening strategies for asymptomatic individuals ≥ 50 years of age • T A. Colonscopy every 10 years • T B. Fecal DNA testing every 3 years • T C. CT colonography every 5 years • T D. Flexible sigmoidoscopy every 5 years • T E. Double-contrast barium enema every 5 years 47. Epidemiologic characteristics of ulcerative colitis • T A. Smoking is preventive • F B. Oral contraceptive pills increase risk • T C. Appendectomy is protective • T D. More common in African Americans than asians • T E. Has two peak ages of onset
  • 101. 48. Pathologic feature of Crohn’s disease • T A. Can affect the mouth and anus • T B. Cobblestone appearance • T C. Perirectal fistulas and anal stenosis • F D. Almost always involves the rectum • F E. Disease process is limited to the submucosa 49. Pathologic features of ulcerative colitis • F A. “Skip areas” or areas of histologically normal mucosa are common • T B. Pseudopolyps may be present • F C. Disease process is transmural • F D. “Creeping fat” or thickened mesentry encasing bowel is a common feature • F E. Granulomas are a pathognomonic feature
  • 102. 50. Extraintestinal manifestations of inflammatory bowel disease more commonly seen in Crohn’s disease than in ulcerative colitis • F A. Pyoderma gangrenosum • T B. Ankylosing sspondylatis • F C. Primary sclerosing cholangitis • T D. Nephrolithiasis • T E. Reactive amyloidosis 51. Indications for surgery of ulcerative colitis • T A. Toxic megacolon • F B. Refractory fistula • T C. Colon cancer prophylaxis • T D. Colon dysplasia • F E. Perianal disease unresponsive to medical therapy 1. • • • • Most important indication for urgent gastrointestinal endoscopy in the list below A. Age>60 Years B. Decrease in hemoglobin C.Decrease in hematocrit D. Orthostatic hypotension
  • 103. 2. What is the most sensitive and specific approach to testing for peptic ulcer disease? • A. Double-contrast barium study • B. Urea breath test • Serum gastrin level • D. Endoscopy 3. Which type of chronic atrophic gastritis is associated with pernicious anemia? • A. Isolated granulomatous gastritis • B. Antral predominant or type B gastritis • C. Multifocal atrophic gastritis • D. Autoimmune or type A gastritis 4. Risk of rebleed is lowest with this type of peptic ulcer • A. ulcer with spurting • B. ulcer with visible vessel • C. ulcer with pigmented spot • D. clean-based ulcer
  • 104. 5. Preferred endoscopic therapy for control of active esophageal variceal bleeding • A. Band ligation • B. Sclerotherapy • C. Balloon tamponade • D. Transjugular intrapatic portosystemic shunt (TIPS) 6. Large-caliber arteriole that runs immediately beneath the gastrointestinal mucosa and bleeds through pinpoint mucosal erosion • A. Gastroesophageal varix type I (GOV1) • B. Mallory-Weiss tear • C. Gastric antral vascular ectasis (GAVE) • D. Dieulafoy’s lesion 7. Sight, smell and taste of food are components of this phase of stimulated gastric and secretion • A. cephalic • B. Gastric • C. Pancreatic • D. Intestinal
  • 105. 8. This cell is responsible for gastric acid secretion • A. Chief cell • B. G cell • C. Parietal cell • D. ECL cell 9. Ulcers within 3cm of the pylorus, commonly accompanied by high gastric acid production and duodenal ulcers • A. Type I gastric ulcers • B. Type II gastric ulcers • C. Type III gastric ulcers • D. Type IV gastric ulcers 10. This type of gastritis produced by chronic H. pylori infection may lead to development of MALT lymphoma • A. Antral-predominant gastritis • B. Nonatrophic pangastritis • C. corpus-predominant atrophic gastritis • D. Fundal gastritis
  • 106. 11. Dose of aspirin deemed safe, as it does not cause peptic ulceration • A. 75mg/day • B. 81mg/day • C. 325mg/day • D. no dose is completely safe 12. Most commonly encountered diagnosis among patient’s seen for upper abdominal discomfort • A. Nonulcer dyspepsia (NUD) • B. Gastroesophaegeal reflux disease (GERD) • C. peptic ulcer disease (PUD) • D. Nonerosive reflux disease (NERD) 13. Cyto preotective agent used in peptic ulcer disease that may lead to aluminum-induced neurotoxicity in patients with chronic renal insufficiency • A. Bismuth subsalicylate • B. Misoprostol • C. Rebamipide • Sucralfate
  • 107. 14. Cytoprotective agent used in peptic ulcer disease contraindicated in women who may be pregnant • A. Sucralfate • B. Colloidal bismuth subcitrate • C. Misoprostol • D. Amitriptyline 15. Proven benefit of H. pylori eradication in patients with peptic ulcer disease • A. Decreases in ulcer recurrence • B. Elimination of risk of ulcer perforation • C. Prevention of gastric cancer • D. Prevention of gastric lymphoma 16. Most feared complication with amoxicillin use in triple therapy of peptic ulcer disease • A. Black tongue • B. Pseudomembranous colitis • C. Allergic reaction • D. Hepatotoxicity
  • 108. 17. Most common cause of treatment failure in compliant patients on triple therapy for peptic ulcer disease • A. underdosing • B. Antibiotic-resistant strains • Drug side effects leading to discontinuation • Lower bioavailability of generic drugs 18. Test of choice for documenting H. pylori eradication • A. Stool antigen test • B. Urea breath test • C. Serologic testing • D. Histopathologic exam 19.Definition of a refractory peptic ulcer • A. Gastric ulcer that fails to heal after 12 weeks • B. gastric ulcer that fails to heal after 6 weeks • C. Duodenal ulcer that fails to heal after 4 weeks • D. Duodenal ulcer that fails to heal after 2 weeks
  • 109. 20. Surgical treatment of choice for a gastric antral ulcer • A. Antrectomy, intraoperative ulcer biopsy, vagotomy • B. Subtotal gastrectomy with Roux-en-Y • C. Antrectomy with a Billroth I anastomosis • D. Ulcer excision with vagotomy and drainage 21. Cornerstone of therapy for patients with dumping syndrome after surgery for peptic ulcer disease • A. Antidiarrheal medications • B. Octreotide • C. Anticholinergic agents • Dietary modification 22. Physical exam finding which indicates the presence of a severe necrotizing pancreatitis a. Abdominal distenstion b. Jaundice c. Pleural effusion d. Blue discoloration around umbilicus
  • 110. 23. Lab value consistent with acute pancreatitis a. Threefold elevated serum lipase b. Fourfold elevated salivary-type amytase c. Hypoglycemia d. hypercalcemia 24. Type of viral hepatitis for which no vaccine exists a. HAV b. HCV c. HDV d. HEV 25. Endovascular stenting is a management option for this type of acute intestinal ischemia • A. Aterial embolus • B. Arterial thrombosis • C. Venous thrombosis • D. Non-occlusive mesenteric ischemia
  • 111. 26. This is the gold standard for the diagnosis and management of acute intestinal arterial occlusive ischemia • A. mesenteric duplex scan • B. CT angiography • C. Lapatoramy • D. Ablation of renin-angiotensin axis 27. This is the most common extrauterine condition requiring abdominal operation during pregnancy • A. neprolithiasis • B. Small-bowel obstruction • C. perinephric abscesses • D. Appendicitis 28. Most common cause of hematocheza in patients >60 years • A. inflammatory bowel disease • B. Colon Cancer • C. Colonic diverticulitis • D. hemorrhoids
  • 112. 29. Which laboratory test is a highly sensitive and specific marker for detecting intestinal inflammation in inflammatory bowel disease? • A. Fecal lactoferin • B. C-reactive protein • C. Erythrocyte sedimentation rate • D. Platelet count • E. Fecalysis 30. In which variant of inflammatory bowel disease does total parenteral nutrition play a major role in inducing remission? • A. Distal ulcerative colitis • B. Extensive ulcerative colitis • C. Inflammatory Crohn’s disease • D. Crohn’s disease with TB ileitis • E. Fistulizing Crohn’s disease 1. • • • • A 40-year-old male with melena is found to have a gastric ulcer with a visible vessel on endoscopy. Which treatment strategy is recommended? A. Discharge home with oral PPI therapy B. ward admission, no IV PPI therapy, no endoscopic therapy C. ICU admission, IV PPI therapy, endoscopic therapy D. Ward admission, IV PPI therapy, endoscopic therapy
  • 113. 2. A 22-year-oldman had hepatitis B serologies done as part of his seafarer’s preemployment workup. Which serologic pattern assures that his childhood vaccination series remain effective? a. HBsAg(+), anti-HBc(+), anti-HBs(-) b. HBsAg(+), anti-HBc(+), anti-HBs(+) c. HBsAG(-), anti-HBc(-), anti-HBs(-) d. HBsAG(-), anti-HBc(-),anti-HBs(+) 3. Your 50-year old uncle is in the hospital for hepatitis. His serologies are (+)HBsAG, (+)IgM anti-HAV, (+)IgM anti-HBc, (-) anti-HCV. What is your interpretation? a. Acute hepatitis B b. Acute hepatitis A and B c. Acute hepatitis A superimposed on chronic hepatitis B d. Acute hepatitis C 4. A 48-year-old man with HBeAg-negative chronic hepatitis B asks to be treated. His viral load is 1,500 IU/mL and his ALT is normal. This would be a reasonable recommendation: • A. Treatment with pegylated interferon • B. treatment with entecavir • C. referral for liver transplantation • D. No treatment
  • 114. 5. A 40-year-old man was admitted from the ER with the following test results, prothrombin time 18 (N=12), serum bilirubin 12mg?dL. Which is the best treatment for his alcoholic hepatitis? • A. High dose thiamine • B. Silymarin • C. IV multivitamins • D. Prednisolone 6. A 50-year-old first degree relative of patient diagnosed with colon cancer at age 65 should undergo colonoscopy this often. • A. every 10 years • B. every 5 years • C. every 3 years • D. every year
  • 115. 6. Deficiencies of the Adaptive Immune System: T a. Severe combined immune deficiency (SCID) T b. Hyper-IgE syndrome (autosomal dominant) T c. Hyper IgM syndrome T d. Common variable immunodeficiency (CVID) F e. Chronic granulomatous disease (CGD) 7. Pathogenesis of Drug Reactions T a. Untoward cutaneous responses can arise as a result of immunologic mechanisms. T b. Immunologic reactions require toward activation of host immunologic pathways and are designated as drug allergy. T c. Drug reactions occurring through nonimmunologic mechanisms may be due to activation of effector pathways, ovedosage, cumulative toxicity, side effects, drug interactions, metabolic alterations, exacerbation of preexisting dermatologic conditions or inherited protein or enzyme deficiencies. T d. Increase in molecular size is associated with increased immunogenicity. F e. Anaphylaxis is more common among oral medications than IV medications.
  • 116. 15. Extent of epidermal detachment in Stevens-Johnson Dyndrome: a. <5% body surface area b. <10% body surface area c. <15% body surface area d. <20% body surface area 1. A 60 year old male with a 50 pack year smoking history consults for chronic cough, hemophylis, weight loss and exertional dysnea. You consider lung cancer. Which of the following would be consistent with his case? T a. A chest x-ray would likely reveal a lung lesion that is central in location. T b. The most likely histologic subtype is going to be squamouss cell carcinoma. F c. The histologic subtype plays in the most important prognostic factor in non-small cell lung cancer. F d. This patient will most likely benefit from anti-angiogenic agents added to systemic therapy. T e. The main etiology factor associated with every histologic subtype of lung cancer is cigarette smoking.
  • 117. 2. Clinical manifestations of lung cancer. T a. Hemoptysis is a manifestations of endobronchial tumor growth. F b. Symptoms of cough, hemoptysis, and stridor are examples of symptoms caused by peripherally growing lesions. T c. Shoulder pain, destruction of the 1st snd 2nd ribs, and involvement odf the 8th cervical and 1st and 2nd thoracic nerves are features of a Pancoast’s syndromes. F d. A Cushing-like sundrome is an uncommon paraneoplastic syndrome associated with adenocarcinomas T e. Non-small cell lung cancers have a predilection for causing metastatis but these don’t cause adrenal insufficiency. 3. A 66 year old female with adenocarcinoma of the lung is found to have a 6 x 5 cm tumor a the lower lobe of the right lung on the CT scan. Mediastinal adenopathy is detected as well as an enlarged right supraclavicular node. Liver and adrenals are free of tumor. Bone scan reveals multiple enhancing lesions at the thoracic and lumbar vertebrae. T a. The tumor may be labelled as T2 F b. Mediastenal adenopathy makes this an N1 lesion. T c. The presence of palpable supraclavicular nodes makes this a N3 staged disease. F d. Because the liver and adrenals are free of tumor this can be labelled as MO. F e. A 5-year survival rate of > 40% is expected with systemic therapy even in the presence of abnormal bone scan.
  • 118. 4. Critical dates in a woman’s life that have a major impact on the incidence of breast cancer: T a. Age of menarche F b. Age of marriage T c. Age of first full-term pregnancy F d. Age of first lactation T e. Age of menopause 5. In the evaluation of breast masses in women, which of the following is applicable? T a. In the absence of certain risk factors cannot be used to avoid the need for biopsy in women with palpable breast masses. F b. In a woman with palpable breast mass, MRI features are diagnostic and may avoid the need for biopsy. T c. Diagnostic mammography defines the procedure done after the discovery of a palpable breast mass in order to evaluate the rest of the breast prior to a biopsy. F d. In pregnant women, the occurrence of a dominant breast mass is most likely due to hormonal changes, thus avoiding the need for a biopsy. T e. Women whose biopsies reveal benign lesions without atypia have little risk of developing cancer.
  • 119. 6. Prognostic factors in breast cancer. T a. Tumor stage T b. ER and PR status T c. Her 2 status T d. S phase function T e. Histologic grade 7. Risk factors in colorectal cancers. T a. Diet high in animal fat F b. Low fiber diet T c. Inflammatory bowel disease F d. Female sex T e. Lynch syndrome 8. Recommended screening procedures for colorectal cancer. F a. Digital rectal exam T b. Feccal occult blood test T c. Flexible sigmoidoscopy F d. Abdominal CT scan F e. Whole body PET scan
  • 120. 9. Predictors of prognosis in patients with colorectal cancer. F a. Age T b. Grade of the tumor T c. Depth of tumor invasion F d. EGFR mutation status T e. Lymph node involvement 10. Clinical features of Gastric cancer. T a. Elderly patients have a history of chronic ulceration with the histopathology most likely an intestinal type of adenocarcinoma. F b. H. pylori increases the risk for gastric cancer by causing mutations in a critical set of tumour suppressor genes. F c. Dysphagia and early satiety are symptoms due to intestinal types of lesions that may involve the entire stomach. T d. Intestinal type lesions occur most frequently in the antrum and lesser curvative of the stomach. F e. The lung is the most common site of hematogenous spread.
  • 121. 11. Etiologic factors believed to be associated with esophageal cancer. T a. Chronic alcohol intake T b. Radiation induced strictures F c. Vitamin C deficiency F d. Chronic intake of cold beverages T e. Fungal toxins in pickled vegetables 12. Risk factors associated with Hepatocellular cancer. T a. Poor hepatitic cirrhosis T b. Ingestion of poorly stored grains F c. Cigarette smoking F d. History of Hepatitis A F e. Juvenile diabetes mellitus
  • 122. 13. A 60 year oldmale alcoholic with a 2 week history of jaundice is noted to have a hard nodular mass on liver palpation. Serum AFP is 500 IU/L. with regards to his management , which of the following isapplicable? T a. The best procedure for evaluating tumor vascularity and size is a triphasis CT scan of the abdomen. T b. Pain is the most common presenting symptom in high risk areas of liver cancer. F c. an alpha feto protein level of > 1000 IU/L is diagnostic for HCC. F d. Radiofrequency ablation or ethanol injection can be very useful for tumor near the main portal areas. F e. The multikinase inhibitor Erlotinib has been found to improve median survival by 3 months in patients with advanced diseas. 14. The clinical presentation of pancreatic cancer. F a. Courvorsier’s sign involves a dilated portal vein in a patient with carcinoma of the pancreatic head T b. Pain is the most frequent symptom in patient’s with tumors of the pancreatic body F c. Elevated CA 19-9 levels are diagnostic of pancreatic cancer but carry no prognostic implications. F d. A detailed clinical staging evaluation provides important therapeutic and prognostic information in patients with advanced disease. F e. 5-fluorouracil + Folinic acid remain the initial combination of choice in patients with metastatic disease.
  • 123. 15. Clinical characteristics of renal cell carcinoma. T a. the proximal tubules are the most common sites of origin of clear cell carcinomas. F b. the strongest risk factor associated with tumor development remains a positive family history. F c. the classic triad of hematuria, flank pain, and flank mass can be seen in more than half of patients. F d. clear cell carcinoma is associated with the inhibition of the epidermal growth factor receptor gene resulting in tumor angiogenesis. T e. agents that inhibit angiogenesis may be useful in the management of metastatic disease. 16. Clinical characteristics of transitional cell cancers of the genito-urinary tract. T a. Polychromatropism, the ability to recur in new sites along the urothelial tract, is a common clinical manifestation T b. Smoking increases risk for developing urothelial carcinomas. T c. Once node positive disease is detected, the patientis labelled as having stage IV disease. F d. The bladder should be suspected as the most common site of microscopic hematuria. F e. In stage IV disease, cure may still be possible in the presence of visceral metastase.
  • 124. 17. Clinical manifestations of prostate cancer. F a. PSA screening is the main reason behind the marked decline in mortality rates. F b. Bicalumatide has been shown to improve survival in the chemo-prevention of prostate cancer. T c. PSA screening has resulted in treatment of many low-grade tumors that would otherwise not have been clinically significant. F d. Elevated PSA is diagnostic for the presence of prostatic carcinoma. T e. the main function of trans-rectal ultrasound is directing prostate biopsies rather than staging. 18. Risk factors for head and neck cancers. T a. Cigarette smoking F b. Age > 60 years T c. Low consumption of fruits and vegetables T d. Human papilloma virus F e. Eat Asian race
  • 125. 19. Management of head and neck cancers. T a. Localized (T1 and T2) squamous cell cancer can be treated with surgery of RT with curative intent T b. Despite toxicity, concomitant chemo-RT has a better survival rate compared to sequential therapy in advanced disease. T c. Concomitant chemo-RT produces better survival rates and allows for organ preservation in advanced head and neck cancers compared to RT alone. F d. Concomitant chemo RT can produce similar benefits in patients with metastatic disease compared to chemotherapy alone squamous cell cancers. F e. Cis-retinoic acid can result in regression of leukoplakia and prevents occurrence of second primary sites. 20. Clinical features of ovarian cancer. T a. the use of oral contraceptives has been associated with a decreased risk of epithelial ovarian cancers. T b. pelvic pain is a common symptom of ovarian cancer and may help identify early stage disease. F c. screening with annual CA-125 determination is expected to improve survival and to identify early stage disease. T d. the most common type of epithelial ovarian tumors is the serous type. T e. prognostic factors for ovarian cancer include tumor stage, tumor grade and extent of residual disease after surgery
  • 126. 21. Clinical features of cervical cancer. T a. the most important etiologic agent is the venereal transmission of the human papilloma virus T b. PAP smear is capable of detecting up to 95% of early cervical lesions F c. the presence of a mass in the cervix is the most common sign of cervical cancer F d. the presence of hydronephrosis immediately classifies a patient as having stage IV disease T e. Tumors that extend beyond the area of the true pelvis are labelled as stage IV disease 22. Paraneoplastic metabolic syndromes and their associated malignancies. F a. Hypercalcemia - Osteosarcomas T b. Cushing’s syndrome – Carcinoid tumors T c. SIADH – Squamous cell CA, lung F d. Hypoglycemia –Adrenal medulla tumors T e. Diarrhea – Pancreatic cancer 23. Paraneoplastic hemotologic syndromes and their associated malignancies. T a. Erythrocytosis – Renal cell cancer T b. Thrombocytosis – Lung cancer T c. Granulocytosis – Hodgkin’s lymphoma F d. Eosinophilia – Neuroendocrine tumors T e. Thrombophlebitis – Breast cancer
  • 127. 24. Clinical features of malignant melanoma. T a. Adults have a higher risk for development compared to children. F b. well-defined boarders are more characteristic of melanoma than benign nevi. F c. The most important prognostic factor for melanoma is tumor size. T d. of the characteristics that can differentiate melanoma from other pigmented lesions, the weakest is diameter of the lesion F e. uniformly dark pigmentation is associated more than benign nevi. 2. What procedure would you advise to a 50 year old female who would want to undergo screening for breast cancer? a. Breast MRI b. Mammography c. Self-breast examination d. Self-breast + annual clinical breast exam
  • 128. 3. A 65 year old woman with breast cancer post-MRM was discovered on pathologic analysis to have a 4 x 3 cm tumor in her left breast. 7 of the 14 isolated axillary nodes are positive for malignancy. Liver US, chest x-ray and bone scan are all normal. What would her stage be? a. I b. II c. III d. IV 4. In which of the following conditions would a biologic immune therapy be indicated as part of the adjuvant treatment of early stage breast cancer? a. Estrogen receptor positive b. Progesterone receptor positive c. Her2/neu receptor positive d. Triple negative
  • 129. 5. Which of the following signs or symptoms is the result of a peripherally located tumor in lung cancer? a. Clubbing b. Pleuritis chest pain c. Hemoptysis d. Facial edema 6. Which situation would argue strongly for proceeding with the resection of a asymptomatic pulmonary nodule? a. Male sex b. Presence of calcification on chest x-ray c. Stable findings on serial chest x-ray d. Presence of atelectasis 7. In which of the following situations is curative surgery or radiotherapy no longer possible in a patient with non-small cell lung cancer? a. A 5 cm tumor 4 cm from the carina b. N1 nodal involvement c. Ipsilateral mediastinal node involvement d. Paralysis of the vocal cords
  • 130. 8. Which of the following is an acceptable strategy for chemotherapy in advanced non-small cell lung cancer? a. Patients with good performance status should be treated with single agents b. Non-platinum doublets are the new standard of care in these situations c. Cisplatin or carboplatin remains a standard component of doublet chemotherapy d. The concomittant administration of tyrosine kinase inhibitors with standard chemotherapy is superior to either used alone 9. Which of the following is a recognized risk factor of carcinogen for salivary gland cancer? a. Cigarette smoking b. Alcohol intake c. Human papilloma virus d. None identified 10. Which of the following is recognized as an acute toxicity of RT in head and neck cancers? a. Mucositis b. Immobilization of the tongue c. Loss of taste d. xerostomia
  • 131. 11. Which of the following subtypes is the most aggressive in melanoma? a. Desmoplastic b. Acral lentiginous c. Nodular d. Lentigo maligna 12. Which of the following is considered a standard form of therapy in metastatic melanoma? a. Dacarbazine b. Interferon c. Cisplatinum d. Interleukein-2 13. Which of the following is consistent with a more aggressive type of basal cell cancer? a. Superficial subtype b. Small lesions c. Located on the scalp d. Pigmented lesions
  • 132. 14. What is an expected response of tumor blood vessels to treatment with bevacizumab, a potenta anti-VEGF monoclonal antibody? a. Inhibition of dendritic cell action b. Inhibition of vascular permeability c. Growth of dilated and leaky vessels d. Sprouting of endothelial cells from host vessels 15. Which of the following is important in enhancing the efficacy of radiation against cancer cells? a. Cold b. Heat c. Oxygen d. Iron 16. In which of the following tumors is cure still possible with chemotherapy even in an advanced stage of the disease? a. Endometrial cancer b. Testicular cancer c. Breast cancer d. Follicular lymphomas
  • 133. 17. Which of the following is an example of a biologic form of therapy? a. Aromastase inhibitors b. Interferon c. Anti-tumor antibiotics d. gefitinib 18. In which of the following conditions would routine G-CSF prophylaxis against febrile neutropenia be required during the first cylce of chemotherapy? a. Conventional dose regimen b. Performance status of 3 c. Age of 50 years d. Adjuvant chemotherapy for breast cancer 19. Hyponatremia as a paraneoplastic syndrome may be seen in which of the following? a. Renal cell cancer b. Rhabdomyosacoma c. Small cell lung cancer d. Non-Hodgkin’s lymphoma
  • 134. 20. Eosinophilia is a paraneoplastic syndrome usually associated with which of the following? a. Gastric cancer b. Breast cancer c. Lymphomas d. Paancreatic cancer 21. Which of the following sarcomas arises from the interstitial cells of Cajal and are dependent on the c-kit oncogene for malignant transformation? a. Leiomyorsarcome b. Primitive neuroectodermal tumor c. Gastrointestinal stromal tumor d. Ewing’s sarcoma 22. Which of the following is a known characteristic of metastatic bone lesions? a. Osteolytic lesions have a higher incidence of hypercalcemia b. Osteolytic lesions produce higher levels of serum alkaline phosphatase c. Osteoblastic lesions are best diagnosed by plain radiographs d. Osteoblastic lesions are associated with the elaboration of parathyroid hormone related peptides
  • 135. 23. An 80 year old female is diagnosed to have peritoneal carcinomatosis from an epithelial ovarian cancer. Which of the following is the best treatment option? a. Paclitaxel-platinum combination b. Intraperitoneal chemotherapy c. Pelvic exanteration followed by chemo-RT d. Targeted agents 24. Which of the following is a known risk factor for endometrial cancer? a. Multiple partners b. Multiparity c. Obesity d. Late menarche 25. Which of the following is the molecular target of the E& protein of HPV 16 which may results in the development of cervical cancer? a. P53 b. Rb c. Bcl-2 d. K-ras
  • 136. 26. According to the American Cancer Society guidelines, when should PAP smear screening begin? a. Menarche b. After the 1st pregnancy c. After the 1st live birth d. Start of sexual activity 27. In which situation should SVC syndrome be considered a true medical emergency? a. Facial edema b. Jugular vein engorgement c. Dyspnea d. dysphagia 28. What is the most common location of esophageal adenocarcinomas? a. Oropharyngeal portion b. Upper third c. Middle third d. Lower third
  • 137. 29. A 75 year old male, chronic smoker, is diagnosed to have an unresectable esophageal cancer and complains of severe dysphagia and anorexia. What is the most promising management approach? a. Radiotherapy b. Chemotherapy + RT c. Endoscopic laser therapy d. Repeated endoscopic dilatation 30. Which of the following is consistent with the intestinal type of gastric adenocarcinomas? a. Found mostly in the cardia b. Presents as linitis plastica c. Ulcerative in presentation d. Usually seen in younger patients 31. Which of the following represents a pre-malignant lesion of colorectal cancer? a. Hamartoma b. Hyperplastic polyp c. Adenomatous polyp d. Juvenile polyp